Anda di halaman 1dari 101

MARCH CURRENT AFFAIRS_2019

PRELIMS QUESTION BANK


(DAILY TEST COMPILATION)

March 2019

Page No.1
MARCH CURRENT AFFAIRS_2019
2019-03-29 from January 2017 till December 2017 under swachh
Economy Industry Bharat Mission-urban (SBM-u)
1) Which of the following is/are correct with regards to The objective of ‘swachh survekshan-2018’
Confederation of Indian Industries? To encourage large scale citizen participation and
1.The Confederation of Indian Industry (CII) is a create awareness amongst all abo sections of society
business association in India importance of working together towards making towns
2. CII correlates with Government, NGOs, and civil and cities a better place to live in. Additional the
society to help industry flourish effective programs survey also intends to foster a spirit of healthy
for social development competition among towns and cities to improve their
service delivery to citizens, towards creating cleaner
3.The membership is open to a company or a firm in
cities.
India engaged in manufacturing activity only
The focus of ‘ swachh survekshan-2018’ been shifted
Select the correct answer using the codes given below
from process and output based indicators to
A) 1 and 2 only B) 2 and 3 only indicating on outcome and sustainability.
C) 1 and 3 only D) 1, 2 and 3 The survey attempts to capture the progress in
Key : A following 6 broad parameters-
Explanation : 1. Collection and Transportation of Municipal Solid
• The Confederation of Indian Industry (CII) is a Waste: to ensure that segregated dry and wet waste is
business association in India. collected daily from the households and our public
• The CII Theme for 2018-19 is ‘India RISE: Responsible, areas are clean
Inclusive, Sustainable and Entrepreneurial. 2. Processing and Disposal of Municipal Solid Waste: to
• CII correlates with Government, NGOs, and civil encourage cities to process their waste erever
society to help industry flourish effective programs possible recycle the dry waste.
for social development. It provides numerous stances 3. Sanitation related progress: to verify whether city is
through which representatives can undertake on ODF with access to toilet available for citizens this
various issues, such as gender equality, microfinance, year even all Petrol Pumps of the country are coming
development of backward districts, HIV/AIDS and forward to offer their toilets as public Toilet.
public health. 4.IES (Information,Education and Communication):
• The membership is open to any company or firm in Whether cities have started campaigns promoting
India engaged in manufacturing activity or providing Swachh Survekshan, engaging citizens in waste
consultancy services also. management, maintenance of community and lets etc.
Objective : To know about the Confederation of Indian 5. Capacity Building: to assess whether officials in the
Industries ULBs were provided sufficient opportuni and trainings
Governance Governmenpolicies and and go for exposure visits.
programs 6. Innovation and Best Practices: This component has
2) Which of the following are the criteria for selecting been introduced first time in this survey encourage
the cities for ranking for Swachh Sarvekshan? cities s to come forward and share their best practices
in Swachh Bharat Mission. It will help the arn how
1. Open defecation-free
our cities have responded the call for Make India
2. Transportation and processing of garbage clean and ODF by october 2019.
3. Education and capacity building Criteria: The criteria for selecting the cities for ranking
4. Field inspection 5. Citizen feedback for this year included 45% marks for cities becoming
Select the correct answer using the code given below open defecation-free; solid waste management like
A) 1 and 2 only B) 1, 2 and 3 only sweeping, collection, transportation and processing of
garbage; education and capacity building. 25% marks
C) 1, 2 and 4 only D) 1, 2, 3, 4 and 5
were given for field inspection and another 30% marks
Key : D were given for citizen feedback. In total, feedbacks from
Explanation : at least 18 lakh citizens across the surveyed cities about
• An October, 2015 the Swachh Bharat mission team in the sanitation were received.
the Ministry of Housing and urban affairs (MoHUA) Objective : To know about the Swachh Sarvekshan
conceptualized and introduced a healthy competition rankings and criteria to select the cities under it.
‘swachh survekshan’ Among cities to improve the 3) Which of the following languages is/are considered
status of urban sanitation in India. as official language in United Nations
• Subsequently, Swachh shan-2016 survey for the rating 1.Russian 2.French
of 73 cities in January 2016 and ‘Swac ekshan 2017’
3. Swedish 4.English
• Ranking 434 cities was conducted in January February
5.Arabic 6.Mandarin
2017 Now. MoHUA now proposes to conduct its third
survey to rank all 4041 cities based on assess o gress Select correct answer using code given below

Page No.2
MARCH CURRENT AFFAIRS_2019
A) 1,2 and 4 only B) 2,3,4 and 6 only whi le The Debt Recovery Tri bunal (DRT) are
C) 1,2,4,5 and 6 only D) 1,2,3,4,5 and 6 adjudicates insolvency resolution for companies.
Key : C Select the answer using code given below
Explanation : A) 1 only B) 2 only
UN is an international organization founded in 1945 C) Both 1 and 2 D) Neither 1 nor 2
after the World War II. Its mission is to maintain Key : A
international peace and security, developing friendly Explanation :
relations among nations and promoting social • The Insolvency and Bankruptcy Code (IBC) was enacted
progress, better living standards and human rights. in 2016 to facilitate a time-bound resolution for ailing
HQ - New York and sick firms.It could either be through closure or
Official Languages - Arabic, Chinese, English, French, revival,while protecting the interests of creditors.
Spanish, Russian Members - 193 (Latest Member South
Sudan) • The National Company Law Tri bunal (NCLT)
Objective : adjudicates insolvency resolution for companies.The
To know about the basic facts about UN Debt Recovery Tribunal (DRT) are adjudicates
Environment Ecology insolvency resolution for individual
Biodiversity Objective :
4) Consider the following statements with respect to the To know about the Insolvency and Bankruptcy Code
“Hump Backed Mahseer”: (IBC)
1. The Humpback is the largest known species of International Organisations
Mahseer and is only found in the Kaveri river basin. 6) Which of the following countries are Permanent
2. Recently the fish has been added to the IUCN Red members of the United Nations Security Council
List as Critically Endangered. (UNSC)?
Which of the statement(s) given above is/are correct? 1. The United States of America
2. The United Kingdom
A) 1 only B) 2 only 3. China 4. France
C) Both 1 and 2 D) Neither 1 nor 2 5. The Russian Federation
Key : C Select the correct answer using the code given below:
Explanation : A) 1, 2, 4 and 5 only B) 1, 3 and 5 only
• The Humpback (or) Tor remadeviiis the largest known C) 1 and 5 only D) 1, 2, 3, 4 and 5
species of Mahseer and, across the world, is only Key : D
found in the Kaveri river basin.
Explanation :
• The effects of construction of dams, regulated flows,
deforestation, drought, pollution and sediment • The Security Council has primary responsibility for
transport has a great toll on this river water species. the maintenance of international peace and security.
It has 15 Members, and each Member has one vote.
• Recently the fish has been added to the IUCN Red List Under the Charter of the United Nations, all Member
as Critically Endangered States are obligated to comply with Council decisions.
• This fish is also called as tiger of the water found in • The Security Council takes the lead in determining the
Pambar, Kabini and Bhavani rivers of the Karveri existence of a threat to the peace or act of aggression.
basin in the states Karnataka, Kerala and Tamil Nadu. It calls upon the parties to a dispute to settle it by
Objective : peaceful means and recommends methods of
To know about the Hump Backed Mahseer, which was adjustment or terms of settlement. In some cases, the
recently added to IUCN’s critically endangered species Security Council can resort to imposing sanctions or
list. even authorize the use of force to maintain or restore
international peace and security.
2019-03-28 PERMANENT AND NON-PERMANENT MEMBERS
Economy Fiscal Policy The Council is composed of 15 Members:
5) Which of the following statements with respect to the Five permanent members: China, France, Russian
Insolvency and Bankruptcy Code (IBC) Federation, the United Kingdom, and the United States,
1.The Insolvency and Bankruptcy Code (IBC) was and ten non-permanent members elected for two-year
enacted in 2016 to facilitate a time-bound resolution terms by the General Assembly (with end of term year):
for ailing and sick firms Belgium (2020)
2.The National Company Law Tri bunal (NCLT) Côte d’Ivoire (2019)
adjudicates insolvency resolution for individuals Dominican Republic (2020)
Equatorial Guinea (2019)

Page No.3
MARCH CURRENT AFFAIRS_2019
Germany (2020) • MLAT is an agreement b/w two nations for the aim of
Indonesia (2020) collecting and exchanging information in an attempt
Kuwait (2019) to implement public laws or criminal laws.
Peru (2019) • Useful in tax matters (in particular as part of
international DTAA. This assistance may take the form
Poland (2019) of examining and identifying people, places and
South Africa (2020) things, custodial transfers, and providing assistance
Objective : with the immobilization of the instruments of criminal
To know about the United Nations Security Council activity.
(UNSC) How would it help?
International Relations • The Mutual Legal Assistance Treaty (MLAT) will enable
International Institutions, mutual assistance in prevention, investigation and
agencies and their structure prosecution of crime, service of summons and other
judicial documents, execution of warrants and other
and mandate
judicial commissions and tracing, restraint, forfeiture
7) Consider the following with respect to International and confiscation of proceeds and instruments of
Energy Agency (IEA) crimes.
1.It is an autonomous intergovernmental organization Is it a new concept?
established in the framework of the Organisation for
No.
Economic Co-operation and Development (OECD)
• India already has this treaty with 32 countries
2.India is member of International Energy Agency (IEA)
including Iran, Kuwait, Mauritius, Switzerland,
Select answer using code given below Mexico, Thailand, Vietnam and Kazakhstan.
A) 1 only B) 2 only • There is a mutual legal assistance treaty with
C) Both 1 and 2 D) Neither 1 nor 2 Australia, Bahrain, Bangladesh, Belarus, Bosnia and
Key : A Herzegovina, Bulgaria, Canada, Egypt, France,
Explanation : Mongolia, Myanmar, Russia, Singapore, South
Africa,South Korea, Spain, Sri Lanka, Turkey, Ukraine,
• The International Energy Agency is a Paris-based
United Arab Emirates, United Kingdom, United States
autonomous intergovernmental organization
of America and Uzbekistan.
established in the framework of the Organisation for
Economic Co-operation and Development in 1974 in • It will enhance effectiveness and provide broad legal
the wake of the 1973 oil crisis.It was established in framework for prevention, investigation and
the framework of the Organisation for Economic Co- prosecution of crimes as well as in tracing, restraint
operation and Development (OECD. The IEA advocates and confiscation of funds meant to finance terrorist
policies to facilitate enhance the reliabi lity, acts. It will help in countering threats posed by
affordability and sustainability of energy in its 30 organized crime and terrorism.
member countries and beyond. Objective :
• India, china and Russia are prominent non-member To know about the Mutual Legal Assistance Treaty and
countries of International Energy Agency (IEA) countries under this treaty with India.
Objective :
To know about the International Energy Agency 9) The Kurdish led Syrian Democratic Forces (SDF)
backed by the US has defeated the Daesh (ISIS) by
capturing the final shred of the territory of:
Indian Diaspora
A) Fallujah of Iraq B) Tikrit of Iraq
8) Which of the following statements are correct with
regards to Mutual Legal Assistance Treaty (MLAT)? C) Raqqa of Syria D) Baghouz of Syria
1. MLAT is an agreement between two nations with an Key : D
aim of collecting and exchanging information in an Explanation :
attempt to implement public laws or criminal laws. • The Kurdish-led Syrian Democratic Forces (SDF)
2. Restraint and confiscation of Funds meant to backed by the US has defeated the Daesh [ISIS] by
finance terrorist acts is also come under this treaty capturing the final shred of the territory of Baghouz
3. Kazakhstan is not included in this treaty with India. in Syria. The forces have declared the end of the self-
declared “caliphate” that once spanned a third of Iraq
Select the correct answer using the code given below
and Syria.
A) 1 and 2 only B) 2 and 3 only
• Even though the U.S.-backed forces have retaken nearly
C) 1 and 3 only D) 1, 2 and 3 all the small pockets in Syria that were still under
Key : A ISIS control, it doesn’t mean the end of ISIS. Reports
Explanation : suggest that many ISIS fighters have blended in with
• What is Mutual Legal Assistance Treaty (MLAT)? the local population and moreover the ISIS’ ideology

Page No.4
MARCH CURRENT AFFAIRS_2019
remains potent and continues to inspire attacks in Key observations:
Europe and Afghanistan. Even though it is unlikely • In India, poverty reduction among children, the
that there is a command structure directing terrorist poorest states, Scheduled Tribes, and Muslims was
attacks around the globe there are local groups which fastest.
identify themselves as ISIS. • Although Muslims and STs reduced poverty the most
Timeline of ISIS over the 10 years, these two groups still had the highest
• The chaos which followed the 2003 U.S.-led invasion rates of poverty.
of Iraq, an al Qaeda offshoot established there and • Bihar was the poorest state in 2015-16, with more
changed its name in 2006 to Islamic State in Iraq. than half its population in poverty. The four poorest
• As the Syrian Crisis began to unfold the group’s leader states —Bihar, Jharkhand, Uttar Pradesh, and Madhya
Abu Bakr al-Baghdadi sent operatives to Syria for Pradesh — were still home to 196 million MPI poor
setting up a Syrian subsidiary. Baghdadi follows in people, which was over half of all the MPI poor people
2013 and renamed the group as “The Islamic State in in India.
Iraq and the Levant”. • Jharkhand had the greatest improvement, followed
• The group captured important cities like Fallujah, by Arunachal Pradesh, Bihar, Chhattisgarh, and
Tikrit and Mosul in Iraq and Raqqa in Syria. Abu Bakr Nagaland.
al-Baghdadi renamed the group as Islamic State (IS) Objective :
and declares a caliphate at Mosul in 2014. To know about Global MPI 2018 Report and its key
• In 2014 United States started building a coalition findings regarding remarkable progress of India.
against ISIS and started air strikes to stop its
momentum. It also started to extend support to the
Kurdish militia to fight against the ISIS. Science & Technology
Space
• By 2016 the forces started recovering the territory
captured by ISIS and the ISIS started facing 11)Consider the following statements with respect to
catastrophic defeats. “Outer Space Treaty”:
The capture of Baghouz and declaration of elimination 1. The Outer Space Treaty bars state parties from
of caliphate has come as a final nail in the coffin of placing weapons of mass destruction and
ISIS. conventional weapons in orbit of Earth.
Objective : 2. India is party to Outer Space Treaty.
To know about the ISIS and SDF conflicts and invasion Which of the statement(s) given above is/are correct?
of Iraq and Syria by ISIS. A) 1 only B) 2 only
C) Both 1 and 2 D) Neither 1 nor 2
Indian Society Poverty and Key : B
Developmental Issues Explanation :
10)The recent Global Multidimensional Poverty Index • Outer Space Treaty is a legal entity and came in 1966
report 2018, covering 105 countries, dedicates a as a legal document towards the General Assembly.
chapter to India because of the remarkable progress. • The Treaty was opened for signature by the three
The report is prepared by depository Governments (the Russian Federation, the
A) United Nations Development Programme (UNDP) United Kingdom and the United States of America) in
B) Oxfam International January 1967, and it entered into force in October
1967.
C) United Nations Department of Economic and Social
Affairs • The Outer Space Treaty bars states party to the treaty
D) Concern Worldwide from placing weapons of mass destruction in orbit of
Earth, installing them on the Moon or any other
Key : A celestial body, or otherwise stationing them in outer
Explanation : space.
• Global MPI 2018 Report prepared by the United • It exclusively limits the use of the Moon and other
Nations Development Programme (UNDP) and the celestial bodies to peaceful purposes and expressly
Oxford Poverty and Human Development Initiative. prohibits their use for testing weapons of any kind,
• The Global MPI 2018, covering 105 countries, conducting military man oeuvres, or establishing
dedicates a chapter to India because of this military bases, installations, and fortifications
remarkable progress. However, India still had 364 (Article IV).
million poor in 2015-16, the largest for any country, • However, the Treaty does not prohibit the placement
although it is down from 635 million in 2005-06. of conventional weapons in orbit.
• India is party to Outer Space Treaty.

Page No.5
MARCH CURRENT AFFAIRS_2019
Objective : How does the virus spread or get transmitted?
To know about the details of “Outer Space Treaty”. • The disease is transmitted to humans through
12)“Mission Shakti” is sometimes seen in the news mosquito bites. Mosquitoes become infected when
recently, refers to which of the following? they feed on infected birds, which circulate the virus
A) Pokhran 1 test B) Pokhran 2 test in their blood for a few days. The virus gets transmitted
into the mosquito’s salivary glands, from where it is
C) Anti-satellite missile test injected into humans as well as animals through
D) Nuclear Triad Mission mosquito bites. The virus can multiply in the process
Key : C and possibly cause illness.
Explanation : • WNL may also be transmitted through contact with
• Indian scientists have successfully carried out the other infected animals, their blood or other tissues.
test of an anti-satellite (A-SAT) missile by bringing • Till date, no human-to-human transmission through
down one of its satellites in the low earth orbit 300 casual contact has been reported.
kilometers from the Earth’s surface. • A very small proportion of human infections have
• Mission Shakti ASAT has tested successfully from APJ occurred through organ transplant, blood
Abdul Kalam island (earlier, known as Wheeler Island) transfusions and breast milk while one case of
launch complex with a purpose of safeguarding transplacental (mother to child) transmission has
India’s own “space assets”. been reported.
• This brings India in the select League of Nations that Objective :
claim to have anti-satellite weapons. To know about the West Nile Virus and West Nile Fever.
• Only the United States, China and Russia have Internal Security
demonstrated this capability till now. Security challenges and their
• Israel is also said to possess this capability, though
management in border areas
it has not carried out a test so far.
14)Which of the following is the purpose of National
• Anti-satellite tests are extremely controversial and
Register of Citizens (NRC) in Assam?
considered to be contributing towards weaponisation
of the space, which is prohibited by the Outer Space A) It is to maintain the permanent record of
Treaty of 1967. households of Assam
Objective : B) It is to calculate the poverty level by having a record
of registered citizens of Assam
To know about the details of Mission Shakti
C) It is to separate illegal immigrants from legitimate
Basic Science
residents of Assam
13)Consider the following statements with regards to
D) It is to calculate the Unemployment rate in Assam
West Nile Virus (WNL):
with the demand of The All Assam Students’ Union
1. West Nile Virus is a viral infection which typically (AASU)
spread by mosquitoes and results in neurological
Key : C
disease as well as death in people.
Explanation :
2. WNL may also be transmitted through contact with
other infected animals, their blood or other tissues. Why in News?
3. Till date, no human – to – human transmission • The Supreme Court recently issued a notice to the
through casual contact has been reported centre and the Election Commission of India on a plea
seeking that the National Register of Citizens (NRC) be
Which of the following statements above is/are
updated to include Tripura.
correct using the code given below?
• The final draft of the NRC in Assam was released,
A) 1 and 2 only B) 2 and 3 only excluding four million residents of the state.
C) 1 and 3 only D) 1, 2 and 3
• This draft of the NRC is however not final and people
Key : A can still appeal against the non-inclusion of their
Explanation : names in the NRC.
What is West Nile Virus? • Several religious and linguistic minority groups are
• West Nile Virus is a viral infection which typically also opposing the NRC as discriminatory and
spread by mosquitoes and results in neurological undemocratic.
disease as well as death in people. Purpose: To separate “ illegal” immigrants from
• The Virus is the member of the flavivirus genus and “legitimate” residents of Assam.
belongs to the Japanese encephalitis antigenic Nodal Agency: Registrar General and Census
complex of the family Flaviviridae. It was first detected Commissioner India.
in a woman in the West Nile district of Uganda in What is National Register of Citizens (NRC)?
1937 and was later identified in birds (crows and
Columbiformes) in the Nile delta region in 1953.

Page No.6
MARCH CURRENT AFFAIRS_2019
• National Register of Citizens, 1951 is a register realistic simulation of Nuclear, Chemical & Biological
prepared after the conduct of the Census of 1951 in warfare to its personnel during their NBC damage
respect of each village, showing the houses or control training, which till now was limited to
holdings in a serial order and indicating against each theoretical training largely.
house or holding the number and names of persons • The steel structure of the simulator represents the
staying therein. relevant NBC compartment of the ship such as upper
• The NRC was published only once in 1951.NRC in decks, citadels, cleansing stations.
Assam Objective :
• The issue of its update assumed importance as Assam To know about the initiatives to tackle effectively with
witnessed large-scale illegal migration from erstwhile Nuclear, Chemical and Biological warfare.
East Pakistan and, after 1971, from present-day Space
Bangladesh.
16)Which of the following is an initiative launched by
• This led to the six-year-long Assam movement from Indian Space Research Organization (ISRO) which is
1979 to 1985, for deporting illegal migrants. aimed at imparting basic knowledge on Space
• The All Assam Students’ Union (AASU) led the Technology and Applications to the young children
movement that demanded the updating of the NRC and and enhance their interest in the emerging areas of
the deportation of all illegal migrants who had Space activities?
entered Assam after 1951. A) YUva VIgyani KAryakram(YUVIKA)
• The movement culminated in the signing of the Assam B) Uchchatar Avishkar Yojana (UAY)
Accord in 1985.
C) RESPOND (Research Sponsored programme)
• It set March 25, 1971, as the cut-off date for the
D) IMPRINT (IMPacting Research Innovation and
deportation of illegal migrants.
Technology)
• Since the cut-off date prescribed under articles 5 and
6 of the Constitution was July 19, 1949 - to give force Key : A
to the new date, an amendment was made to the Explanation :
Citizenship Act, 1955, and a new section was • The Indian Space Research Organisation (ISRO) has
introduced. launched a special programme for School Children
• It was made applicable only to Assam. called “Young Scientist Programme” “YUva VIgyani
KAryakram”from this year. The Program is primarily
• There had been intermittent demands from AASU and
aimed at imparting basic knowledge on Space
other organisations in Assam for updating the NRC,
Technology, Space Science and Space Applications to
an Assam based NGO filed a petition at the Supreme
the younger ones with the intent of arousing their
Court.
interest in the emerging areas of Space activities. ISRO
• In December 2014, a division bench of the apex court has chalked out this programme to “Catch them
ordered that the NRC be updated in a time-bound young”. The residential training programme will be of
manner. around two weeks duration during summer holidays
• The NRC of 1951 and the Electoral Roll of 1971 (up to and it is proposed to select 3 students each from each
midnight of 24 March 1971) are together called Legacy State/ Union Territory to participate in this programme
Data. Persons and their descendants whose names covering state, CBSE, and ICSE syllabus. The students
appeared in these documents are certified as Indian belonging to rural areas have been given special
citizens weightage under the selection criteria set by ISRO.
Objective : Objective :
To know about the National Register of Citizens (NRC). To know about the initiatives taken by ISRO in
2019-03-27 promoting the inclusion of students in space
Science & Technology programmes.
Defence Basic Science
15)Which among the following is a Nuclear, Biological, 17)The World Health Organization (WHO) has launched
and Chemical training facility recently commissioned a Global Influenza Strategy for 2019-2030, which aims
by Chief of Naval Staff ? to:
A) INS Shivaji B) SAHYOG 1. Prevent seasonal influenza
C) SURYA HOPE D) ABHEDYA 2. Control the virus’s spread from animals to humans
Key : D 3. Prepare for the next pandemic
Explanation : Select the correct answer using the code given below
• Admiral Sunil Lanba, Chief of the Naval Staff A) 1 and 2 only B) 1 only
commissioned a state of the art ‘Nuclear, Biological, C) 2 and 3 only D) 1, 2 and 3
Chemical Training Facility - Abhedya’, at INS Shivaji. Key : D
This facility will assist Indian Navy in providing a Explanation :

Page No.7
MARCH CURRENT AFFAIRS_2019
• The World Health Organization (WHO) has launched To know about the Global Influenza Strategy 2019-
a Global Influenza Strategy for 2019-2030, which aims 2030 by WHO.
to prevent seasonal influenza, control the virus’s Governance Government policies and
spread from animals to humans and prepare for the programs
next pandemic 18)Consider the following statements about
• The WHO warned that new pandemics are “inevitable”. TB(Tuberculosis):
Pandemic 1. It is a vector-borne disease which often affects the
• A pandemic is the worldwide spread of a new disease. lungs and is spread by Aedes mosquitoes.
• An influenza pandemic occurs when a new influenza 2. Mycobacterium Tuberculosis is the bacteria which
virus emerges and spreads around the world, and causes this disease.
most people do not have immunity. Viruses that have 3. Nikshay Poshan Yojana is a DBT(Direct Benefit
caused past pandemics typically originated from Transfer) Scheme aimed at providing nutritional
animal influenza viruses. support to Tuberculosis patients.
• Influenza epidemics are largely seasonal and affect Select the correct statements using the code given
around one billion people and kill hundreds of below:
thousands annually making it one of the world’s A) 1 and 2 only B) 2 and 3 only
greatest public health challenges. E.g.: C) 1 and 3 only D) 1, 2 and 3
• H5N1 (called avian influenza or “bird flu”) is a type of Key : B
influenza virus that causes a highly infectious, severe
respiratory disease in birds. Explanation :
• Nikshay Poshan Yojana (NPY): It is a scheme for
The new strategy is the most comprehensive and far-
nutritional support to Tuberculosis (TB) patients
reaching that WHO has ever developed for influenza.
launched by Ministry of Health and Family Welfare,
The strategy meets one of WHO’s mandates to improve Government of IndiaFinancial incentive of Rs.500/-
core capacities for public health, and increase global per month is provided to each notified TB patient for
preparedness. the duration for which the patient is on anti-TB
It outlines a path to protect populations every year treatment.
and helps prepare for a pandemic through • Tuberculosis (TB) is caused by bacteria
strengthening routine programmes. Features of the (Mycobacterium tuberculosis) that most often affect
new strategy are: the lungs. Tuberculosis is curable and preventable.
• Every country should strengthen routine health • TB is spread from person to person through the air.
programmes and develop tailor-made influenza When people with lung TB cough, sneeze or spit, they
programmes that strengthen disease surveillance, propel the TB germs into the air. A person needs to
response, prevention, control, and preparedness. inhale only a few of these germs to become infected.
• WHO recommends annual flu vaccines as the most • About one-quarter of the world’s population has latent
effective way to prevent the spread of the disease, TB, which means people have been infected by TB
especially for healthcare workers and people at higher bacteria but are not (yet) ill with the disease and
risk of influenza complications. cannot transmit the disease.
• Development of more effective and more accessible • Symptoms include cough, fever, night sweats, or weight
vaccines and antiviral treatments. loss. HIV patients are the worst to be affected by TB.
• Due to its mutating strains, vaccine formulas must be Objective : To know about the common diseases and the
regularly updated. government initiatives in providing support to the
The new influenza strategy builds on and benefits from patients.
successful WHO programmes like: 19)With reference to ‘National Health Mission’, consider
• Global Influenza Surveillance and Response System the following statements:
(GISRS): Launched in 1952, GISRS comprised of WHO 1. The main programmatic components include Health
Collaborating Centres and national influenza centres. System strengthening rural and urban areas-
It focuses on monitoring seasonal trends and Reproductive-Maternal-Neonatal-Child and
potentially pandemic viruses. This system serves as Adolescent Health (RMNCH+A), and Communicable and
the backbone of the global alert system for influenza. Non-Communicable Diseases.
• Pandemic Influenza Preparedness Framework is a 2. Rogi Kalyan Samiti, Kayakalp and Kilkari are the
unique access and benefit sharing system that initiatives under this.
supports the sharing of potentially pandemic viruses,
3.Its key goals include reduction of IMR, MMR and
provides access to life saving vaccines and treatments
TFR among others
in the event of a pandemic and supports the building
of pandemic preparedness capacities in countries Which of the statements given above is/are correct?
through partnership contributions from industry. A) 1 only B) 2 and 3 only
Objective : C) 3 only D) 1, 2 and 3

Page No.8
MARCH CURRENT AFFAIRS_2019
Key : D Key : B
Explanation : Explanation :
The National Health Mission: • In 2018, the government also launched Nikshay
• The National Health Mission (NHM) encompasses it Poshan Yojana. It is a direct benefit transfer (DBT)
two sub-Missions, the National Rural Health Mission scheme for nutritional support to TB patients. Under
(NRHM) and the newly launched National Urban Health the scheme, financial incentive of Rs.500 per month
Mission (NUHM). is provided to each notified TB patient for duration
• The main programmatic components include Health for which the patient is on anti-TB treatment
system strengthening in rural and urban areas- Objective :
Reproductive-Maternal-Neonatal-Child and To know about the Nikshay Poshan Yojana
Adolescent Health (RMNCH+A), and Communicable and Geography
Non-Communicable Diseases. Map & current affairs based
• The NHM envisages achievement of universal access questions
to equitable, affordable & quality health care services
that are accountable and responsive to people’s needs. 21)“A Geographical indication (GI) is a sign used on
products that have a specific geographical origin and
Goals: possess qualities or a reputation that are due to that
1. Reduce MMR to 1/1000 live births origin” . In this context, which of the following are
2. Reduce IMR to 25/1000 live births given GI tag?
3. Reduce TFR to 2.1 1.Madhubani Paintings of Bihar
4. Prevention and reduction of anaemia in women aged 2.Nakshi Kantha of West Bengal
15-49 years. 3.Araku valley Arabica Coffee of Andhra Pradesh
5. Prevent and reduce mortality & morbidity from 4.Kancheepuram Silk of Tamil Nadu.
communicable, non communicable; injuries and Select the correct answer using the code given below
emerging diseases
A) 1, 2 and 4 only B) 1, 3 and 4 only
6. Reduce household out-of-pocket expenditure on total
C) 1 and 4 only D) 1, 2, 3 and 4
health care expenditure
Key : D
7. Reduce annual incidence and mortality from
Tuberculosis by half Explanation :
8. Reduce prevalence of Leprosy to <1/10000 population • A geographical indication (GI) is a sign used on
and incidence to zero in all districts products that have a specific geographical origin and
possess qualities or a reputation that are due to that
9. Annual Malaria Incidence to be <1/1000
origin. In order to function as a GI, a sign must identify
10.Less than 1 per cent microfilaria prevalence in all a product as originating in a given place.
districts
Recent GI tags:
11.Kala-azar Elimination by 2015, <1 case per 10000
Coorg Arabica Coffee
population in all blocks
• Kodagu district of Karnataka is famous for Coffee
Rogi Kalyan Samiti/Hospital Management Society:
cultivation. The unique biotic and abiotic conditions
• This committee is a registered society whose members provide uniqueness to the Coffee grown in the region
act as trustees to manage the affairs of the hospital of Kodagu. The broader landscape provides it with a
and is responsible for upkeep of the facilities and unique aroma and flavour to the coffee when roasted.
ensure provision of better facilities to the patients in
the hospital. Wayanad Robusta Coffee
• Wayanad Robusta Coffee both as a pure crop and as
• Launch of Kayakalp: an initiative for award to public
mixed crop along with pepper in the Wayanad region
health facilities
of Kerala. Wayanad produces almost around 90% of
Objective : Kerala’s Coffee produce and forms the backbone of
To know about The National Health Mission the coffee economy of Kerala.
20)Consider the following statement with regards to Chikmagalur Arabica Coffee
Nikshay Poshan Yojana • Chikmagalur Arabica Coffee is grown in Chikmagalur
1.It provides free medical check up and medication to district, Karnataka. It is the region in India where the
TB patient coffee cultivation first began.
2.financial incentive of Rs.500 per month is provided Bababudangiri Arabica Coffee
to each notified TB patient for duration for which the • Bababudangiri Arabica Coffee is grown in the
patient is on anti-TB treatment Bababudangiri region of Karnataka. Bababudangiri
Select the correct option using code given below in Chikmanglur District of Karnataka is where the
A) 1 only B) 2 only coffee was first grown in India. The coffee grown here
C) Both 1 and 2 D) Neither 1 nor 2 is known for its unique flavour and aroma. Selectively

Page No.9
MARCH CURRENT AFFAIRS_2019
hand-picked and processes by natural fermentation, Do you know?
the cup exhibits full body, acidity, mild flavour and • Darjeeling tea became the first product to get this tag
striking aroma with a note of chocolate which makes in 2005.
it unique. • The Pashmina from Kashmir, Nagpur mangoes,
Araku Valley Arabica Coffee Madhubani paintings of Bihar, are some of the
• Araku Valley Arabica Coffee has a pleasant acidity examples.
with a citrus note of grapefruit and a mild jaggery- Objective : To know about the GI tag in India.
like sweetness with light to medium strength. It is International Relations
grown in regions of Visakhapatnam district in Andhra
India-US-China
Pradesh and Koraput district in Odisha.
23)2+2 dialogue sometimes seen in news, with regards
Objective :
to it,consider the following statements:
To know about the GI tag and GI tags in India.
1. It is aimed enhancing strategic coordination
between India and U.S.
22)‘Sirsi Supari’ received the GI tag in Uttara Kannada is 2.It puts strategic, defence and security relationship
the name of: between two countries at forefront and centre stage.
A) Arabica coffee B) Paintings 3.It replaced earlier India – US Strategic and
C) Arecanut D) Saree Commercial Dialogue
Key : C Which of the following statements above is/are
Explanation : correct by using the code given below?
• For the first time in the arecanut sector, ‘Sirsi Supari’ A) 1 and 2 only B) 2 and 3 only
grown in Uttara Kannada has received the GI tag. C) 1 and 3 only D) 1, 2 and 3
• It is cultivated in Yellapura, Siddapura and Sirsi taluks Key : D
of Karnataka. Explanation :
• Its GI number is 464. • The first edition of 2+2 dialogue between India-US
• The arecanut grown in these taluks have unique held in New Delhi. In the inaugural meeting, Minister
features like a round and flattened coin shape, of External Affairs (MEA) Sushma Swaraj and Defence
particular texture, size, cross-sectional views, taste, Minister Nirmala Sitharaman hosted US Secretary of
etc. State Michael R. Pompeo and US Secretary of Defence
• These features are not seen in arecanut grown in any James Mattis.
other regions. 2+2 dialogue
Important value additions: • 2+2 dialogue is second highest level of engagement
Geographical Indication between both countries after summit-level engagement
• According to the World Intellectual Property Rights, between the Prime Minister and the US President. It
“Geographical Indication is the sign used on the was agreed between both during visit of Prime
products that have specific geographical origin and Minister Narendra Modi to US in June, 2017. It is
posses’ reputation and some qualities that are due to similar to India-Japan 2+2 dialogue format between
the origin.” foreign and defence secretaries and ministers of the
• In India Geographical Indication tag is governed by two countries.
the Geographical Indication of Goods (Registry and • 2+2 dialogue is aimed enhancing strategic
Protection) Act of 1999. coordination between both countries and maintaining
About GI Act, 1999: peace and stability in Indo-Pacific region. It puts
strategic, defence and security relationship between
• GIs indicate goods as originating in a specific two countries at forefront and centre stage. It replaced
geographical region, the characteristics, qualities or earlier India-US Strategic and Commercial Dialogue.
reputation thereof essentially attributable to such
region. • 2+2 dialogue insulates India-US strategic relationship
from feuds over trade issues and deep divide on
• Complying with the World Trade Organisation-Trade- economic integration policies as trade and
Related Aspects of Intellectual Property Rights (WTO- commercial issues that collided with strategic
TRIPS) obligations, India enacted the Geographical relationship between both countries’ discussed in
Indications of Goods (Registration & Protection) Act, Strategic and Commercial Dialogue earlier.
1999 (GI Act) and has set up a registry in Chennai to
register such names. Objective :
• Covering agricultural goods, manufactured and To know about the 2+2 Dialogue between India and
natural goods, textiles, handicrafts and foodstuffs, U.S.
the GI Registry’s website lists popular GIs like Basmati
rice, Darjeeling tea and Pashmina shawls etc.

Page No.10
MARCH CURRENT AFFAIRS_2019
International Institutions, agencies and their The acts
structure and mandate • The Acts which deal with Alternative Dispute
24)Consider the following with respect to Global Energy Resolution are Arbitration and Conciliation Act,
Transition Index(GETI) 1996 and the Legal Services Authorities Act, 1987.
1. The index is released by the World Economic Forum • Section 89 of the Civil Procedure Code, 1908 makes
(WEF). it possible for Arbitration proceedings to take place
2.Index considers both the current state of the in accordance with the Acts stated above.
country’s energy system and their structural Advantage of Alternate Dispute Resolution:
readiness to adapt to future energy needs. • It is less expensive.
Select the correct answer using code given below: • It is less time consuming.
A) 1 only B) 2 only • It is free from technicalities as in the case of
C) Both 1 and 2 D) Neither 1 nor 2 conducting cases in law Courts.
Key : C • The parties are free to discuss their difference of
Explanation : opinion without any fear of disclosure of this fact
• The index is released by the World Economic Forum before any law Courts.
(WEF).It ranks 115 countries on how well they are • The last but not the least is the fact that parties are
able to balance energy security and access with having the feeling that there is no losing or winning
environmental sustainability and affordability. The feeling among the parties by at the same time they
WEF index considers both the current state of the are having the feeling that their grievance is
countries energy system and their structural redressed and the relationship between the parties
readiness to adapt to future energy needs. is restored.
• India has been ranked 76th out of 115 countries on Objective :
2019 Global Energy Transition Index(GETI). Sweden To know about the Alternative Dispute Resolution
has been ranked first and is followed by Switzerland (ADR)
and Norway. 2019-03-26
Objective : International Relations International
To know about the Global Energy Transition Institutions, Agencies and
Index(GETI) their Structure and mandate
Polity Judiciary 26)Consider the following statements with respect to
25)Which of the following is/are correct with regards “United Nations World Food Programme”:
to Alternative Dispute Resolution (ADR)? 1.The World Food Programme is governed by an
1.ADR in India was founded on the Constitutional Executive Board which consists of representatives
basis of Articles 14 and 21. from member states.
2.It also tries to achieve the Directive Principle of 2.The WFP operations are funded by voluntary
State Policy relating to equal justice and free legal donations from world governments, corporations
aid. and private donors.
Select the correct answer using the code given below 3. The major objective of the programme is to
A) 1 only B) 2 only achieve Zero Hunger by 2030.
C) Both 1 and 2 D) Neither 1 nor 2 Which of the statements given above are correct?
Key : C A) 1 and 2 only B) 2 and 3 only
Explanation : C) 1 and 3 only D) 1, 2 and 3
ADR and Constitution Key : D
• ADR first started as a quest to find solutions to the Explanation :
perplexing problem of the ever increasing burden • The World Food Programme (WFP) is the food
on the courts. It was an attempt made by the assistance branch of the United Nations and the
legislators and judiciary alike to achieve the world’s largest humanitarian organization
“Constitutional goal” of achieving Complete Justice. addressing hunger and promoting food security.
• Alternative Dispute Resolution in India was founded • The WFP strives to eradicate hunger and
on the Constitutional basis of Articles 14 and 21 malnutrition, with the ultimate goal in mind of
which deal with Equality before Law and Right to eliminating the need for food aid itself.
life and personal liberty respectively. • It is a member of the United Nations Development
• ADR also tries to achieve the Directive Principle of Group and part of its Executive Committee.
State Policy relating to Equal justice and Free Legal • Born in 1961, WFP pursues a vision of the world in
Aid as laid down under Article 39-A of the which every man, woman and child has access at
Constitution. all times to the food needed for an active and

Page No.11
MARCH CURRENT AFFAIRS_2019
healthy life. The WFP is governed by an Executive Governance Government Policies and
Board which consists of representatives from member Programs
states. 28)Consider the following about NIVARAN-Grievance
• The WFP operations are funded by voluntary Portal:
donations from world governments, corporations and 1.It is the platform for resolution of service related
private donors. WFP food aid is also directed to fight grievances of railway commuters
micro-nutrient deficiencies, reduce child mortality, 2.‘NIVARAN-Grievance Portal’ is the first IT application
improve maternal health, and combat disease, to be launched on the RailCloud
including HIV and AIDS.
Select the correct statements based on the codes
The objectives of the World Food Programme are: below:
1. Save lives and protect livelihoods in emergencies. A) 2Only B) 2 Only
2. Support food security and nutrition and (re)build C) Both 1 and 2 D) Neither 1 nor 2
livelihoods in fragile settings and following
Key : B
emergencies.
Explanation :
3. Reduce risk and enable people, communities and
countries to meet their own food and nutrition needs. • Following new railway initiatives were recently
launched:
4. Reduce under-nutrition and break the inter-
generational cycle of hunger. RailCloud:
5. Zero Hunger in 2030. • Rail Cloud works on Cloud Computing system. Most
Important works are done through Cloud Computing.
Objective :
• It is developed by rail PSU RailTel.
To know about the details of “United Nations World
Food Programme”. NIVARAN-Grievance Portal:
• ‘NIVARAN-Grievance Portal’ is the first IT application to
be launched on the RailCloud.
Science & Technology Basic Science
(Hence statement 2 is correct).
27)Consider the following statements with respect to
“Muon elementary subatomic particles”: • It is the platform for resolution of service related
grievances of serving and former railway employees.
1. The muon is an elementary particle similar to the
(Hence statement 1 is incorrect)
electron, with an electric charge of -1 e.
Objective :
2. The Gamma Ray Astronomy PeVEnergieS phase-3 or
GRAPES-3 muon telescope is located in Ladakh. To know about the initiatives taken in railways
Which of the statement(s) given above is/are correct? 29)Consider the following about sovereign gold bond
scheme:
A) 1 only B) 2 only
1.Investments in such bonds by banks will be counted
C) Both 1 and 2 D) Neither 1 nor 2
in calculation in SLR
Key : A
2.Bonds cannot be used as collateral for loans.
Explanation :
3.It is issued by the RBI on behalf of the Government
• The muon is an elementary particle similar to the of India.
electron, with an electric charge of ?1 e and a spin of
Which of the above statements is/are correct:
1/2, but with a much greater mass. It is classified as a
lepton. A) 1 and 2 only B) 1 and 3 only
• A muon is relatively unstable, with a lifetime of only C) 2 and 3 only D) 1,2 and 3
2.2 microseconds, since muons are charged they lose Key : B
energy by displacing electrons from atoms (ionization) Explanation :
before decaying. • The Government announced a few changes in its
• In nature, high energy cosmic rays (usually protons) Sovereign Gold Bond (SGB) Scheme recently.
hit nuclei in the upper atmosphere and produce • The primary change was the increase in the limit to 4
“showers” of particles, including muons and muon kg (from 0.5kg) for individuals, HUF and 20 kg for
neutrinos. Trusts.
• The Gamma Ray Astronomy PeVEnergieS phase-3 or • This was probably done to encourage high net-worth
GRAPES-3 muon telescope is located at Ooty, Tamil individuals, rich farmers as well as trusts to invest in
Nadu. these bonds.
• It is designed to study cosmic rays with an array of • The Government also introduced flexibility in the
air shower detectors and a large area muon detector. scheme to design and introduce variants to cater to a
Objective : cross-section of investors.
To know about the details of muon subatomic • SGBs are government securities denominated in grams
particles. of gold.

Page No.12
MARCH CURRENT AFFAIRS_2019
• It was first launched under the gold monetization Monetary Policy
scheme of 2015. 31)Which of the following also acts as a mechanism for
• It is issued by the RBI on behalf of the Government of government borrowing;
India. (Hence statement 3 is correct) 1.CRR 2.Repo Rate
• The Sovereign Gold Bonds will be available both in 3.Reverse Repo Rate 4.SLR
demat and paper form. Select the correct answer using the codes below
• The tenor of the bond is for a minimum of 8 years with A) 1,2 and 3 only B) 2 and 3 only
option to exit in 5th, 6th and 7th years.
C) 4 only D) 2 , 3 and 4 only
• Bonds can be used as collateral for loans. (Hence
Key : D
statement 2 is incorrect)
• Tradable through National Stock Exchange (NSE) and Explanation :
Bombay Stock Exchange (BSE). • CRR or cash reserve ratio is the amount of money a
• Fixed Interest rate of 2.5% per annum payable once in commercial bank must keep with RBI at all times.
6 months. • REPO rate or policy rate is the rate at which
commercial banks borrow money from the RBI
• Capital Gains Tax exempted on redemption.
• Investments in such bonds by banks will be counted • Reverse Repo rate is the rate at which commercial
in calculation in SLR (Statutory Liquidity Ratio). (Hence banks lend to the RBI
statement 1 is correct) • SLR or statutory liquidity ratio is the amount of money
a bank must keep with itself in the form of liquid assets
Objective : To know about the Sovereign Gold Bond (SGB)
for day to day operations. Purchases of government
Scheme
treasury bills form a huge part of SLR. Thus, SLR is a
Economy mechanism to indirectly finance government
Basics of Economy borrowings.
30)FinTech Conclave-2019 twas inaugurated by Governor, Objective :
Reserve Bank of India (RBI) in New Delhi on 25th To test the conceptual clarity in economy
March, 2019. The Conclave was organized by
32)Infrastructure Leasing & Financial Services (IL&FS) is
A) Reserve Bank of India a RBI Registered Core investment Company formed by
B) Ministry of Finance 1. The Central Bank of India (CBI)
C) Ministry of Science & Technology 2.Housing Development Finance Corporation (HDFC)
D) NITI Aayog 3. Unit Trust of India (UTI)
Key : D Select the correct answer using the codes given below
Explanation : A) 1 and 2 only B) 2 and 3 only
• FinTech Conclave-2019 to be inaugurated by Governor, C) 1 and 3 only D) 1, 2 and 3
Reserve Bank of India (RBI) in New Delhi on 25th
March, 2019. Key : D
• The conclave, whose objective is to shape India’s Explanation :
continued ascendancy in Financial Technology • IL&FS was formed in 1987 as an “RBI registered Core
(FinTech), build the narrative for future strategy and Investment Company” by three financial institutions,
policy efforts, and to deliberate steps for namely the Central Bank of India, Housing
comprehensive financial inclusion is organised by Development Finance Corporation (HDFC) and Unit
the NITI Aayog. Trust of India (UTI), to provide finance and loans for
major infrastructure projects. Gradually, as the
• The Reserve Bank of India has categorized these
organization needed better financing, it additionally
financial technology (or fintech) companies as peer-
opened itself to two large international players,
to-peer (P2P) lenders.
namely Mitsubishi (through Orix corporation Japan)
• Fintech (Financial technology) describes an emerging and the Abu Dhabi Investment authority. Subsequently,
financial services sector. Life Insurance Corporation India, Orix and ADIA
• Originally, the term referred to computer technology became its largest shareholders, a pattern that
applied to the back office of banks or trading firms, continues to this day.
and now the term has expanded to include any Objective :
technological innovation in the financial sector
To know about the IL&FS.
including innovations in financial literacy and
education, retail banking, investment and even crypto- Polity
currencies like bitcoin. Constitutional bodies (Origin,
Objective : structure and functions)
To know about the details of Fintech sector. 33)Consider the following statements with regards to
Comptroller and Auditor General:

Page No.13
MARCH CURRENT AFFAIRS_2019
1.Audit of all transactions of the Union and of the 2. At a General Election to Lok Sabha or Legislative
state with respect to contingency fund and public Assembly, the party polls 6% of votes in four States
accounts and in addition it wins 4 Lok Sabha seats.
2.Audit of receipts and expenditure of bodies or 3. A party gets recognition as a State Party in four or
authorities which are mostly financed by the Union more states
or the states. Objective :
3.RBI is audited by CAG To know about the eligibilities or conditions to get
4.Conduct audit on request of any bodies or status of National Party.
authorities. Legislature
Select the correct answer using the codes given below 35)Select the correct from the following with regards to
A) 1, 2 and 3 only B) 1, 2 and 4 only Public Accounts Committee (PAC)
C) 2, 3 and 4 only D) 1, 2, 3 and 4 1.PAC consists of 22 members; out of it 15 are
Key : B appointed from Rajya Sabha and 7 from Lok Sabha.
Explanation : 2. Chairman of the PAC should be the senior member
The Audit functions of the CAG are: of the main Opposition Party
1. Audit of all transactions of the union and of the state 3.The key function of the PAC is to examine annual
with respect to contingency fund and public accounts audit reports of CAG after they are tabled in
parliament by president
2. Audit of all trading, manufacturing, profit and loss
accounts and balance sheets and other subsidiary Select the correct answer using the codes given below
funds maintained by any department of the Union or A) 1 and 2 only B) 2 and 3 only
of a state, and present an report on the expenditure, C) 1 and 3 only D) 1, 2 and 3
transactions of the accounts audited. Key : B
3. Audit receipts and expenditure of bodies or authorities Explanation :
which are mostly financed by the Union or the States. About PAC:
4. Conduct audit on request of any bodies or authorities. • PAC is one of the most important finance committees
Can CAG audit RBI? in the Parliament along with the Committee on
• Why RBI is not audited by CAG? The RBI Act 1934 has Estimates and the Committee on Public Undertakings.
an enabling provision (Section 51) to have the • It is constituted every year and consists of 22 members
accounts audited by special auditors of the – 15 from Lok Sabha and 7 from Rajya Sabha.
Government. However, this has never been done • The members from Lok Sabha are elected from
probably because RBI has been conducting its amongst its members according to the principle of
business meticulously and never given room for any proportional representation by means of single
allegations of malpractices. transferable vote. Members from Rajya Sabha elected
Objective : by that House in like manner are associated with the
To know about the Comptroller and Auditor General Committee.
34)Which of the following condition (s) is/are required • Speaker usually appoints the Chairman of the PAC. It
for recognition of registered party as National Party? should be noted that a minister cannot be elected to
1. The party wins 2% of seats in the Lok Sabha from at as member of Committee.
least 3 different States. • It should be noted that since 1967, Chairman of PAC
2.At a General Election to Lok Sabha or Legislative is senior member of the main Opposition party.
Assembly, the party polls 8% of votes in four States • The key function of PAC is to examine annual audit
and in addition it wins 4 Lok Sabha seats. reports of Comptroller and Auditor General (CAG) after
3.A party gets recognition as a State Party in 3 or they are tabled in Parliament by President.
more states. Objective :
Select the correct answer using the codes given below To know about the Public Accounts Committee, its
A) 1 only B) 2 only structure and functions.
C) 3 only D) 1, 2 and 3 2019-03-25
Key : A Economy External Sector
Explanation : 36)Consider the following statements with regards to
• A registered party is recognized as a National Party Generalized System of Preferences (GSP):
only if it fulfils any one of the following three 1. It is to promote economic growth in the developing
conditions: world by providing preferential duty-free entry.
1. The party wins 2% of seats in the Lok Sabha (as of 2.GSP promotes sustainable development in
2014, 11 seats) from at least 3 different States. beneficiary countries by helping these countries to

Page No.14
MARCH CURRENT AFFAIRS_2019
increase and diversify their trade with the United Objective :
States To know about the CAATSA
Which of the above is/are correct using the code given Infrastructure
below? 38)Which of the following is/are correct with regards to
A) 1 only B) 2 only “Strategic Petroleum Reserves Programme (SPR)”?
C) Both 1 and 2 D) Neither 1 nor 2 1.Central Government under Phase I of SPR programme
Key : C has built 3 underground crude oil storage facilities
Explanation : (SPR) at V isakhapatnam, Mangalore and Padur
Generalised System of Preferences (GSP): (Kerala).
• The Generalized System of Preferences (GSP) is a U.S. 2.They are maintained by Indian Strategic Petroleum
trade program designed to promote economic growth Reserves Ltd (ISPRL) under Ministry of Petroleum &
in the developing world by providing preferential Natural Gas.
duty-free entry for up to 4,800 products from 129 Select the correct answer using the code given below
designated beneficiary countries and territories. A) 1 only B) 2 only
What is the objective of GSP? C) Both 1 and 2 D) Neither 1 nor 2
• The objective of GSP was to give development support Key : C
to poor countries by promoting exports from them Explanation :
into the developed countries. GSP promotes • Central Government under phase I of SPR programme
sustainable development in beneficiary countries by has built three underground crude oil storage
helping these countries to increase and diversify their facilities (Strategic Petroleum Reserves) with total
trade with the United States. GSP provide capacity of 5.33 MMT capacity at Vishakhapatnam
opportunities for many of the world’s poorest (Andhra Pradesh, storage capacity of 1.33 million
countries to use trade to grow their economies and tonnes), Mangalore (Karnataka, 1.5 million tonnes)
climb out of poverty. and Padur (Kerala, 2.5 million tonnes) to provide
Objective : energy security of 10 days of consumption in response
To know about the Generalized System of Preferences to external supply disruptions. They are maintained
(GSP). by Indian Strategic Petroleum Reserves Ltd (ISPRL), a
special purpose vehicle (SPV), which is wholly owned
37)CAATSA Stands for “Countering America’s Adversaries subsidiary of Oil Industry Development Board (OIDB)
Through Sanctions Act”, with regards to it consider under Ministry of Petroleum & Natural Gas.
the following statements: Objective :
1.It mandates US administration to impose sanctions To know about the Strategic Petroleum Reserves
on any country carrying out significant defence and Programme under Ministry of Petroleum & Natural
energy trade with sanctioned entities in North Korea, Gas.
Iran and Russia
2.India is now under CAATSA. Environment_Ecology National Environment Laws &
Which of the above is/are correct using the code given Policies
below? 39)Recently, the government introduced a new category
A) 1 only B) 2 only of forests - “Production forest”, by amending the
C) Both 1 and 2 D) Neither 1 nor 2 Indian Forest Act, 1927. Consider the following
Key : C statements with respect to “Production forest”:
Explanation : 1.These will be forests with specific objectives for
production of timber, pulp, non-timber forest produce,
• CAATSA stands for “Countering America’s Adversaries medicinal plants.
Through Sanctions Act”
2.The objective of this introduction is to increase
• This punitive act was signed by President Donald forest production in the country.
Trump in August 2017.
Which of the statement(s) given above is/are correct?
• It mandates US administration to impose sanctions
on any country carrying out significant defence and A) 1 only B) 2 only
energy trade with sanctioned entities in North Korea, C) Both 1 and 2 D) Neither 1 nor 2
Iran and Russia. Key : C
• This is an act by the Congress, thus the President of Explanation :
the United States of America doesn’t have too much of • In an attempt to address contemporary challenges to
authority over it. India’s forests, the government is amending the Indian
• Why in news? India is buying S-400 system from Forest Act, 1927.
Russia that might invite sanctions under CAATSA

Page No.15
MARCH CURRENT AFFAIRS_2019
• The amendment also introduces a new category of each other. The key difference is that a positron is
forests — production forest. These will be forests with antimatter and a proton is not.
specific objectives for production of timber, pulp, Objective :
pulpwood, firewood, non-timber forest produce, To know about the details of Anti matter and its
medicinal plants or any forest species to increase properties.
production in the country for a specified period.
Polity
Objective :
To understand the recent amendments to the Indian
Constitutional bodies (Origin, Structure and Functions)
Forest Act, 1927.
41)Consider the following statements with respect to
“Systematic Voters’ Education and Electoral
Science & Technology Participation program (SVEEP)”:
Current affairs on Science and 1. SVEEP’s primary goal is to build a truly participative
technology democracy in India by encouraging all eligible citizens
40)Physicists from the Large Hadron Collider beauty to vote and make an informed decision during the
(LHCb) Collaboration at CERN have observed, for the elections.
first time, the matter-antimatter asymmetry. With 2. It is the flagship program of the NGO “Association
respect to “Anti Matter ”, consider the following for Democratic Reforms”.
statement: Which of the statement(s) given above is/are correct?
1. Antimatter is just regular matter with a few A) 1 only B) 2 only
properties flipped, such as the electric charge.
C) Both 1 and 2 D) Neither 1 nor 2
2.Antimatter is matter that does not interact
Key : A
electromagnetically.
Explanation :
3.When antimatter comes in contact with its regular
matter counterpart, they mutually destroy each other. • Systematic Voters’ Education and Electoral
Participation program, better known as SVEEP, is the
Which of the statement(s) given above is/are correct?
flagship program of the Election Commission of India
A) 1 and 2 only B) 2 and 3 only for voter education, spreading voter awareness and
C) 1 and 3 only D) 1, 2 and 3 promoting voter literacy in India. Since 2009, we have
Key : C been working towards preparing India’s electors and
Explanation : equipping them with basic knowledge related to the
• Antimatter is just regular matter with a few properties electoral process.
flipped, such as the electric charge. For example, the • SVEEP’s primary goal is to build a truly participative
antimatter version of an electron is a positron. They democracy in India by encouraging all eligible citizens
both have the same mass, but have opposite electric to vote and make an informed decision during the
charge. Antimatter has regular mass and accelerates elections. The programme is based on multiple general
in response to forces just like regular matter. Also, as well as targeted interventions which are designed
antimatter is gravitationally attracted to other forms according to the socio-economic, cultural and
of matter just like regular matter. demographic profile of the state as well as the history
• For every particle that exists, there is an antimatter of electoral participation in previous rounds of
counterpart (some particles such as photons are their elections and learning thereof.
own anti-particles). What makes antimatter unique Objective :
is that when antimatter comes in contact with its To know about the details of Systematic Voters’
regular matter counterpart, they mutually destroy Education and Electoral Participation program.
each other and all of their mass is converted to energy. 2019-03-23
This matter-antimatter mutual annihilation has been International Relations International Institutions,
observed many times and is a well-established
principle. In fact, medical PET scans routinely use Agencies and their Structure
annihilation events in order to form images of and mandate
patients. Antimatter is therefore only distinct from 42)Consider the following statement with regards to
regular matter in that it annihilates when meeting Bloomberg Healthiest Country Index 2019
regular matter. 1.Bloomberg Healthiest Country Index has ranked
• For instance, a proton and a positron are somewhat Switzerland as the world’s healthiest country
similar. They both have regular mass. They both have 2.India ranked lowest among SAARC countries.
a positive electric charge of the same strength. They Select the correct answer using code given below
both have a quantum spin of one half. But when a
A) 1 only B) 2 only
proton meets an electron, it forms a stable hydrogen
atom. When a positron meets an electron, they destroy C) Both 1 and 2 D) Neither 1 nor 2
Key : D

Page No.16
MARCH CURRENT AFFAIRS_2019
Explanation :
• The 2019 edition of the Bloomberg Healthiest Country Governance Government Policies and
Index has ranked Spain as the world’s healthiest Programs
country, followed by Italy, Iceland and Switzerland, 45)Consider the following about Nikshay Poshan Yojana
Sweden, Australia, Singapore and Norway. These are
also the countries with universal healthcare where 1.This scheme aims to support Leprosy patient
over 70% of healthcare spending is done by the 2.Under this Patient will receive Rs500 on monthly
government. Cuba (30) is the only country not basis
classified as “high income” by the World Bank to be Select the correct statements using code given below
ranked that high India ranked 120 compared to 119 A) 1 only B) 2 only
in the 2017 ranking. Pakistan (124), Myanmar (129) C) Both 1 and 2 D) Neither 1 nor 2
and Afghanistan (153) are the only countries in the
Key : B
Asian region with a lower ranking than India.
Explanation :
Objective :
• Nikshay Poshan Yojana is direct benefit transfer (DBT)
To know about the reports and indices which are in
scheme for nutritional support to Tuberculosis (TB)
news
patients. Under this Financial incentive of Rs.500/-
43)Consider the following statement with regards to per month in cash or Kind is given for each notified
Bloomberg Healthiest Country Index 2019 TB patient for duration for which the patient is on
1.Bloomberg Healthiest Country Index has ranked anti-TB treatment
Switzerland as the world’s healthiest country Objective :
2.India ranked lowest among SAARC countries. To know about various government schemes like
Select the correct answer using code given below Nikshay Poshan Yojana
A) 1 only B) 2 only 46)Project Saamrath sometimes seen in the news
C) Both 1 and 2 D) Neither 1 nor 2 recently;Consider the following statements
Key : D 1.It aimed at skilling, upskilling and reskilling 10 lakh
Explanation : personnel involved in the rubber sector, by 2020.
• The 2019 edition of the Bloomberg Healthiest Country 2.The training is being provided under the Recognition
Index has ranked Spain as the world’s healthiest of Prior Learning Scheme of Pradhan Mantri Kaushal
country, followed by Italy, Iceland and Switzerland, Vikas Yojana (PMKVY).
Sweden, Australia, Singapore and Norway. These are Which of the statement(s) given above is/are correct?
also the countries with universal healthcare where A) 1 only B) 2 only
over 70% of healthcare spending is done by the C) Both 1 and 2 D) Neither 1 nor 2
government. Cuba (30) is the only country not
Key : C
classified as “high income” by the World Bank to be
ranked that high India ranked 120 compared to 119 Explanation :
in the 2017 ranking. Pakistan (124), Myanmar (129) • The Rubber Skill Development Council (RSDC) has
and Afghanistan (153) are the only countries in the launched Project Saamrath, which is aimed at skilling,
Asian region with a lower ranking than India. upskilling and reskilling 10 lakh personnel involved
Objective : in the rubber sector, by 2020.
To know about the reports and indices which are in • Over 10 lakh personnel are involved in NR (natural
news rubber) plantation work and the requirement of
skilling is huge for increasing production and
44) Ease of Living Index recently in news; is released by
productivity.
A) World bank
• The Council has joined hands with the Rubber Board
B) Ministry of corporate affairs in upskilling the growers.
C) Ministry of housing and urban affairs • The training is being provided under the Recognition
D) Niti Aayog of Prior Learning Scheme of Pradhan Mantri Kaushal
Key : C Vikas Yojana across natural rubber plantations, tyre
Explanation : services and rubber manufacturing.
• “Ease of Living Index” rankings launched by the Objective : To know about the Saamrath sometimes seen
Ministry of Housing and Urban Affairs (MoHUA). Ease in the news recently
of Living Index is a transformative initiative of the Polity
Ministry to help the cities assess their liveability vis- Miscellaneous
à-vis national and global benchmarks. 47)Consider the following statement with regards to
Objective : Article 35-A
To know about the recently released reports and
indioces

Page No.17
MARCH CURRENT AFFAIRS_2019
1.Article 35-A was incorporated in the Constitution in liquidation, individual insolvency resolution and
1954 through constitutional amendment individual bankruptcy under the Code.
2.It gives the Jammu and Kashmir Legislature a • It is a key pillar of the ecosystem responsible for
complete freedom to decide, who are all ‘Permanent implementation of the Insolvency and Bankruptcy
Residents’ of the State. Code that consolidates and amends the laws relating
Select incorrect answer using code given below to reorganization and insolvency resolution of
A) 1 only B) 2 only corporate persons, partnership firms and individuals
in a time bound manner.
C) Both 1 and 2 D) Neither 1 nor 2
The Governing Board of IBBI consists of Chairperson
Key : A
and seven members.
Explanation :
Objective :
• Article 35-A was incorporated in the Constitution in
To know about the IBBI (Insolvency and Bankruptcy
1954 through Presidential order.It gives the Jammu
Board of India).
and Kashmir Legislature a complete freedom to
decide, who are all ‘Permanent Residents’ of the State. 49)Who among the following is the adjudicating
This confers on them (a) special rights and privileges authority for insolvency resolution in case of
in public sector jobs (b) acquisition of property in individuals as per the Insolvency and Bankruptcy
the State (c)scholarships as well as public aid and Code?
welfare and (d)If a native woman marries a man not A) Insolvency and Bankruptcy Board of India
holding a permanent resident certificate of Jammu & B) National Company Law Tribunal
Kashmir, then she would be restricted from her C) Debt Recovery Tribunal D) IRDAI
property right. Key : C
Objective : Explanation :
To know about the Article 35-A of indian constitution • Debt Recovery Tribunal is the adjudicating authority
2019-03-22 for individuals National Company Law Tribunal is the
Economy Monetary Policy adjudicating authority for companies.
48)Consider the following statements related to Objective :
Insolvency and Bankruptcy Board of India (IBBI): to know about the adjudicating authorities for
1.It is a statutory organisation insolvency resolution.
2.It has regulatory oversight over the insolvency Service sector
professionals, insolvency professional agencies and 50)Consider the following statements with regards to
information utilities SWIFT (Society for Worldwide Interbank Financial
3.The Union Finance Minister is the Chairperson of Telecommunication):
the IBBI 1.The SWIFT is to develop a secure electronic
Which of the above statement (s) is/are correct? messaging service and common standards to facilitate
A) 1 and 2 only B) 2 and 3 only cross-border payments.
C) 1 and 3 only D) 1, 2 and 3 2.SWIFT does facilitate funds transfer
Key : A 3.India’s largest private sector bank called Axis Bank
Explanation : became first Indian bank to go live on SWIFT’s.
• The Insolvency and Bankruptcy Board of India (IBBI) Which of the above is/are correct using the code given
and the Reserve Bank of India (RBI) signed a below?
Memorandum of Understanding (MoU). Under the A) 1 only B) 2 only
MoU, IBBI and RBI will assist and co-operate with C) Both 1 and 2 D) Neither 1 nor 2
each other for the effective implementation of the Key : A
Insolvency and Bankruptcy Code. Explanation :
About Insolvency and Bankruptcy Board of India (IBBI) • The SWIFT is a global member-owned cooperative that
• The IBBI was established under the Insolvency and is headquartered in Brussels, Belgium. It was founded
Bankruptcy Code, which was approved by the in 1973 by a group of 239 banks from 15 countries
Parliament in the form legislation in May 2016. which formed a co-operative utility to develop a
• It is a unique regulator as it regulates a profession as secure electronic messaging service and common
well as transactions. It has regulatory oversight over standards to facilitate cross-border payments. It
the insolvency professionals, insolvency professional carries an average of approximately 26 million
agencies and information utilities. financial messages each day. In order to use its
• It writes and enforces rules for transactions, namely, messaging services, customers need to connect to the
corporate insolvency resolution, corporate SWIFT environment.

Page No.18
MARCH CURRENT AFFAIRS_2019
Functions: World Geography
• SWIFT does not facilitate funds transfer: rather, it 52)Cyclone ‘Idai’ recently seen in news; It caused
sends payment orders, which must be settled by devastation in
correspondent accounts that the institutions have A) Sri Lanka B) Indonasia
with each other. C) Mozamique D) Uruguay
• The SWIFT is a secure financial message carrier — in Key : C
other words, it transports messages from one bank to
Explanation :
its intended bank recipient.
• Cyclone ‘IDAI’ recently caused devastation in
• Its core role is to provide a secure transmission
Mozambique. The ships of First Training Squadron of
channel so that Bank A knows that its message to
Indian Navy (Sujata, Sarathi and Shardul) operating
Bank B goes to Bank B and no one else. Bank B, in turn,
in the Southern Indian Ocean were diverted to Port
knows that Bank A, and no one other than Bank A,
Beira, Mozambique based on request received from
sent, read or altered the message en route. Banks, of
the Government of Mozambique to provide
course, need to have checks in place before actually
Humanitarian Assistance and Disaster Relief (HADR)
sending messages.
to the local population post the devastation caused
• India’s largest private sector lender ICICI Bank became by cyclone ‘IDAI’ which struck the coast of
first Indian bank to go live on SWIFT’s (Society for Mozambique.
World Interbank Financial Telecommunication
Objective : To know about the cyclones and disasters
System) Global Payment Innovation (GPI), an
which are in news
improved cross-border payments service.
Objective : Science & Technology
Current affairs on Science
To know about the Society for Worldwide Interbank
Financial Telecommunication. and technology
53)Which of the following is/are correct with regards to
Geography Climatology The Central Drugs Standard Control Organisation
(CDSCO)?
51)Which of the following factors affect the Ocean
1.CDSCO is responsible for approval of New Drugs,
Currents?
conduct of Clinical trials, laying down the standards
1. Heating by solar energy;2.Wind for drugs, control over the quality of imported drugs
3.Gravity 4.Coriolis force. in the country.
Select the correct answer using the code given below 2.Drugs Technical Advisory Board (DTAB) is a part of
A) 1, 3 and 4 only B) 1, 2, and 3 only the CDSCO under the ministry of Health and Family
C) 1, 2 and 4 only D) 1, 2, 3 and 4 Welfare.
Key : D Select the correct answer using the code given below
Explanation : A) 1 only B) 2 only
The primary forces that influence the currents are: C) Both 1 and 2 D) Neither 1 nor 2
(i) Heating by solar energy (ii) Wind Key : C
(iii) Gravity (iv) Coriolis force. Explanation :
• Heating by solar energy causes the water to expand. • The Central Drugs Standard Control Organisation
That is why, near the equator the ocean water is about (CDSCO) under Directorate General of Health Services,
8 cm higher in level than in the middle latitudes. This Ministry of Health & Family Welfare, Government of
causes a very slight gradient and water tends to flow India is the National Regulatory Authority (NRA) of
down the slope. Wind blowing on the surface of the India.
ocean pushes the water to move. Friction between the • Functions: Under the Drugs and Cosmetics Act, CDSCO
wind and the water surface affects the movement of is responsible for approval of New Drugs, Conduct of
the water body in its course. Gravity tends to pull the Clinical Trials, laying down the standards for Drugs,
water down the pile and create gradient variation. control over the quality of imported Drugs in the
The Coriolis force intervenes and causes the water to country and coordination of the activities of State
move to the right in the northern hemisphere and to Drug Control Organizations by providing expert advice
the left in the southern hemisphere. These large with a view to bring about the uniformity in the
accumulations of water and the flow around them enforcement of the Drugs and Cosmetics Act.
are called Gyres. These produce large circular currents • CDSCO along with state regulators, is jointly
in all the ocean basins. responsible for grant of licenses of certain specialized
Objective : categories of critical Drugs such as blood and blood
To know the mechanism and distribution of ocean products, I. V. Fluids, Vaccine and Sera.
currents and oceanic circulation.

Page No.19
MARCH CURRENT AFFAIRS_2019
Drugs Technical Advisory Board (DTAB) Developmental delays
DTAB is the highest statutory decision-making body on • People who have Down syndrome usually have
technical matters related to drugs in India. It was cognitive development profiles that suggest mild to
constituted under the provisions of the Drugs and moderate intellectual disability. However, cognitive
Cosmetics Act, 1940. DTAB is part of the Central Drugs development and intellectual ability are highly
Standard Control Organization (CDSCO) under the variable.
Ministry of Health and Family Welfare. • Children with Down syndrome often reach
Objective : developmental milestones a little later than their
To know about the Central Drugs Standard Control peers.
Organisation (CDSCO). • Most children with Down syndrome have mild to
moderate cognitive impairment. Language is delayed,
Biotechnology and both short and long-term memory is affected.
54)Consider the following statements about “Down • Fine motor skills may also be delayed. They can take
Syndrome”. time to develop after the child acquires gross motor
skills.
1.It is a viral disease that affects the neurons in the
brain. Objective : To know about the Down Syndrome
2.In children affected by this syndrome language is
delayed and there is a high risk of Alzheimer’s in aged Governance Government Policies and
people. Programs
Select the correct statement(s) using the code given 55)Consider the following statements regarding
below: Corporate Social Responsibility(CSR)
A) 1 only B) 2 only 1.A company having turnover of 10,000 crore or more
C) Both 1 and 2 D) Neither 1 nor 2 need to spend on corporate social responsibility
Key : B (CSR).
Explanation : 2.Any Foreign entity operating from India need not to
spend on CSR activities.
• Down syndrome is a chromosomal condition that
occurs when an error in cell division results in an Which of the statements given above is/are correct?
extra chromosome 21. Down syndrome can affect a A) 1 only B) 2 only
person’s cognitive ability and physical growth, cause C) Both 1 and 2 D) Neither 1 nor 2
mild to moderate developmental issues, and present Key : D
a higher risk of some health problems. Down Explanation :
syndrome occurs in around 1 in every 700
• Corporate social responsibility (CSR) was initiated
pregnancies. It is determined by many factors, but
through the Companies Act, 2013.The act mandated
research suggests that there is a higher chance if the
companies and government organisations with
mother is older than 35 years of age.
(a)turnover of Rs1,000 crore or more(b)net worth
• All cells in the body contain genes that are grouped exceeding Rs 500crore or (c)having more than Rs 5
along chromosomes in the cell nucleus. There are crore in net profits,to set aside 2 per cent of their
normally 46 chromosomes in each cell — 23 inherited average net profits for CSR activities.
from the mother and 23 from the father.
• Further as per the CSR Rules, the provisions of CSR
• When some or all of a person’s cells have an extra are not only applicable to Indian companies, but also
full or partial copy of chromosome 21, Down applicable to branch and project offices of a foreign
syndrome occurs. company in India.
• Individuals with Down syndrome commonly have Objective :
distinct physical features, unique health issues, and
variability in cognitive development. To know about the corporate social responsibility
Physical features
Some physical characteristics of Down syndrome 56)Voluntary Code of Ethics sometimes seen in the news
include: recently was associated with which of the following?
A) Coastal regulation Zones
eyes that slant upward, have oblique fissures, have
epicanthic skin folds on the inner corner, and have B) Treatment of Prisoners of wars
white spots on the iris low muscle tone small stature C) Social Media Regulations
and a short neck a flat nasal bridge single, deep D) Gene editing guidelines
creases across the center of the palms a protruding Key : C
tongue a large space between large and second toe a
Explanation :
single flexion furrow of the fifth finger
• The Social Media Platforms and Internet and Mobile
Association of India (IAMAI) have recently presented

Page No.20
MARCH CURRENT AFFAIRS_2019
a “Voluntary Code of Ethics for the General Election 59)Consider the following statements regarding
2019”. Monopolistic Restrictive and Trade Practices (MRTP),
• The ‘Code of Ethics” has been developed to ensure 1969:
free, fair & ethical usage of Social Media Platforms 1.According to this act already established industries
including Facebook, WhatsApp, Twitter, Google, who possess net assets value is 25cr, they cannot
ShareChat and TikTok etc. to maintain the integrity of acquire & establish new industries.
the electoral process for the General Elections 2019. 2.The limit for net asset value is increased to 50cr
• The Code voluntarily agreed upon by the Participants and 100cr in 1980 and 1985 respectively.
comes into operation with immediate effect. 3.After 1991 it is removed & Liberalisation,
Objective : Privatization and Globalization came to exist to
To know about the Voluntary Code of Ethics which is encourage Industrial revolution.
in news recently 4.MRTP act is abolished, now it is replaced by
Competition Commission of India (CCI) 2002.
International Relations International Institutions, Select the correct statement (s) form the above using
Agencies and their Structure the code given below
and Mandate A) 1, 2 and 3 only B) 2, 3 and 4 only
57)Which of the following is not a member country of C) 1, 3 and 4 only D) 1, 2, 3 and 4
UNESCO? Key : D
1. USA 2. Israel Explanation :
3. Palestine 4. Afghanistan 5. India • On the basis of recommendation of Dutt Committee,
Which of the options given above is/are correct? MRTP Act was enacted in 1969 to ensure that
concentration of economic power in hands of few rich.
A) 1,3 and 4 only B) 3 only
The act was there to prohibit monopolistic and
C) 1 and 2 only D) 2,3 and 4 only restrictive trade practices. It extended to all of India
Key : C except Jammu & Kashmir.
Explanation : The aims and objectives of this act were:
• All are members except USA and Israel. USA and Israel 1. To ensure that the operation of the economic system
have recently withdrawn their memberships from does not result in the concentration of economic
UNESCO. power in hands of few rich.
Objective : 2.To provide for the control of monopolies, and
To know the recent developments in UNESCO 3.To prohibit monopolistic and restrictive trade
2019-03-21 practices.
Economy Industry Objective : To know about the MRTP act, 1969.
58)Which of the following can be the reason for failure Monetary Policy
of IPR 1956? 60)‘Gadgil Report’, one of the several area-specific reports
1.Most of the power retained by the Central were publish during 1940s which was on:
Government A) Agricultural Development B) Rural Credit
2. Most of the rich private persons got license C) Agricultural Prices D) Cooperatives
3.Public Sector Units are not produced goods Key : B
according to demands of the society Explanation :
Select the correct answer using the code given below • The idea for the need of a planned development of
A) 1 and 2 only B) 2 and 3 only India became more and more popular by the decade
C) 1 and 3 only D) 1, 2 and 3 of the 1940s. It was under this popular pressure that
Key : D the Government of India started taking some planned
actions in this direction. In the 1940s, we see several
Explanation :
area-specific reports being published:
Why this IPR, 1956 failed?
• Gadgil Report on Rural Credit
(a)Most of the power retained by the central
• Kheragat Report on Agricultural Development
Government
• Krishnamachari Report on Agricultural Prices
(b) Most of the rich private persons got license
• Saraiya Report on Cooperatives
(c) PSU’s are not produced goods according to
demands of the society. • A series of Reports on Irrigation (ground water, canal,
etc.)
Objective :
All these reports, though prepared with great care and
To know about the Industrial Policy resolutions in
due scholarship, the Government had hardly any zeal
India.
to implement plans on their findings. But independent

Page No.21
MARCH CURRENT AFFAIRS_2019
India was greatly benefited when the planning started Growth and Development
covering all these areas of concerns. 62)Which of the following is/are correct in comparing
Objective : India and china with regards to some parameters?
To know about the various reports published during 1.India defeated the china in terms of fastest growing
1940s country of the world. Now India is the world leader in
India’s Economic Growth terms of the growth rate of GDP
61)Consider the following statements regarding the 2.India is more debt ridden country as compared to
important features included by the constitutional china.
provisions which pertain to the objectives of planning Select the correct answer using the code given below
in the country: A) 1 only B) 2 only
1.‘Economic and social planning’ is a concurrent C) Both 1 and 2 D) Neither 1 nor 2
subject Key : B
2.The constitution includes provision for promoting Explanation :
co – operation on a voluntary basis between the union
and the states. GDP Growth Rate (2018)
A. India: 7.3% (2018)
3.The constitution also sets out in broad outline the
pattern of the welfare state envisaged and the B. China: 6.6% (2018)
fundamental principles on which it should rest. • India defeated the china in terms of fastest growing
Which of the above is/are correct using the code given country of the world. Now India is not the world leader
below? in terms of the growth rate GDP. As Nauru, Ethiopia,
Turkemenistan, Qatar are the world leaders currently
A) 1 only B) 1 and 2 only
in terms of the growth rate to GDP.
C) 2 and 3 only D) 1, 2 and 3
Debt to GDP Ratio:
Key : D
A. India: 50.1 % of GDP (2017)
Explanation :
B. China: 18.6 % of GDP (2017)
• There are three important features included by the
It means India is more debt ridden country as compare
Constitutional provisions which pertain to the
to china.
objectives of planning in the country:
Objective :
• ‘Economic and social planning’ is a concurrent
subject. Also, while framing the ‘Union’, ‘State’ and To know about the different parameters comparison
‘Concurrent’ list, allocating subjects and other between china and India.
provisions, the Constitution vests power in the Union
to ensure co-ordinated development in essential fields Fiscal Policy
of activity while preserving the initiative and authority 63)Consider the following statements about GST Council
of the states in the spheres allotted to them.
1.The Prime Minister will be the Chairperson
• The Constitution includes provisions for promoting
2.The Union Minister of State in charge of Revenue or
co-operation on a voluntary basis between the Union
Finance will be a member
and the states and among states and groups of states
in investigation of matters of common interest, in 3.The Minister in charge of finance or taxation or any
legislative procedures and in administration, thus other Minister nominated by each State government,
avoiding the rigidities inherent in federal will be its members.
constitutions (Articles 249, 252, 257, 258, 258-A, and Select the incorrect statements using code given below
312). In other words, the objective is co-operative A) 1 only B) 2 and 3 only
federalism. C) 1 and 2 only D) 1,2 and 3
• The Constitution also sets out in broad outline the Key : A
pattern of the welfare state envisaged and the
Explanation :
fundamental principles on which it should rest.
• According to the article, GST Council will be a joint
• These are the major cornerstones of planning and its
forum for the Centre and the States. It consists of the
objectives enshrined in the Constitution that will
following members:
breed enough Union–State tussle in coming decades
and make it compulsive for the Government to resort • The Union Finance Minister will be the Chairperson.
to ‘reforms with a human face’ rhetoric. We can see • Member – the Union Minister of State in charge of
the methodology of planning taking a U-turn in the Revenue or Finance.
era of the economic reforms since early 1990s. • The Minister in charge of finance or taxation or any
Objective : To know about the important features included other Minister nominated by each State government,
by the constitutional provisions which pertain to the as members.
objectives of planning in India? Objective :
To know about the structure of GST council.

Page No.22
MARCH CURRENT AFFAIRS_2019
Environment_Ecology Biodiversity countries have been a part of the Shanghai 5 since
64)Recently Astrobatrachus kurichiyana frog is 1996.
dicovered; consider the following statements • The cooperation was renamed to Shanghai
1.It has been discovered in west ghat region Cooperation Organization after Uzbekistan joined the
2.The species has been named Astrobatrachus organization in 2001. India and Pakistan joined SCO
kurichiyana for its constellation-like markings and as full members in June 2017 in Astana, Kazakhstan.
the indigenous people of Kurichiyarmala, the hill Objective :
range where it was found. To know about the details of recently conducted
Select correct statements using code given below military exercises.
A) 1 only B) 2 only 66)Consider the following statements with respect to
C) Both 1 and 2 D) Neither 1 nor 2 “Operation Sunrise”:
Key : C 1. Operation Sunrise was a joint operation carried
out by both Indian and Myanmar armies.
Explanation :
2. The target of the Operation Sunrise was the Arakan
• An international research team, with members from Army.
IISc, have discovered a new species of frog in the
Western Ghats. Which of the statement(s) given above is/are correct?
• The species has been named Astrobatrachus A) 1 only B) 2 only
kurichiyana for its constellation-like markings and C) Both 1 and 2 D) Neither 1 nor 2
the indigenous people of Kurichiyarmala, the hill Key : C
range where it was found. It is dark brown with a Explanation :
bright orange underbelly, and speckled with pale blue • ndia has carried out Operation Sunrise to wipe out
dots. the insurgent camps planning to hit the Indian workers
Objective : at the Kaladan Transit project. The Operation was
To know about the recent discoveries in western ghats reportedly carried between mid-February to early
International Relations March.
India-South Asia(India and its • Operation Sunrise was a joint operation carried out
neighbhors) by both Indian and Myanmar armies. During the
operation, at least 10 camps of an insurgent group in
65)Consider the following statements with respect to
neighbouring Myanmar were attacked.
recently conducted exercises:
• The Indian Army did not cross into the Myanmar side
1. Mitra Shakti is an annual joint naval exercise
but there was a massive mobilisation along the
between India and Sri Lanka.
border, on the Mizoram side. Indian side supported
2. Sary- Arka is an anti-terrorism exercise between Operation Sunrise by providing surveillance
the shanghai Cooperation Organization’s (SCO) equipment and deploying additional troops on the
member states. Indian side.
Which of the statement(s) given above is/are correct? • The target of the Operation Sunrise was the Arakan
A) 1 only B) 2 only Army, an insurgent group trained by the Kachin
C) Both 1 and 2 D) Neither 1 nor 2 Independence Army (KIA) which is reportedly trained
Key : B by China.
Explanation : • India had carried out a similar operation in 2017
which had inflicted heavy casualties on the insurgent
• Exercise MITRA SHAKTI is conducted annually as part
National Socialist Council of Nagaland-Khaplang
of military diplomacy and interaction between armies
(NSCN-K) group during the retaliatory fire along the
of India & Sri Lanka. The aim of the exercise is to
India-Myanmar border.
build and promote close relations between armies of
both the countries and to enhance ability of joint Objective :
exercise commander to take military contingents of To know about the details of Operation Sunrise
both nations under command.
• Sary- Arka is an antiterrorism exercise between the Science & Technology Concept, Scope and
shanghai Cooperation Organization’s (SCO) member Nature of Intellectual Property Rights
states. 67)Recently, Google has been granted a copyright license
• The Shanghai Cooperation Organization, also known by the Indian Performing Right Society Limited (IPRS).
as the Shanghai Pact, is a Eurasian political, In this context, consider the following statements:
economic, and military organization.
1.IPRS is the country’s only copyright society,
• It was founded in 2001 in Shanghai by the leaders of authorized to carry out the copyright business for
China, Kazakhstan, Kyrgyzstan, Russia, Tajikistan, and musical works and literary works (lyrics), associated
Uzbekistan. Apart from Uzbekistan, the other five with its members.

Page No.23
MARCH CURRENT AFFAIRS_2019
2.Members of IPRS comprises of the author (lyricists), • Any sketch, plan, model, article, note, document or
music composers and music publishers. information which relates to or is used in a prohibited
Which of the statement(s) given above is/are correct? place.
A) 1 only B) 2 only • Not just sharing the above-mentioned information,
C) Both 1 and 2 D) Neither 1 nor 2 also retaining such information is a punishable under
act.
Key : C
• In 1967, the act was amended and the scope of section
Explanation :
5 and section 8 were widened
• Google has been granted a copyright license by the
Problems of OSA:
Indian Performing Right Society Limited (IPRS).
• There is no clear definition of “secret” documents or
• The copyright license will enable Google to use IPRS
information, government can declare any document
members’ work and material in India on YouTube and
as official secrets.
other services.
• The act is in contravention of the Right to Information
• Google recently launched YouTube Music, a paid music (RTI) Act that came into effect in 2005.
streaming service, in the country.
• OSA’s background is the colonial climate of mistrust
• IPRS is the country’s only copyright society, authorized of people and the primacy of public officials in dealing
to carry out the copyright business for musical works with the citizens, which do not suit to the present
and literary works (lyrics), associated with its democratic India.
members.
• It provides a mechanism to promote culture of secrecy
• Members of IPRS comprises of the author (lyricists),
in the governance and makes disclosure an exception,
music composers and music publishers.
going against the transparency requirement of
• The IPRS was founded on 23rd August 1969. democratic governance.
• The IPRS is a representative body of Owners of Music, • Section 5, which deals with potential breaches of
viz. The Composers, Lyricists (or Authors) and the national security, is often misinterpreted and misused
Publishers of Music and is also the sole Authorized to frame journalists
Body to issue Licences for usage of Musical Works &
Objective :
Literary Music within India by any person.
To know about the Official Secrets Act, 1923.
Objective :
To know about the details of Indian Performing Right
Society Limited (IPRS). 69)The Committee to look around India’s Official Secrets
Act and suggest changes to the colonial law to
complete the transition from the secrecy regime of
Polity Fundamental Rights the past century to a more modern and democratic
68)Consider the following statements with regards to transparency regime constituted by Union Home
Official Secrets Act (OSA): Ministry is:
1.OSA was enacted immediately after Independence A) LC Goyal Committee
of India. B) Abid Hussain Committee
2.It deals with cases of espionage, sedition and any C) Athreya Committee
other threat to the unity and sovereignty of the country.
D) Dave Committee
3.Government can declare any document as official Key : A
secrets
Explanation :
Select the correct answer using the code given below
LC Goyal Committee:
A) 1 and 2 only B) 2 and 3 only
• In February 2015, the Union Home Ministry had
C) 1 and 3 only D) 1, 2 and 3
constituted a committee (headed by LC Goyal to look
Key : B around India’s Official Secrets Act and suggest
Explanation : changes to the colonial law to complete the transition
• OSA was enacted during British rule in 1923. It deals from the secrecy regime of the past century to a more
with cases of espionage, sedition and any other threat modern and democratic transparency regime.
to the unity and sovereignty of the country. • Abid Hussain Committee: On Small Scale Industries
• The law is applicable to government servants and • Athreya Committee: On Restructuring of IDBI
citizens. • Dave Committee: Pension Scheme for Unorganised
• The punishment under the Act involves fine or Sector
imprisonment from three to 14 years or both. Objective :
• As per section 5 of OSA information can be in the form To know about the various committees constituted by
of government of India.

Page No.24
MARCH CURRENT AFFAIRS_2019
Miscellaneous 71)Which of the following is/are correct with regards to
70)Lokpal is appointed based on the recommendation of Lokpal act, 2013?
the five-member panel comprising the: 1.The Lokpal act provides for setting up of an anti-
1.Prime Minister corruption ombudsman called Lokpal at the centre
2.The Lok Sabha Speaker and Lokayukta at the state – level.
3.The Leader of the Opposition 2.The ambit of Lokpal would cover all categories of
public servants, including the Prime Minister except
4.Minister of Law and Justice
the armed forces
5.The chief justice of India
3.50% of members of Lokpal shall be from SC/ST/OBCs,
Select the correct answer using the code given below minorities and women
A) 1, 2, 3 and 5 only B) 1, 2 and 4 only 4.Justice Pinaki Chandra Ghose would be the first
C) 1, 3, 4 and 5 only D) 1, 2, 3, 4 and 5 Lokpal of India.
Key : A Select the correct answer using the code given below
Explanation : A) 1, 2 and 3 only B) 2, 3 and 4 only
• The name of former Supreme Court Judge Justice Pinaki C) 1, 3 and 4 only D) 1, 2, 3 and 4
Chandra Ghose has been cleared by the Lokpal Key : D
Selection Committee headed by the Prime Minister.
The notification for the appointment is expected to be
released soon. Justice Pinaki Chandra Ghose would Explanation :
be the first Lokpal of India. • The name of former Supreme Court Judge Justice
Pinaki Chandra Ghose has been cleared by the Lokpal
Selection Committee headed by the Prime Minister.
Salient features of the Lokpal Act 2013
The notification for the appointment is expected to be
• The Lokpal Act provides for setting up of an anti- released soon. Justice Pinaki Chandra Ghose would
corruption ombudsman called Lokpal at the Centre be the first Lokpal of India.
and Lokayukta at the State-level.
Salient features of the Lokpal Act 2013
• The Lokpal would consist of a chairperson and a
• The Lokpal Act provides for setting up of an anti-
maximum of eight members of which 50 per cent shall
corruption ombudsman called Lokpal at the Centre
be judicial members. 50 per cent of members of Lokpal
and Lokayukta at the State-level.
shall be from SC/ST/OBCs, minorities and women.
• The Lokpal would consist of a chairperson and a
• The ambit of Lokpal would cover all categories of
maximum of eight members of which 50 per cent
public servants, including the Prime Minister with
shall be judicial members. 50 per cent of members of
the exception of armed forces.
Lokpal shall be from SC/ST/OBCs, minorities and
• The Lokpal Act mandated the states to institute women.
Lokayukta within one year of the commencement of
• The ambit of Lokpal would cover all categories of
the Act.
public servants, including the Prime Minister with
• Lokpal will also have powers of superintendence and the exception of armed forces.
direction over any investigation agency including CBI
• The Lokpal Act mandated the states to institute
for cases referred to them by the Lokpal.
Lokayukta within one year of the commencement of
• Lokpal is appointed based on the recommendation of the Act.
the five-member panel comprising the Prime Minister, • Lokpal will also have powers of superintendence and
the Lok Sabha Speaker, the Leader of the Opposition, direction over any investigation agency including CBI
the Chief Justice of India and an eminent jurist for cases referred to them by the Lokpal.
nominated by the President.
• Lokpal is appointed based on the recommendation of
• President would appoint the eminent jurist based on the five-member panel comprising the Prime
the recommendations of the other members of the Minister, the Lok Sabha Speaker, the Leader of the
panel. Opposition, the Chief Justice of India and an eminent
• The government has proposed an amendment to the jurist nominated by the President.
Lokpal Act 2013 to enable the leader of the largest • President would appoint the eminent jurist based on
Opposition party to be the member of the selection the recommendations of the other members of the
panel in the absence of the leader of Opposition. To panel.
designate the leader as Leader of Opposition the
largest opposition party must have a strength of at • The government has proposed an amendment to the
least 10% of the total members in the House and none Lokpal Act 2013 to enable the leader of the largest
of the parties managed to cross this mark in 2014 Opposition party to be the member of the selection
general elections. panel in the absence of the leader of Opposition. To
designate the leader as Leader of Opposition the
Objective :
largest opposition party must have a strength of at
To know about the Lokpal appointment in India.

Page No.25
MARCH CURRENT AFFAIRS_2019
least 10% of the total members in the House and none • Budgetary support for funding cost of enabling
of the parties managed to cross this mark in 2014 infrastructure i.e. roads and bridges on case to case
general elections. basis as per actual, limited to Rs. 1.5 crore per MW
Objective : for upto 200 MW projects and Rs. 1.0 crore per MW
To know about the Lokpal system and Lokpal act, for above 200 MW projects.
2013 in India. Objective :
Geography To know about the existing renewable energy
Human and Economic resources in India.
Geography
72)Which of the following is/are renewable energy 2019-03-20
resources? International Relations
1. Wind Energy India & Africa
2. Small Hydro power projects 73)Consider the following statements with respect to
3. Large Hydro Power Projects AFINDEX-19:
4. Solar 1.It is a field training exercise between India and
African nations.
5. Geo thermal Energy
2. It aims to train the participating contingents in
Select the correct answer using the code given below
Humanitarian Mine Assistance and Peace Keeping
A) 1, 2, 4 and 5 only B) 1, 4 and 5 only Operations under the United Nations Charter.
C) 1, 2 and 4 only D) 1, 2, 3, 4 and 5 Which of the statement(s) given above is/are correct?
Key : D A) 1 only B) 2 only
Explanation : C) Both 1 and 2 D) Neither 1 nor 2
What is renewable energy? Key : C
• Energy generated from renewable sources (which are Explanation :
constantly replenished) is known as renewable energy.
• Africa-India Joint Field Training Exercise (AFINDEX-
Example: solar power, wind energy, tidal energy,
19) between the Indian Army and 16 African nations
geothermal energy etc.
will be conducted in Pune from March 18 to 27.
• The Union Cabinet, chaired by the Prime Minister
• 10 personnel, each from the participating nations
Narendra Modi, has approved Measures to promote
from the African continent and personnel of Maratha
Hydro Power Sector. These include Declaring Large
Light Infantry of the Indian Army will participate in
Hydropower Projects (HPO) as part of non-solar
the joint exercise.
Renewable Purchase Obligation (RPO)
Details:
• The joint exercise would be held at Pune’s Foreign
• Large Hydropower Projects to be declared as
Training Node at Aundh Military Station and College
Renewable Energy source (as per existing practice,
of Military Engineering in Kirkee.
only hydropower projects less than 25MW are
categorized as Renewable Energy). • AFINDEX-19 aims to train the participating contingents
in Humanitarian Mine Assistance (HMA) and Peace
• HPO as a separate entity within non-solar Renewable Keeping Operations (PKO) under the United Nations
Purchase Obligation to cover LHPs commissioned Charter through practical and comprehensive
after notification of these measures (SHPs are already discussions and tactical exercises
covered under Non-Solar Renewable Purchase
Obligation). The trajectory of annual HPO targets will • Contingents from Egypt, Ghana, Nigeria, Senegal,
be notified by Ministry of Power based on the Sudan, South Africa, Tanzania, Namibia, Mozambique,
projected capacity addition plans in hydropower Uganda, Niger & Zambia are part of the joint exercise
sector. Necessary amendments will be introduced in together with officers from Rwanda, Democratic
the Tariff Policy and Tariff Regulations to Republic of Congo and Madagascar as Observers.
operationalize HPO. • The joint exercise will also focus on achieving
• Tariff rationalization measures including providing interoperability, learning each other’s methodologies
flexibility to the developers to determine tariff by back and tactics through synchronised operational level
loading of tariff after increasing project life to 40 planning and tactical level training.
years, increasing debt repayment period to 18 years • The joint exercise will also cover the dynamics of
and introducing escalating tariff of 2%; raising a Mission headquarters, nuances of
• Budgetary support for funding flood moderation establishing military observer team sites in disturbed
component of hydropower projects on case to case Objective :
basis; and To know about the details of AFINDEX 19.

Page No.26
MARCH CURRENT AFFAIRS_2019
Science & Technology Objective : To know about the mechanism behind the
Current affairs on Science formation of sunspot cycle.
and technology Computer, IT and Tele
74)Recently the scientists have found that solar tsunamis Communications
are responsible for triggering an 11-year cycle of 75)‘Aurora Supercomputer’ is sometimes seen in the news.
sunspots. In this context, consider the following Consider the following statements with respect to
statements: Aurora Supercomputer:
1.Sunspots are temporary phenomena on the Sun’s 1. Aurora supercomputer will be the world’s fastest
photosphere that appear as spots darker than the super computer and has to be built by Peoples’
surrounding areas. Republic of China.
2.They are regions of reduced surface temperature 2. It will be the first machine to reach a milestone
caused by concentrations of magnetic field flux that called “exascale” performance, i.e. one billion billion
inhibit convection. calculations per second.
3.It is believed that the “solar dynamo” a naturally Which of the statement(s) given above is/are correct?
occurring generator which produces electric and A) 1 only B) 2 only
magnetic fields in the sun is linked to the production C) Both 1 and 2 D) Neither 1 nor 2
of sunspots.
Key : B
Explanation :
Which of the statement(s) given above is/are correct?
• The United States has decided to build the fastest
A) 1 only B) 1 and 2 only supercomputer. The supercomputer is named as
C) 2 and 3 only D) 1, 2 and 3 Aurora.
Key : D • It is scheduled to be delivered to the Argonne National
Explanation : Laboratory near Chicago in 2021.
• A group of solar physicists suggests that a “solar • It will be the first machine to reach a milestone called
tsunami” is at work that triggers the new sunspot cycle, “exascale” performance, i.e. one billion billion
after the old one ends. calculations per second.
• Solar tsunami is the waves of the magnetic field and • This speed is around seven times the speed of the
hot, ionized gas moving across the Sun at about 400 most powerful system built to date, or 1,000 times
km per second. Tsunamis are generated after a release faster than the first “petascale” systems built in 2008.
of matter into space called a coronal mass ejection • The new machines will let researchers create
(CME). significantly more accurate simulations of
• Sunspots are areas that appear dark on the surface of phenomena such as drug responses, climate changes,
the Sun. They appear dark because they are cooler the inner workings of combustion engines and solar
than other parts of the Sun’s surface. panels.
• It is believed that the “solar dynamo” a naturally • An International Business Machines (IBM)
occurring generator which produces electric and Corporation system called Summit(OLCF-4), built for
magnetic fields in the sun is linked to the production the Oak Ridge National Laboratory in the USA is the
of sunspots. fastest supercomputer in the world with a clocked
How Solar Tsunami leads to Sunspot? speed of 143.5 petaflops.
• The extreme temperature and pressure conditions that Objective :
prevail some 20,000 km below the sun’s surface cause To know about the details of Aurora Supercomputer.
its material to form a plasma consisting primarily of Space
hydrogen and helium in a highly ionized state. 76)“ISRO and French counterpart CNES has sealed an
• The plasma is confined with huge magnetic fields agreement to set up a joint maritime surveillance
inside the sun. Holding these fields in their place system by putting up a low-earth orbiting satellites”
requires that there is extra mass (plasma mass) in this context consider the following statements:
pushing at the bands from higher latitudes. 1.The system will be augmented with the launch of
• Thus, a magnetic dam is formed which is storing a big Ocean sat -3-Argos mission along with a joint infrared
mass of plasma. At the end of a solar cycle, this Earth –observation satellite
magnetic dam can break, releasing huge amounts of 2.These will identify and track movement of ships
plasma cascading like a tsunami towards the poles. globally, In particular ships moving in the Indian
• These tsunami waves travel at high speeds of about Ocean region where France has its Reunion Islands
1,000 km per hour carrying excess plasma to the mid- Which of the above statement (s) is/are correct using
latitudes. There they give rise to magnetic flux the code given below
eruptions. These are seen as the bright patches that
A) 1 only B) 2 only
signal the start of the next cycle of sunspots.
C) Both 1 and 2 D) Neither 1 nor 2

Page No.27
MARCH CURRENT AFFAIRS_2019
Key : C parties across the political spectrum have generally
Explanation : adhered to its letter and spirit.
• ISRO and its French counterpart CNES has sealed an • The Code was issued for the first time in 1971 before
agreement to set up a joint maritime surveillance the 5th Lok Sabha elections. Since then, it has been
system in the country. issued before every central and state election and
• The two nations will explore putting up a revised from time to time. The broad objectives of the
constellation of low-Earth orbiting satellites. code of conduct are to ensure a level playing field for
all political parties, prevent conflicts between parties,
Oceansat-3-Argos Mission and ensure law and order in the heat of election
• The system will be augmented with the launch of season. Still, its primary purpose is to ensure that the
Oceansat-3-Argos mission in 2020 along with a joint ruling party does not misuse or use to its advantage
infrared Earth-observation satellite. the government machinery for its election campaign
• These will identify and track movement of ships purpose. This function of the code is the primary
globally – and in particular those moving in the Indian reason for its indisputable legitimacy across all
Ocean region where France has its Reunion Islands. parties. Each party follows the code in the hope that
• Before that, they will initially share data from their other parties, especially the ruling party, will also
present space systems and develop new algorithms follow it and hence no party will gain unfair
to analyse them, according to the Paris based National advantage in the elections. Another reason for the
Centre for Space Studies. success of the code lies in its restrained nature that
• They work together for the design and development of the parties find tolerable to follow.
joint products and techniques, including those • The Code of Conduct comes into force immediately
involving Automatic Identification System (AIS), to after the elections are announced by the EC. It applies
monitor and protect the assets in land and sea. to political parties, their candidates and polling
Other collaborations agents, the government in power and all government
employees. It prevents the ruling party from using
• The two agencies have put up two climate and ocean
official machinery for electioneering work. Public
weather monitoring satellites Megha Tropiques (of
places for holding election rallies and helipads for
2011) and SARAL-AltiKa (2013) that is considered a
flights in connection with elections are to be made
model.
available to all parties on the same terms and
Objective : conditions on which they are used by the party in
To know about the Ocean sat – 3- Argos and joint power. Ministers and other government authorities
infrared Earth observation satellite. should not announce any financial grants to the
people; they should not lay foundation stones for or
Polity inaugurate any projects; they should not promise
public facilities like roads; and they should not make
Elections and representation
any ad hoc government appointments.
of people act
Objective :
77)“The code of conduct comes into force immediately
To know about the Model Code of Conduct before
after the elections are announced by the Election
elections conducted in India.
Commission”. In this context, the code of conduct
applies to: Economy
1.Political parties Growth and Development
2.Candidates of their political parties 78)Which of the following is/are correct with regards to
features of the Special Economic Zones (SEZ)?
3.Polling agents
1.SEZ are allowed for manufacturing, trading and
4.The government in power
service activities
5.All government employees
2.The units must become net foreign exchange earners
Select the correct answer using the code given below within 3 years
A) 1, 2 and 3 only B) 1, 2, 3 and 4 only 3.A SEZ does not require a license for imports.
C) 1, 2, 4 and 5 only D) 1, 2, 3, 4 and 5 Select the correct answer using the code given below
Key : D A) 1 and 2 only B) 2 and 3 only
Explanation : C) 1 and 3 only D) 1, 2 and 3
• The Model Code of Conduct is a set of guidelines issued Key : D
by the Election Commission of India to regulate the
Explanation :
conduct of political parties and their candidates in
the run up to elections and is aimed at ensuring free • Special Economic Zones Act, 2005, was passed by
and fair elections. Though the Code doesn’t have any Parliament in May, 2005 and received Presidential
statutory basis, it has an indisputable legitimacy and assent on the 23rd of June, 2005. The act envisages
that the SEZs would attract a large flow of foreign and

Page No.28
MARCH CURRENT AFFAIRS_2019
domestic investment in infrastructure and productive 2. For the first time, the rules included the provision
capacity leading to generation of additional economic to distinguish between hazardous waste and other
activity and creation of employment opportunities. wastes.
Salient Features: Which of the above statement (s) is/are correct using
• A SEZ is a designated duty free enclave to be treated the code given below
as foreign territory for the purpose of trade operations A) 1 only B) 2 only
and duties and tariffs. C) Both 1 and 2 D) Neither 1 nor 2
• A SEZ does not require a license for imports. Other Key : C
notable features are as follows: Explanation :
• The units must become net foreign exchange earners • The Hazardous and Other Wastes (Management &
within 3 years Transboundary Movement) Rules, 2016 were
• SEZ are allowed manufacturing, trading and service established on April 2, 2016 to ensure resource
activities. recovery and disposal of hazardous waste in an
• Full freedom for subcontracting. environmentally sound manner.
• The domestic sales from the SEZ are subject to full • For the first time, the rules included the provision to
custom duties and import policy is in force, when distinguish between hazardous waste and other
they sell their produce to domestic markets. wastes. The other wastes that comprise waste tyre,
• There was no routine examination by the custom paper waste, metal scrap and used electronic items
authorities. were recognised as a resource for recycling and reuse.
• The corporation in SEZs will not have to pay any • Under the rules, the import of metal scrap, paper
income tax on their profits for the first five years and waste and various categories of electrical and
only 50% of the tax for 2 more years thereafter. electronic equipment for re-use purpose was exempted
• If half of the profit is reinvested in the corporation, from the need of obtaining Environment Ministry’s
the concession of 50% tax is extendable for next 3 permission.
years. • Further, the basic necessity of infrastructure to
• For SEZ developers , the raw material from cement to safeguard the health and environment from waste
steel to electrical parts are subject to zero tax and processing industry was prescribed as Standard
duty. Operating Procedure (SOPs) specific to waste type.
• For the SEZ, the Government acquires vast land tracts • The rules also outlined the responsibilities of state
and gives to the developers. The basic condition governments for environmentally sound management
involves that 25% of the area of the SEZ must be used of hazardous and other wastes.
only for export related activities. Rest 75% area can • Under the rules, the state governments are required
be used for economical and social infrastructure. to allot industrial space for recycling, pre-processing
However, all SEZ benefits are applicable over the and other utilisation of hazardous or other waste as
entire SEZ area. well as to register the workers involved in recycling,
• There were provisions for sector specific SEZs and pre-processing and other utilisation activities.
Multiproduct SEZs. • Further, the rules prohibit the import of waste edible
• The Sector specific SEZ may have 7500 houses, hotels fats and oil of animals, or vegetable origin, household
with 100 rooms, 25 bed hospital , schools and other waste, critical care medical equipment, tyres for
institutions, a multiplex in 50000 sq. meters. direct re-use purpose, solid plastic wastes including
pet bottles, waste electrical and electronic assemblies
• Multiproduct SEZ are allowed to build 25000 houses. scrap and other chemical wastes especially in solvent
250 room hotel and 100 bed hospital along with a form.
multiplex with 2 lakh sq. meters.
• The rules also directed the State Pollution Control
Objective : Board to prepare an annual inventory of the waste
To know about the Features of Special Economic Zone. generated, waste recycled, recovered, utilised
including co-processed, waste re-exported and waste
Geography disposed of and submit the same to the Central
Disaster management Pollution Control Board by September 30 every year.
79)Consider the following statements with regards to the • The rules further identified hazardous waste as any
Hazardous and other Waste (Management & Tran waste, which by reason of characteristics, such as
boundary Movement) Rules, 2016: physical, chemical, biological, reactive, toxic,
flammable, explosive or corrosive, causes danger to
1. It is to ensure resource recovery and disposal of
health, or environment.
hazardous waste in an environmentally sound
manner. • It mandated scientific disposal of hazardous waste
through collection, storage, packaging, transportation
and treatment, in an environmentally sound manner,

Page No.29
MARCH CURRENT AFFAIRS_2019
minimizing the adverse impact on human health and together, are called). The quantity of a good that the
environment. consumer chooses can increase or decrease with the
• Under the rules, the hazardous waste can be disposed rise in price of a related good depending on whether
at a captive treatment facility installed by the the two goods are substitutes or complementary to
individual waste generators or at Common Hazardous each other.
Waste Treatment, Storage and Disposal Facilities Objective :
(TSDFs). To know about the Types of goods and demand or
• There are 40 Common Hazardous Waste Treatment, consumer choice according to income of a consumer.
Storage and Disposal Facilities (TSDFs) available in
17 states and union territories. 81)Which of the following is/are conditions required to
Objective : satisfy that the product or service is said to have a
To know about the Hazardous and other Waste demand?
(Management & Tran boundary Movement) Rules, 2016 1.Desire on the part of the buyer to buy
2019-03-19 2.Willingness to pay for it
Economy 3.Ability to pay the specified price for it
Basics of Economy Select the correct answer using the code given below
80)Select the incorrect from the following statements with A) 1 and 2 only B) 2 and 3 only
regards to different types of goods in an economy: C) 1 and 3 only D) 1, 2 and 3
A) For most of the goods, the quantity that a consumer Key : D
chooses increases as the consumer’s income increases
Explanation :
and Decreases as consumer’s income decreases. Such
goods are called Normal goods. Demand:
B) For complementary and substitute goods, the price • Every wants supported by the willingness and ability
of one goods affects the demand of the other and vice to buy, constitutes demand for a particular product
versa. or service. In other words if I want a car and I cannot
pay for it, there is no demand for the car from my
C) Tea & Sugar, Pen & Ink, these combinations of goods
side.
are the example for Substitutes.
A product or service is said to have a demand when
D) As income of a consumer increases the demand for
three conditions satisfied
the inferior goods falls and vice versa.
1.Desire on the part of the buyer to buy
Key : C
2.Willingness to pay for it
Explanation :
3. Ability to pay the specified price for it.
• The quantity of good that the consumer demand can
increase or decrease with the rise in income, Unless all these conditions are fulfilled, the product
depending on the nature of the good is not said to have any demand.
• For most of the goods, the quantity that a consumer A product with more no. of uses is naturally more in
chooses increases as the consumer’s income increases demand than one with single use.
and decreases as the consumer’s income decreases. Objective :
Such goods are called Normal Goods. Thus a To know about the Demand analysis of goods or
consumer’s demand for a normal goods move in the services.
direction as the income of a consumer.
• However, there are some goods the demand for which 82)Which of the following is/are the factors (s)
moves in the opposite direction of the income of the determining the Demand of a particular product?
consumer. Such goods are called as inferior goods.
1.Price of the product
As the income of the consumer increases, the demand
for the inferior goods falls, and the income decreases, 2.Income of a consumer
the demand for inferior goods increases. 3.Price of a relative goods
• For complementary and substitutes the price of one 4.Expected prices of a good in future
good affects the demand of the other. 5.Size of a population
• Complementary goods: examples are Tea & Sugar, Pen Select the correct answer using the code given below
& Ink, and here as the price of the sugar increases A) 1, 2 and 5 only B) 1, 2, 3 and 5 only
then the demand of the tea decreases.
C) 1, 2, 4 and 5 only D) 1, 2, 3, 4 and 5
Substitutes: Examples are Tea & Coffee, here as the price
Key : D
of the coffee increases then the demand on the Tea
increases. Explanation :
• Option c is incorrect as Tea & sugar, Pen & Ink are the • The demand for a particular product depends on
complementary goods (or goods, which are consumed several factors, the following are:

Page No.30
MARCH CURRENT AFFAIRS_2019
1.Price of the product (p) A) 1 only B) 2 only
2.Income of a consumer (I) C) Both 1 and 2 D) Neither 1 nor 2
3.Taste and preferences (T) Key : A
4.Price of relative goods which may constitute Explanation :
substitutes/complementary (PR) • Several factors are collectively responsible for
5.Expected prices in future (EP) inflationary rise in the general price level. These are
6.Expected income in future (EI) broadly classified as “Demand Pull” and “Cost Push”
7.Size of a population (SP) factors. The former sets of factors are those due to
which there is an overall rise in the demand for Goods
8.Distribution of consumers over different regions (DC)
and Services in general. On the other hand, the latter
9.Advertising efforts (A) are those factors due to which there is a shortfall/
10.Any other factor capable of effecting the demand scarcity in the supply of goods and services and/or
(O) an overall increase in the cost of production/
Demand function: distribution of Goods and Services.
• Qd = f (P, I, T, PR, EP, EI, SP, DC, A, O) • Aggregate Demand (AD) = Government Expenditure
Objective : (GE) + Investment Expenditure (IE) + Consumer
Expenditure (CE) + other Expenditure (OE) as shown in
To know about the Factors determining the Demand
below figure.
83) Which of the following is true with regards to Income
• Form the above figure, as Aggregate Demand curve
elasticity of Demand (e) for superior goods and
shifts up, due to increase of any expenditure, it leads
inferior goods respectively?
to inflation called as Demand Pull inflation.
A) e>1, e<1 B) e<1, e>1
• Cost push inflation is due to supply related factors
C) e>1, e=1 D) e<1, e=1 like scarcity of supply.
Key : A Objective :
Explanation : To know about the Types of and causes for inflation.
Income Elasticity in Demand:
• It refers to the Quantity of demand of a commodity in 85)Which of the following is/are describe/describes as
response to a given change in income of a consumer. a reason (s) for License Raj Regime?
Income elasticity is normally positive, which
1.Equal distribution of the wealth to all the population
indicates that the consumer tends to buy more and
more with every increase in income. Income elasticity 2.In order to reduce concentration of economic power
of a Demand is the ration between Proportionate in hands of few people
change in quantity of Demand for product X and 3.Regional Balanced development of India
Proportionate change in Income. 4.Encouragement to the village and small scale
• Here the demand is more responsive to a change in enterprise
income. Select the correct answer using the code given below
• For superior good as income increases then Demand A) 1, 2 and 3 only B) 2, 3 and 4 only
increases and as income decreases Demand C) 1, 3 and 4 only D) 1, 2, 3 and 4
decreases, it means Both Income and Demand goes Key : D
positive. Hence Elasticity of Demand is greater than
1. For inferior goods it is negative hence Elasticity of Explanation :
Demand is less than 1. Reasons for the License Raj:
Objective : 1.Equal Distribution of the wealth to the all population
To know about the various elasticity of demand and 2.In order to reduce concentration of economic power
its analysis with respect to goods in hands of few people
84)“Aggregate Demand is the sum of all expenditures done 3.Regional Balanced Development of India.
by Government, Firms, Consumers and all some other 4.Encouragement to the village and small scale
expenditures” in this context what happens when any enterprises.
of the expenditure increase? Objective :
1. Increase of government expenditure leads to upward To know about the IPR 1956 or License raj or License
shift of Aggregate Demand Curve, which leads to Raj Quota regime in Indian Industrial Policy
inflation, such inflation called as Demand Pull Resolution.
Inflation Industry
2.Increase of Investment expenditure leads to upward 86)Which of the following is the main thrust of Industrial
shift of Aggregate Demand curve, which leads to Policy Resolution (IPR), 1948?
Inflation, such inflation called as Cost Push Inflation.
Select the correct answer using the code given below

Page No.31
MARCH CURRENT AFFAIRS_2019
A) The main thrust of 1948 IPR was to lay the Service sector
foundation of a mixed economy in which both private 88)Consider the following statements with regards to
& public enterprises would march hand in hand to Insurance Regulatory and Development Authority of
accelerate the pace of Industrial development. India (IRDAI):
B) The main thrust of IPR 1948 was encouraging only 1.It is the apex non statutory body that regulates and
Public enterprises to accelerate the pace of Industrial develops insurance Industry in India.
Development 2.Current FDI cap in Insurance sector is 26%
C) Private enterprises were allowed in category like Which of the above statements is/are incorrect using
Railway, arms and ammunition etc of IPR 1948 to the code given below?
develop the industries in India.
A) 1 only B) 2 only
D) None of the above.
C) Both 1 and 2 D) Neither 1 nor 2
Key : A
Key : C
Explanation :
Explanation :
• The IPR of April, 1948 contemplated a mixed economy
reserving a sphere for the private sector and another • Insurance Regulatory and Development Authority of
for public sector. India or the IRDAI is the apex body responsible for
regulating and developing the insurance industry in
• The Industries were divided into 4 broad categories India. It is an autonomous body. It was established
they are by an act of Parliament known as the Insurance
(a) The Manufactures of arms and ammunition, the Regulatory and Development Authority Act, 1999.
production and control of atomic energy were to be IRDAI is headquartered in Hyderabad in Telangana.
the “exclusive monopoly of the central government”. Prior to 2001, it was headquartered in New Delhi. The
(b) The second category covered coal, iron & steel, organization fought for an increase in the FDI limit in
aircraft manufacture, ship building manufacture of the insurance sector to 49% from the previous 26%.
telephone & telephone and wireless apparatus etc. The FDI cap was hiked to 49% in July of 2014.
be undertaken only by the state. • Its primary purpose is to protect the rights of the
(c) The third category was made up of industries of policyholders in India.
such basic importance that the central government • It gives the registration certificate to insurance
would feel it necessary to plan and regulate them. companies in the country.
(d) A fourth category comprising the remainder of the • It also engages in the renewal, modification,
industrial field was left open to private enterprise, cancellation, etc. of this registration.
individual as well as co operative. • It also creates regulations to protect policyholders’
Objective : To know about the IPR, 1948. interests in India.
Objective : To know about the IRDAI
87)Which of the following Industrial Policy Resolution
(IPR) was called the License Raj Regime or License Raj International Organisations
Quota?
89)The Financial Action Task Force (FATF) is an inter-
A) IPR, 1948 B) IPR, 1956 governmental Organisation established with an aim
C) Industrial Reforms, 1991 to:
D) Foreign Exchange Regulation Act (FERA), 1973. A) Combating money laundering, terrorist financing
Key : B and other related threats to the integrity of the
Explanation : international financial system.
• The IPR, 1956 had accepted the socialist pattern of B) Checks and regulates the corruption between the
society as the basic aim of social and economic policy. countries financial organisations under the purview
The resolution laid down three categories which bear of IMF.
a close resemblance to the earlier classification. They C) Regulates the Central Banks of different countries
are over an issue of loan to Individual or an organisation,
i) Schedule A ii) Schedule B which are insolvent to pay back.
iii) Schedule C D) None of the above
• For Schedule A and Schedule B industries require the Key : A
permission from union government this regime is Explanation :
known as “License Raj Regime” or “License Quota • The Financial Action Task Force (FATF) is an inter-
Regime”. governmental organisation established to set
Objective : To know about the various IPR standards and promote effective implementation of
legal, regulatory and operational measures for
combating money laundering, terrorist financing and
other related threats to the integrity of the

Page No.32
MARCH CURRENT AFFAIRS_2019
international financial system. It was formed in 1989 3.Apart from these lucrative commercial benefits,
by the G7 Summit which was held in Paris. The FATF is china also has huge strategic and geopolitical
really a policy-making body that works with advantages in the Indian Ocean region
governments to bring about national legislation and Select the correct answer using the code given below
regulatory reforms in these areas. A) 1 and 2 only B) 2 and 3 only
FATF Recommendations C) 1 and 3 only D) 1, 2 and 3
• The FATF has developed a series of recommendations Key : D
that have become global standards for fighting money
Explanation :
laundering and terror financing, as well as the
proliferation of weapons of mass destruction. The • China-Pakistan Economic Corridor is a whopping 46
first set of recommendations was issued in 1990 billion dollar project which will connect Kashgar in
followed by revisions in 1996, 2001, 2003 and lastly Xinjiang province of China, with Gwadar port in
in 2012. Baluchistan which is the largest province in Pakistan.
It is connected through a vast and complex network
FATF Activities
of roads as well as other infrastructure projects such
• The organisation monitors its member countries on as dams, hydropower projects, railways, and
the progress they have made in implementing reform pipelines.
measures, and reviews their counter-measures to
What is Pakistan’s interest in CPEC?
money laundering and terror financing.
• The decision-making body of FATF is called the FATF • Under the ‘1+4’ cooperation framework of CPEC, ‘1’
Plenary. It meets three times annually. represents ‘CPEC’ and the 4 includes Energy, Gwadar
Port, Transport Infrastructure and Industrial
• The organisation included combating terror financing cooperation. The Chinese investments are supposed
among its objectives after the September 11 terror to boost Pakistan’s 274 billion dollar GDP by over 15
attacks on the USA in 2001. percent.
FATF Blacklist • Energy needs of Pakistan: The large scale energy
• Officially called the “Non-Cooperative Countries or production has been termed as the biggest
Territories (NCCTs), the FATF Blacklist is a list of breakthrough of the project. Energy projects such as
countries which the FATF considers to be non- Karot Hydropower project, Karachi Nuclear power
cooperative in the international fight against terrorist plant and World’s largest solar power plant in
financing and money laundering. This list is regularly Pakistan’s Punjab Province etc are part of this
updated, with countries being either deleted off the initiative that can double Pakistan’s energy capacity.
list of new countries being added to the list. It is to be • Infrastructure development: Like any other developing
noted that some names are on the list not because of country Pakistan is also suffering due to lack of basic
their non-cooperative stance towards fighting this infrastructure facilities. With CPEC, Pakistan expects
menace, but because they lack the necessary infrastructural enhancement which includes
infrastructure and mechanisms to engage in this fight. construction of a 2,000 km of road and rail network
FATF Countries (FATF Members) worth 10.63 billion dollars.
• Currently, there are 38 member countries in the FATF. • Employment Opportunities: This corridor promises
India is a member of the organisation since June 2010. huge employment opportunities to all sections of the
It had earlier acquired ‘Observer’ status at FATF in society.
2006. • Away from Western Influence: CPEC provides Pakistan
• FATF Observers are two in number, namely, Saudi with an opportunity to work closely with a more
Arabia and Indonesia. There are many other Observer reliable friend China independent of Western
organisations and Associate Members. influence especially the US.
Objective : What is China’s interest in CPEC?
To know about the Financial Action Task Force (FATF). • Access to the Middle East and Europe: By materialising
the CPEC project, China can save millions of dollars
90)“China-Pakistan Economic Corridor (CPEC) is a project every year by shortening its route by about 12,000 km
which will connect Kashgar in Xinjiang province of which is critical for energy imports.
China, with Gwadar port in Baluchistan, Pakistan”. • Gwadar Port: A fully operational Gwadar port in
Which of the following could be the possible benefits Pakistan provides the following benefits.
that Gwadar port provides? • It provides a link between Maritime Silk Road and the
1.A link between Maritime Silk Road and the Arabian Arabian Sea.
Sea. • The port at the mouth of the Persian Gulf provides
2.The port at the mouth of the Persian Gulf provides China with the shortest route to the oil-rich Middle
China with the shortest route to the oil rich middle East, Africa, and most of the Western hemisphere.
east, Africa, and most of the Western hemisphere. Gwadar will have the estimated capacity to handle
19 million tonnes of crude oil per year, which will be

Page No.33
MARCH CURRENT AFFAIRS_2019
sent to China after being refined at the port. At present, • The Environment Assembly meets biennially (not
China transports 80% of its oil through the Strait of annually) to set priorities for global environmental
Malacca. policies and develop international environmental law.
• Apart from these lucrative commercial benefits, China • The Assembly is the governing body of the UN
also has huge strategic and geopolitical advantages Environment Programme (UN Environment) and the
in the Indian Ocean region. successor of its Governing Council, which was
Objective : composed of 58 member States. The UN Environment
To know about the China – Pakistan Economic Corridor Assembly, with a universal membership, is now
(CPEC). composed of 193 Member States.
Objective :
International Relations To know about the United Nations Environment
Assembly and its functions
International Institutions,
agencies and their structure
Governance
and mandate
Government policies and
91)Consider the following statements with respect to
International Criminal Court (ICC) programs
1.It was established through the Rome Statute. 93)Which of the following statements about Namami
Gange is/are correct
2.Its headquarters is located in The Hague,
Netherlands. 1.The National Mission for Clean Ganga implements the
Namami Gange programme.
3.India is not a party to the ICC.
2.The Namami Gange is an Integrated Conservation
Which of the statement(s) given above is/are correct?
Mission which seeks effective abatement of pollution,
A) 1 and 2 only B) 1 and 3 only conservation and rejuvenation of River Ganga.
C) 2 only D) 1,2 and 3 A) 1 only B) 2 only
Key : D C) Both 1 and 2 D) Neither 1 nor 2
Explanation : Key : C
• International Criminal Court (ICC) was created by the Explanation :
‘Rome Statute’.
• The Namami Gange is an Integrated Conservation
• The Rome Statute is multilateral treaty which serves Mission which seeks effective abatement of pollution,
as ICC’s foundational and governing document. conservation and rejuvenation of River Ganga. The
• India is not a signatory to the Statute. NMCG implements the Namami Gange programme.
• USA and China is also non-signatory of the statute. • NMCG is the implementation wing of the National
• Its headquarters is located in The Hague, Netherlands. Ganga Council. It was registered as a under the
Objective : Societies Registration Act 1860. The government
through the River Ganga (Rejuvenation, Protection and
To know abou the International Criminal Court (ICC)
Management) Authorities Order, 2016, declared NMCG
and whether India is a signatory to it or not.
as an Authority. It has been powers to issue directions
and also to exercise the powers under the Environment
92)Consider the following statements about UN (Protection) Act, 1986
Environment Assembly Objective :
1.The United Nations Environment Assembly is the To know about the various government schemes and
world’s highest-level decision-making body on the policies
environment
94)Consider the following statements with regards to
2.The Environment Assembly meets annually to set Pradhan Mantri Shram Yogi Maan-Dhan Yojana
priorities for global environmental policies and (PMSYM):
develop international environmental law
1.It is a voluntary and contributory pension scheme
Select the correct statements using code given below which will engage as many as 42 crore workers in the
A) 1 only B) 2 only un-organised sector
C) Both 1 and 2 D) Neither 1 nor 2 2.The unorganised sector workers, with income less
Key : A than Rs 15,000 per month and who belong to the entry
Explanation : age group of 18-40 years, will be eligible for the
scheme.
• The United Nations Environment Assembly is the
world’s highest-level decision-making body on the 3.Each subscriber under the scheme will receive
environment. It addresses the critical environmental minimum assured pension of Rs 3000 per month after
challenges facing the world today. attaining the age of 60 years.

Page No.34
MARCH CURRENT AFFAIRS_2019
Which of the above statement (s) is/are correct using Objective :
the code given below To know about the Pradhan Mantri Shram Yogi Maan
A) 1 and 2 only B) 2 and 3 only – Dhan Yojana.
C) 1 and 3 only D) 1, 2 and 3 95)Consider the following statements with regards to
Key : D Corruption Perception Index (CPI):
Explanation : 1.Corruption Perception Index (CPI) is based on how
• The Union Ministry of Labour and Employment has corrupt a country’s public sector and judiciary are
launched the Pradhan Mantri Shram Yogi Maan-dhan perceived to be.
Yojana (PM-SYM), a mega pension scheme for 2.It does not consider corruption in the business
unorganised sector. The scheme was announced in sector.
the Interim Budget 2019. 3.CPI is released by Transparency International, an
• About Pradhan Mantri Shram Yogi Maan-Dhan Yojana: international non-governmental organization.
• PM-SYM is a voluntary and contributory pension 4.On a scale of 0-100, a score of 0 points perceived
scheme that will engage as many as 42 crore workers as a country with sound integrity systems and 100
in the unorganised sector. points perceived as highly corrupt.
Eligibility: Which of the above statement (s) is/are correct using
• The unorganised sector workers, with income of less the code given below
than Rs 15,000 per month and who belong to the entry A) 1, 2 and 3 only B) 2, 3 and 4 only
age group of 18-40 years, will be eligible for the C) 1, 3 and 4 only D) 1, 2, 3 and 4
scheme. Key : A
• Those workers should not be covered under New Explanation :
Pension Scheme (NPS), Employees’ State Insurance • Corruption Perceptions Index of Transparency
Corporation (ESIC) scheme or Employees’ Provident International ranks countries and territories based
Fund Organisation (EPFO). on how corrupt a country’s public sector and judiciary
• He or she should not be an income tax payer. are perceived to be by experts and business
Benefits: executives.
• Minimum Assured Pension: Each subscriber under the • It is the most widely used indicator of corruption
scheme will receive minimum assured pension of Rs worldwide and it does not consider corruption in the
3000 per month after attaining the age of 60 years. business sector. The corruption perception index
• In case of death during receipt of pension: If the draws on 13 surveys and expert assessments to
subscriber dies during the receipt of pension, his or measure public sector corruption in 180 countries
her spouse will be entitled to receive 50 percent of and territories.
the pension as family pension. This family pension is • The scores indicate the perceived level of public
applicable only to spouse. sector corruption on a scale of 0-100. A score of 0
• In case of death before the age of 60 years: If a points that a country is perceived as highly corrupt
beneficiary has given regular contribution and dies and a 100 points that a country has sound integrity
before attaining the age of 60 years, his or her spouse systems.
will be entitled to continue the scheme subsequently Where does India stand?
by payment of regular contribution or may even exit The Corruption perception index makes the following
the scheme. observations about India:
Contribution to the scheme: • India’s score has been marginally improved 41 from
• Contribution by the Subscriber: The subscriber is 40 in 2017.
required to contribute the prescribed contribution • India’s ranking increased from 81st in 2017 to 78 in
amount from the age of joining the scheme till the age 2018.
of 60 years. • India had slid from 79th rank in 2016.
• Medium of contribution: The subscriber can • The report cities countries like India along with
contribute to the PM-SYM through ‘auto-debit’ facility Malaysia, Maldives and Pakistan will be important
from his or her savings bank account or from his or to watch moving forward.
her Jan- Dhan account.
• The report notes that all these countries mentioned
• Equal contribution by the Central Government: Under above have witnessed massive public mobilisation
the PM-SYM, the prescribed age-specific contribution against corruption coupled with significant political
by the beneficiary and the matching contribution by participation and voter turnout resulting in new
the Central Government will be made on a ‘50:50 governments that promised extensive anti-corruption
basis’. reforms.
• The report mentions that these encouraging
developments are yet to show some tangible solid

Page No.35
MARCH CURRENT AFFAIRS_2019
action, especially when it comes to combating elusive • ‘Criminal contempt’ is ‘the publication (whether by
forms of grand corruption. words, spoken or written, or by signs, or by visible
• The report noted that since India gears up for general representation, or otherwise) of any matter or the
elections, there was a little significant movement in doing of any other act whatsoever which:
its CPI score, which moved from 40 in 2017 to 41 in
2018. 1. Scandalizes or tends to scandalize, or lowers or tends
• The report notes that in spite of spectacular public to lower the authority of, any court.
mobilisation in 2011, where citizens demanded the 2. Prejudices, or interferes or tends to interfere with the
government to take action against corruption and due course of any judicial proceeding.
advocated for the passage of the comprehensive Jan 3. Interferes or tends to interfere with, or obstructs or
Lokpal bill, the efforts ultimately fizzled and fell flat, tends to obstruct, the administration of justice in any
with little to no movement on the ground to build the other manner.’
specialist anti-corruption infrastructure required.
Objective :
• These findings gain importance at the time Supreme
To know about the details of Contempt of Court
Court has set the ball rolling for the appointment of
Lokpal by setting a deadline for the search committee
to recommend names for selection committee headed Non-Constitutional, statutory,
by Prime Minister. regulatory and other quasi
Objective : judicial bodies(Origin,
To know about the Corruption Perception Index (CPI) structure and functions)
of Transparency International. 97)Who among the following are part of the selection
Polity panel of lokpal?
Judiciary 1. Prime Minister
96)Consider the following statements with respect to the 2. Speaker of Lok Sabha
“Contempt of Court”: 3. leader of opposition in Lok Sabha
1. Article 129 and Article 215 of Indian Constitution 4. Chief Justice of India
empowers the Supreme Court and the High Courts
5. An Eminent jurist to be nominated by President
respectively, to punish people for their contempt.
Select the correct answer using the code given below:
2. Criminal contempt is a willful disobedience to any
judgment, decree, direction, order, writ or other A) 1, 2 and 3 only B) 1, 2, 3 and 4 only
processes of a Court C) 1, 3, 4, and 5 only D) 1, 2, 3, 4 and 5
Which of the statement(s) given above is/are correct? Key : D
A) 1 only B) 2 only Explanation :
C) Both 1 and 2 D) Neither 1 nor 2 • The selection panel led by Prime minister is likely to
Key : A name Retired Supreme Court judge Pinaki Chandra
Ghose as India’s first Lokpal,the anti-corruption
Explanation :
ombudsman.
• Article 129 and Article 215 empowers the Supreme
• The Government had enacted the Lokpal and Lokayukta
Court and the High Courts respectively, to punish
Act,2013.The Act allows setting up of anti-corruption
people for their contempt. Article 129, states that “The
ombudsman called Lokpal at the Centre and Lokayukta
Supreme Court shall be a court or record and shall
at the State-level.The Lokpal will cover all categories
have all the powers of such a court including the
of public servants,including the Prime Minister.But
power to punish for contempt of itself”.
the armed forces do not come under the ambit of
• The power of contempt is often invoked to ensure Lokpal.
compliance with the orders given by the courts and in
their execution, and for punishing those who are
responsible for the lapses in the manner of • The Act also incorporates provisions for attachment
compliance. and confiscation of property acquired by corrupt
means,even while the prosecution is pending.The
• Contempt jurisdiction is exercised to uphold the
States will have to institute Lokayukta within one year
dignity of the judicial system which includes within
of the commencement of the Act.
itself the dignity of courts and tribunals as well and
to ensure the majesty of judicial institutions so that • The Lokpal consists of a chairperson and a maximum
it may not be lowered. of eight members.The chairperson and at least half of
the members have to be current or former judges of
• ‘Civil contempt’ is a ‘willful disobedience to any
the Supreme Court or chief justices of high courts.
judgment, decree, direction, order, writ or other
processes of a Court or willful breach of an • The search panel setup for lokpal recommends at least
undertaking given to the court’. five names for chairperson and at least three times
the number of vacancies in the case of members.

Page No.36
MARCH CURRENT AFFAIRS_2019
• The names proposed by the search committee would 2. Earlier,only smaller projects of less than 25 MW in
be scrutinized by the selection committee headed by capacity were categorized as renewable energy
Prime Minister and comprising of Speaker of Lok projects.
Sabha, leader of opposition in Lok Sabha, Chief Justice Which of the statement(s) given above is/are correct?
of India or a sitting Supreme Court judge nominated A) 1 only B) 2 only
by CJI and an Eminent jurist to be nominated by
President of India on basis of recommendations of C) Both 1 and 2 D) Neither 1 nor 2
the first four members of the selection committee Key : C
through consensus. Explanation :
Objective : • Recently,Union Cabinet had approved a new
To know about the selection committee of Lokpal. hydroelectric policy aimed at boosting the
sector.According to the new policy,large hydro
projects will also be designated as renewable energy
Science & Technology projects.
Defence • Earlier,only smaller projects of less than 25 MW in
98)The Navy is set to induct the second Scorpene capacity were categorized as renewable energy
submarine Khanderi by early May. In this context, projects and were eligible for various incentives like
consider the following statements with respect to financial assistance and cheaper credit.
“PROJECT 75”: • This decision will help in meeting the targets set for
1. Under Project 75, six Scorpene submarines are being the renewable energy sector.India is aiming 175 GW
built with assistance and technology transfer from of renewable energy by 2022.
DCNS of France. Objective :
2. The Scorpene class submarines are a class of To know about the details of recently proposed new
nuclear powered submarines. hydroelectric policy.
3.The first of the series INS Kalvari is already
completed sea trials and commissioned into the
100)Which one of the following energies is the biggest
Indian Navy.
contributor to the India’s renewable energy sector?
Which of the statement(s) given above is/are correct?
A) Solar Energy B) Wind Energy
A) 1 and 2 only B) 2 and 3 only
C) Hydroelectric energy D) Tidal energy
C) 1 and 3 only D) 1, 2 and 3
Key : C
Key : C
Explanation :
Explanation :
• Recently,Union Cabinet had approved a new
• Under Project 75 India will construct 6 next generation hydroelectric policy aimed at boosting the
diesel submarines with Air Independent Propulsion sector.According to the new policy,large hydro
System (AIP) technology for the Indian Navy by 2022. projects will also be designated as renewable energy
The project is built with assistance and technology projects.
transfer from DCNS of France under deal signed in
• This policy has also changed the renewable energy
October 2005.
mix as well.Wind energy which contributed nearly
• The Scorpene class submarines are a class of diesel- 50% of all renewable energy capacity will now make
electric powered submarines. The first of the series up only 29.3%.Similarly, solar energy’s share will fall
INS Kalvari is already completed sea trials and from 34.68% to 21.61%.However,Hydro sector will see
commissioned into the Indian Navy. its share grow from just over 6% to over 41%.
• The other four submarines (Karanj, Vela, Vagir, Objective :
Vagsheer) are expected to be launched at nine-month
intervals after the INS Khanderi. To know about the details of recently proposed new
hydroelectric policy.
Objective :
2019-03-18
To know about Project 75 and Scorpene class
submarines. Environment Ecology
Environment & Ecology
101)The “Centre for Marine Living Resources & Ecology
Energy
(CMLRE)” is organizing, coordinating and promoting
99)The Union Cabinet has recently approved a new ocean development activities in the country, under:
hydroelectricity policy. Consider the following
statements in this regard: A) The Ministry of Environment, Forest and Climate
Change
1. According to the new policy,large hydro projects
B) The Ministry of Earth Sciences
will also be designated as renewable energy projects.
C) Ministry of Water Resources, River Development
and Ganga Rejuvenation

Page No.37
MARCH CURRENT AFFAIRS_2019
D) Ministry of Science & Technology Select correct statements using code given below
Key : B A) 1 and 2 only B) 1 and 3 only
Explanation : C) 2 and 3 only D) 1,2 and 3
• Union Minister for Earth Sciences, Dr Harsh Vardhan Key : D
inaugurated Atal Bhavan, the new campus of Centre Explanation :
for Marine Living Resources and Ecology (CMLRE), at • West Nile virus can cause a fatal neurological disease
Puthuvype, Cochin, in Kerala. in humans.
• Centre for Marine Living Resources & Ecology • However, approximately 80% of people who are
• The Centre for Marine Living Resources & Ecology infected will not show any symptoms.
(CMLRE) under the Ministry of Earth Science is • West Nile virus is mainly transmitted to people
organizing, coordinating and promoting ocean through the bites of infected mosquitoes.
development activities in the country which includes
mapping of the living resources, preparing inventory • The virus can cause severe disease and death in
of commercially exploitable living marine resources, horses.
their optimum utilization through ecosystem • Vaccines are available for use in horses but not yet
management and R&D in basic sciences on Marine available for people.
Living Resources and Ecology. • Birds are the natural hosts of West Nile virus.
Mandate of CMLRE Objective :
• To develop management strategies for marine living To know about the diseases caused by virus which
resources through Ecosystem monitoring and are frequently in news.
modelling efforts.
• Evolving, coordinating and implementing time CONSERVATION EFFORTS
targeted national /regional R&D programmes in the 103)The National Tiger Conservation Authority (NTCA) has
field of marine living resources and ecology through stated in the Supreme Court that Spotted Cheetahs
effective utilisation of Fishery and Oceanographic being translocated from Namibia would be kept at
Research Vessel Sagar Sampada. Nauradehi Sanctuary, which is located at:
• Strengthening of research on marine living resources A) Uttar Pradesh B) Jharkhand
and Ecology including the establishment of a data
C) Chhattisgarh D) Madhya Pradesh
centre for storage and dissemination of data/
information to end users. Key : D
• Coordinating the national programmes relating to Explanation :
Southern Ocean Living Resources (Antarctic marine • The National Tiger Conservation Authority (NTCA) has
living resources). stated in the Supreme Court that Spotted Cheetahs
• CMLRE was established at Kochi in 1998 by upgrading being translocated from Namibia would be kept at
the erstwhile Sagar Sampada Cell with exclusive Nauradehi sanctuary in Madhya Pradesh.
facilities for implementing the Marine Living • NTCA even stated that the International Union for
Resources Programme. Conservation of Nature (IUCN) has given no objection
• Marine Living Resources Programme envisages for the translocation and the decision to reintroduce
survey, assessment and exploitation of the marine spotted cheetahs in India was supported by Wildlife
living resources and studies on the response of marine Institute of India, Dehradun.
resources to changes in the physical environment. Cheetah Extinction from India
The studies are undertaken with an objective of • India’s last spotted cheetah had died in 1947. In 1952,
developing an ecosystem model for the management the animal was declared extinct in the country.
of the living resources in the Indian EEZ. The Fishery Cheetah Re-introduction Project
Oceanographic Vessel Sagar Sampada is fully utilized
• The central government had set up an expert panel for
for these studies.
reintroducing the cheetah in India. The panel
Objective : recommended that the home of the fastest animal
To know about the Centre for Marine Living Resources could be Kuno Palpur in Madhya Pradesh, Velavadar
& Ecology National Park in Gujarat and Tal Chapar sanctuary in
Rajasthan.
102)Recently West Nile virus is in news ; Consider the • NTCA, a statutory body under Union Environment
following statements regarding this virus Ministry had committed Rs.50 crore to State for this
1.It can cause a fatal neurological disease in humans. project in the year 2011. Cheetah Re-introduction
Project was conceived in the year 2012.
2.It is mainly transmitted to people through the bites
of infected mosquitoes. • But the activists went to the Supreme Court against
the decision. Supreme Court appointed a panel to
3.Birds are natural host of this virus.
review the project. The Supreme Court appointed panel

Page No.38
MARCH CURRENT AFFAIRS_2019
advised against the reintroduction citing India does Economy
not have habitat and prey density” to support Monetary Policy
cheetahs. The report even accused government-run 105)Consider the following statements with respect to
institutions of “incorrectly compressing” data to get “Domestic Systemically Important Bank”:
the approval of the apex court to seek the
reintroduction of the big cat from Africa. As a result, 1. Banks whose assets exceed 2% of GDP are
the project had gone into dead storage. considered part of Domestic Systemically Important
Banks.
• In the year 2018, the government of Madhya Pradesh
wrote to NTCA to revive the project. The NTCA has 2. In India, only one public sector bank is part of this
approached the Supreme Court seeking a clarification group.
that there is no blanket ban of the apex court on re- Which of the statement(s) given above is/are correct?
introduction of cheetahs in the country. A) 1 only B) 2 only
Objective : C) Both 1 and 2 D) Neither 1 nor 2
To know about the endangered species like Asiatic Key : C
Spotted cheetahs and National Tiger Conservation Explanation :
Authority (NTCA). • RBI categorizes IDBI Bank as Private Sector Bank; SBI,
ICICI and HDFC remain Systemically Important Banks.
Science & Technology • D-SIB means that the bank is too big to fail. According
Defence to the RBI, some banks become systemically important
104)Consider the following statements with respect to due to their size, cross-jurisdictional activities,
“Man-Portable Anti-tank Guided Missile (MPATGM)”: complexity and lack of substitute and interconnection.
1. MPATGM is a third-generation anti-tank guided Banks whose assets exceed 2% of GDP are considered
missile (ATGM), developed by DRDO. part of this group.
Significance:
2. MPATGM has a top attack capability and has a
maximum engagement range of about 25 kilometers. 1. Should such a bank fail, there would be significant
disruption to the essential services they provide to
Which of the statement(s) given above is/are correct?
the banking system and the overall economy.
A) 1 only B) 2 only
2. The too-big-to-fail tag also indicates that in case of
C) Both 1 and 2 D) Neither 1 nor 2 distress, the government is expected to support these
Key : A banks. Due to this perception, these banks enjoy
Explanation : certain advantages in funding. It also means that these
• Defence Research and Development Organisation banks have a different set of policy measures
(DRDO) has successfully test-fired indigenously regarding systemic risks and moral hazard issues.
designed and developed Man Portable Anti-Tank Objective :
Guided Missile (MPATGM) in the deserts of Pokhran To know about the details of Domestic Systemically
in Rajasthan. Important Banks.
• Man-Portable Anti-Tank Guided Missile (MPATGM) Fiscal Policy
incorporates many advanced features, including, 106)Which one of the following statements is not correct
image infrared radar (IIR) seeker with integrated with respect to the “Financial Stability and
avionics. Development Council (FSDC)”?
• MPATGM is a third-generation anti-tank guided A) It is a statutory body under FSDC Act, 2008
missile (ATGM), developed by DRDO in association
with VEM Technologies Ltd. B) The Union Finance Minister of India is the
chairperson of the council
• MPATGM is fitted with high-explosive anti-tank (HEAT)
warhead. C) Chairmen of RBI, SEBI, IRDA, PFRDA and IBBI are the
members of the Council
• MPATGM has a top attack capability and has a
maximum engagement range of about 2.5 kilometers. D) All of the above are correct
Key : A
• MPATGM capable of being fired from the shoulder and
can be used during day and night. Explanation :
• MPATGM are being developed to replace the French • The Financial Stability and Development Council
origin anti-tank guided missile Milan and Soviet semi- (FSDC) was constituted in December, 2010. The FSDC
automatic wire-guided missile Konkur in service with was set up to strengthen and institutionalize the
Indian Army. The test trail at Rajasthan was the fourth mechanism for maintaining financial stability,
overall development trial of the MPATGM. enhancing inter-regulatory coordination and
Objective : promoting financial sector development. An apex-
level FSDC is not a statutory body.
To know about Man-Portable Anti-Tank Guided Missile
(MPATGM).

Page No.39
MARCH CURRENT AFFAIRS_2019
• The Council is chaired by the Union Finance Minister Geography
and its members are Governor, Reserve Bank of India; Climatology
Finance Secretary and/or Secretary, Department of 108)Recently a powerful bomb cyclone has impacted
Economic Affairs; Secretary, Department of Financial several part of the US causing heavy floods. In this
Services; Chief Economic Adviser, Ministry of Finance; context, consider the following statements:
Chairman, Securities and Exchange Board of India;
Chairman, Insurance Regulatory and Development 1. Bomb cyclone is a mid-latitude cyclone that
Authority and Chairman, Pension Fund Regulatory intensifies rapidly with sudden and deep drop of
and Development Authority. It also includes the atmospheric pressure.
chairman of the Insolvency and Bankruptcy Board 2. Deep drops in barometric pressure occur when a
(IBBI). region of warm air meets one of cold air.
• Recently, the government through a gazette 3.The direction of bomb cyclone is counterclockwise
notification, had included ministry of electronics and in the Northern hemisphere.
information technology (MeitY) secretary in the FSDC Which of the statement(s) given above is/are correct?
in view of the increased focus of the government on A) 1 and 2 only B) 2 and 3 only
digital economy. C) 1 and 3 only D) 1, 2 and 3
• The Council deals, inter-alia, with issues relating to Key : D
financial stability, financial sector development,
Explanation :
inter–regulatory coordination, financial literacy,
financial inclusion and macro prudential supervision • The term bomb cyclone is used by meteorologists to
of the economy including the functioning of large indicate a mid-latitude cyclone that intensifies
financial conglomerates. No funds are separately rapidly.
allocated to the Council for undertaking its activities. • A bomb cyclone happens when atmospheric pressure
Objective : in the middle of the storm drops at least 24 millibars
over 24 hours, quickly increasing in intensity. The
To know about the details of Financial Stability and
lower the pressure, the stronger the storm.
Development Council (FSDC).
• Deep drops in barometric pressure occur when a region
of warm air meets one of cold air. The air starts to
Governance move and the rotation of the earth creates a cyclonic
Government policies and effect. The direction is counterclockwise in the
programs Northern hemisphere leading to winds that come out
107)Recently Union Government sets up Subhash Chandra of the northeast.
Garg Committee to Objective :
A) Look into the development and regulation of the To know about the details of bomb cyclone.
financial technology
B) Regulate online portals, including news websites, Polity
entertainment sites and media aggregators Non-Constitutional, statutory,
C) Reform the process of higher defence planning and regulatory and other quasi
national security strategy
judicial bodies(Origin,
D) Manage the Indian cricket board
structure and functions)
Key : A
109)With respect to Lokpal Act, 2013; Consider the
Explanation : following statements
• The Union Government has set up 8 member steering 1.The Leader of Opposition from the Rajya sabha is
committee look into the development and regulation one member of the Selection Committee.
of the Financial technology (ntech) sector in India. It
2.When no person has been recognised as the Leader
will be headed by Economic Affairs Secretary Subhash
of Opposition, the leader of the single largest group
Garg. The move follows an announcement by Finance
in opposition of the Government would be the a
Minister Arun Jaitley in the Budget.
member in selection committee.
• Committee’s objective is to consider various issues
Select the anwser using code given below
relating to development of ntech sector in India. It
will look into issues to make ntech-related regulations A) 1 only B) 2 only
more exible and generate enhanced entrepreneurship C) Both 1 and 2 D) Neither 1 nor 2
in area where India has distinctive comparative Key : D
strengths vis-à-vis other emerging economies Explanation :
Objective : • The Chairperson and Members of the Lokpal shall be
To know about the various committees appointed by appointed by the President after obtaining the
central government. recommendations of a Selection Committee.

Page No.40
MARCH CURRENT AFFAIRS_2019
The Selection Committee consist of: 1. It secures at least six percent of the valid votes polled
Prime Minister in the State at a general election, either to the House
Speaker of the House of the People of the People or to the Legislative Assembly of the
State concerned; and
Leader of Opposition in the House of the People
2. In addition, it wins at least two seats in the Legislative
Chief Justice of India or a Judge of the Supreme Court Assembly of the State concerned. OR
nominated by him
3. It wins at least three percent (3%) of the total number
One eminent jurist
of seats in the Legislative Assembly of the State, or at
• There is no provision in the Lokpal Act to include the least three seats in the Assembly, whichever is more.
leader of the single largest party in the opposition as
Objective :
a member of the selection committee.
To know about the important provisions of the
• Congress leader Mallikarjun Kharge has, for the
Representation of the People Act, 1951.
seventh time, rejected the government’s invitation to
him to attend the meeting of the Lokpal selection
panel, arguing that there was no provision for a Internal Security
special invitee to attend such a meet Various security forces and
Objective : agencies and their mandate
Recently lokpal is in news. To know about the 111)Consider the following statements with respect to
provisions of the act this question is given. Man Portable Anti-Tank Guided Missile (MPATGM)
1. It is an indigenous missile developed by DRDO
Elections and representation 2. It is a low weight system based on fire and forget
of people act principle
110)Which one of the following provisions is governed 3. The system is equipped with state-of-the-art Imaging
by the provisions of Section 29A of the Representation Infrared Radar (IIR)
of the People Act, 1951? Select the correct statements using code given below
A) Model Code of Conduct A) 1 only B) 1 and 2 only
B) Registration of Political parties C) 2 and 3 only D) 1,2 and 3
C) Electoral rolls for parliamentary constituencies Key : D
D) Notification for election to the Council of States Explanation :
Key : B • The DRDO developed Man Portable Anti-Tank Guided
Explanation : Missile (MPATGM) was successfully test fired in the
• Registration of Political parties is governed by the desert ranges of Rajasthan recently.
provisions of Section 29A of the Representation of the • The indigenously developed missile has many
People Act, 1951. A party seeking registration under advanced features like a state-of-the-art Imaging
the said Section with the Commission has to submit Infrared Radar (IIR) seeker with integrated avionics.
an application to the Commission within a period of • The missile launched from the shoulder is low weight,
30 days following the date of its formation as per fire and forget missile.
guidelines prescribed by the Election Commission of Objective :
India in exercise of the powers conferred by Article To know about the recent developments in missiles
324 of the Commission of India and Section 29A of that are made by DRDO
the Representation of the People Act, 1951.
2019-03-15
• To be eligible for a ‘National Political Party of India,’
Science & Technology
the Election Commission has set the following
criteria: Concept, Scope and Nature of
1. It secures at least six percent of the valid votes Intellectual Property rights
polled in any four or more states, at a general election 112)Erode Turmeric has recently received GI tag from the
to the House of the People or, to the State Legislative Geographical Indication Registry. Erode Turmeric is
Assembly; and grown in which of the following state?
2. In addition, it wins at least four seats in the House A) Keral B) Karnataka
of the People from any State or States. OR C) Tamilnadu D) Maharashtra
3. It wins at least two percent seats in the House of Key : C
the People (i.e., 11 seats in the existing House having Explanation :
543 members), and these members are elected from
• Erode Turmeric has finally received GI tag from the
at least three different States.
Geographical Indication Registry after a long and
To be eligible for a ‘State Political Party,’ the Election cumbersome process which lasted for about 8 years.
Commission has set the following criteria: Erode is the city in the state of Tamil Nadu.

Page No.41
MARCH CURRENT AFFAIRS_2019
Features of Erode Turmeric Energy
• The claim for the uniqueness of the Erode Turmeric 114)Unlocking NATional Energy Efficiency potential
highlighted the following characteristics: (UNNATEE) is recently in news,which of following is/
• Mean length of the fingers of Erode turmeric was about are correct about UNNATEE?
4.15cm. 1.It is developed by Bureau of Energy Efficiency (BEE)
• Mean circumference was about 3.03cm. 2.It offers a comprehensive roadmap to address
• Mean bulb length of the mother rhizome is about India’s environmental and climate change mitigation
4.54cm. action through energy efficiency measures.
• Mean circumference is 6.54cm. A) 1 only B) 2 only
• Erode Turmeric is golden yellow in colour with C) Both 1 and 2 D) Neither 1 nor 2
resistance to pests after boiling. Key : C
• Erode turmeric contains 2.5 to 4.5% of curcumin Explanation :
content. • Bureau of Energy Efficiency (BEE) has developed a
• Erode Turmeric is grown in hot moist conditions national strategy document for accelerating energy
prevalent in the area with temperature ranging from efficiency in India.
20 degrees to 37.9 degrees Celsius with an average of • The strategy document titled UNNATEE (Unlocking
600 to 800 mm rainfall in a year. NATional Energy Efficiency potential) describes a plain
Objective : framework and implementation strategy to establish
To know about Erode turmeric. a clear linkage between energy supply-demand
scenarios and energy efficiency opportunities.
113)Recently cabinet approves a proposal to align Indian • The document offers a comprehensive roadmap to
Trademark standards with global trademark system. address India’s environmental and climate change
In this context, consider the following statements: mitigation action through energy efficiency measures.
1. Statutory protection of trademark in India is Objective :
administered by the Controller General of Patents, To know about Unlocking NATional Energy Efficiency
Designs and Trade Marks, Under Department for potential (UNNATEE) which is recently in news.
Promotion of Industry and Internal Trade.
2. The Nice Agreement on the International Current affairs on Science and
classification of goods and services for the figurative technology
elements of marks. 115)“IP Nani” sometimes seen in the news recently, is?
3. The V ienna Agreement for setting up an A) A program launched by Ministry of Women and
International classification of the purposes of Child development
registration of marks.
B) India’s first Intellectual Property Mascot
Which of the statement(s) given above is/are
C) Internet Protocol Standards developed by Facebook
incorrect?
A) 1 only B) 1 and 2 only D) Google’s project to promote Internet penetration in
remote areas
C) 2 and 3 only D) 1 2 and 3
Key : B
Key : C
Explanation :
Explanation :
• ‘IP Nani’ is India’s first Intellectual Property (IP)
• The Cabinet has approved the proposal to harmonise Mascot aimed at spreading awareness about the
the classification systems for examination of importance of Intellectual Property Rights (IPRs)
trademark and design applications, in line with the amongst children.
systems, followed globally. As a step forward in this
Objective :
direction the cabinet gas approved India’s accession
for the following agreements: To know about the recent developments in Intellectual
• Nice Agreement on the International classification of property right issues in India.
goods and services for the purposes of registration of
marks. Basic Science
• Vienna Agreement for setting up an International 116)Recently Karnataka government has flagged off a new
classification of the figurative elements of marks. cloud seeding programme “Project Varshadhare”.
• Locarno Agreement for establishing an International Consider the following statements with respect to
classification for industrial designs. ‘Cloud seeding’:
Objective : 1. Cloud Seeding is a process of intervening
To know about International Trademark agreements. chemically to induce precipitation rain or snow from
clouds.

Page No.42
MARCH CURRENT AFFAIRS_2019
2. The agents used as nuclei are iodides of silver or • A ‘secretive’ new species of orange-bellied frog with a
potassium, dry ice, or liquid propane. brown back, covered in tiny spots that resemble a
Which of the statement(s) given above is/are correct? starry sky, has been discovered recently in the
A) 1 only B) 2 only Western Ghats.
C) Both 1 and 2 D) Neither 1 nor 2 Objective :
Key : C To know about new species found in India
Explanation :
• Cloud Seeding is a process of intervening chemically 119)A ‘secretive’ new species of orange-bellied frog with
to induce precipitation rain or snow from clouds. a brown back resemble a starry sky is recently found
Rain occurs when moisture in the air reaches levels in
at which it can no longer be held. A) Eastern Himalayas
• Cloud seeding aims to facilitate and accelerate the B) Western ghats
process by making available chemical ‘nuclei’ for C) Eastern ghats of Andhra Pradesh
condensation to take place. The agents used as nuclei D) Andaman islands
are iodides of silver or potassium, dry ice (solid Key : B
carbon dioxide), or liquid propane.
Explanation :
Objective :
• A ‘secretive’ new species of orange-bellied frog with a
To know about the details of Cloud seeding. brown back, covered in tiny spots that resemble a
starry sky, has been discovered recently in the
Economy Western Ghats.
International Organisations Objective :
117)Consider the following statements about Asian To know about new species found in India
Development Bank
1.ADB is a regional development bank headquartered Environment & Ecology
in Manila, Philippines. 120)The sixth edition of “Global Environment Outlook”
2.The voting in ADB is based on ‘One Member-One report has been recently released. The report was
Vote’ System. released by
Which of the above statement(s) is/are correct? A) World wide Fund for Nature
A) 1 only B) 2 only B) United Nations Environment Programme
C) Both 1 and 2 D) Neither 1 nor 2 C) International Union for Conservation of Nature
Key : A D) Global Environmental Facility
Explanation : Key : B
• ADB is a regional development bank headquartered
in Manila, Philippines. Explanation :
• It admits the members of the UN Economic and Social • The Global Environment Outlook (GEO) is the UN
Commission for Asia and the Pacific and non-regional Environment Programme’s (UNEP) flagship
developed countries. environmental assessment. The first publication was
• The voting system is modeled closely on the World in 1997 and was originally requested by the Member
Bank where votes are distributed in proportion with States. The Global Environment Outlook Report 2019
members’ capital subscriptions. is the sixth edition.
Objective : • The report notes that a quarter of all premature deaths
To know about ADB and voting system in it. and diseases worldwide are due to manmade
pollution and environmental damage.
Environment Ecology • The report warns that deadly emissions, chemicals
Biodiversity polluting drinking water, and the accelerating
destruction of ecosystems crucial to the livelihoods
118)A ‘secretive’ new species of orange-bellied frog with of billions of people are driving a worldwide epidemic
a brown back resemble a starry sky is recently found that hampers the global economy.
in
• The report highlights the growing divide between rich
A) Eastern Himalayas B) Western ghats and poor as rampant overconsumption, pollution and
C) Eastern ghats of Andhra Pradesh food waste in the developed world leads to hunger,
D) Andaman islands poverty and disease elsewhere.
Key : B • The report notes that as greenhouse gas emissions
Explanation : continue to rise amid a preponderance of droughts,
floods and superstorms made worse by climbing sea

Page No.43
MARCH CURRENT AFFAIRS_2019
levels, there is a growing political consensus that advertisements posted on social media to their overall
climate change poses a future risk to billions. expenditure for the election.
• The report expresses concern that the health impacts Objective :
of pollution, deforestation and the mechanised food- To know about the details of Model Code of Conduct
chain are less well understood. (MCC).
• The report notes that poor environmental conditions
cause approximately 25% of global disease and Non-Constitutional, statutory,
mortality and resulted in around 9 million deaths in
regulatory and other quasi
2015 alone.
judicial bodies(Origin,
• Due to lack of access to clean drinking supplies, 1.4
million people die each year from preventable structure and functions)
diseases such as diarrhoea and parasites linked to 122)Consider the following statements with respect to
pathogen-riddled water and poor sanitation. Securities Appellate Tribunal (SAT)
• The report notes that chemicals pumped into the seas 1.It is a statutory body
causes potentially multi-generational adverse health 2.It deals with appeals filed against the orders of SEBI,
effects, and land degradation through mega-farming IRDAI and also PFRDA.
and deforestation occurs in areas of Earth home to A) 1 only B) 2 only
3.2 billion people.
C) Both 1 and 2 D) Neither 1 nor 2
• The report states that air pollution causes 6-7 million
Key : C
early deaths annually.
Explanation :
• The report calls for a root-and-branch detoxifying of
human behaviour while insisting that the situation is • Securities Appellate Tribunal (SAT) is a statutory body
not unassailable. Food waste for instance, which established under the Securities and Exchange Board
accounts for 9% of global greenhouse gas emissions, of India Act, 1992.It deals with appeals filed against
could be slashed. The world currently throws away a the orders of SEBI, IRDAI and also PFRDA.
third of all food produced. In richer nations, 56% goes • It has only one bench which sits at Mumbai and has
to waste. jurisdiction across whole of India.
Objective : Objective :
To know about “Global Environment Outlook” and its The Union Cabinet has recently approved the proposal
findings. for creation of the Post of Technical Member in
Securities Appellate Tribunal (SAT), Mumbai.In this
context one has to know about the SAT
Polity
2019-03-14
Constitutional bodies (Origin,
Economy
structure and functions)
International Organisations
121)Which of the following statement is incorrect about
Model Code of Conduct (MCC)? 123)1. Consider the following statements with regards to
“The International Food Policy Research Institute
A) Kerala was the first state to adopt a code of conduct
(IFPRI):
for elections.
1.IFPRI provides research based policy solutions to
B) Election Commission can take punitive action
sustainably reduce poverty and end hunger and
against it violations.
malnutrition in developing countries.
C) It does not apply to the content posted on social
2.Promoting Healthy Diets and Nutrition for all is one
media
of the strategic research areas.
D) It is not a legally enforceable document
Select the correct answer using the code given below
Key : C
A) 1 only B) 2 only
Explanation :
C) Both 1 and 2 D) Neither 1 nor 2
• Kerala was the first state to adopt a code of conduct
Key : C
for elections. In 1960, ahead of the Assembly
elections, the state administration prepared a draft Explanation :
code that covered important aspects of electioneering About IFPRI
such as processions, political rallies, and speeches. • The International Food Policy Research Institute
• On October 25, 2013, the Commission laid down (IFPRI) provides research-based policy solutions to
guidelines to regulate the use of social media by sustainably reduce poverty and end hunger and
parties and candidates. Candidates have to provide malnutrition in developing countries. Established in
their email address and details of accounts on Twitter, 1975, IFPRI currently has more than 600 employees
Facebook, YouTube, etc., and add the expenditure on working in over 50 countries. It is a research center

Page No.44
MARCH CURRENT AFFAIRS_2019
of CGIAR, a worldwide partnership engaged in • Its core role is to provide a secure transmission
agricultural research for development. channel so that Bank A knows that its message to
Vision and Mission Bank B goes to Bank B and no one else. Bank B, in turn,
• IFPRI’s vision is a world free of hunger and knows that Bank A, and no one other than Bank A,
malnutrition. Its mission is to provide research-based sent, read or altered the message en route. Banks, of
policy solutions that sustainably reduce poverty and course, need to have checks in place before actually
end hunger and malnutrition. sending messages.
What We Do Objective :
• Global, regional, and national food systems face Student has to know that Facilitation of payments is
major challenges and require fundamental not a component of shift mechanism.
transformations. More than ever, responding to these
challenges will require a systems-oriented, Inflation
multidisciplinary approach to reshape food systems 125)Recently cost push inflation is in news, Which of the
so they work for all people sustainably. IFPRI’s following will lead to cost push inflation for a
Strategy Refresh 2018–2020 builds on the strong base particular industry
of work developed under the Institute’s 2013–2018 1.Shortage of skilled labour
strategy and focuses on five strategic research areas:
2.Rising NPAs of banks
• Fostering Climate-Resilient and Sustainable Food
Supply A) 1 only B) 2 only
• Promoting Healthy Diets and Nutrition for All C) Both 1 and 2 D) Neither 1 nor 2
Key : C
• Building Inclusive and Efficient Markets, Trade
Systems, and Food Industry Technique : Elimination
• Transforming Agricultural and Rural Economies
• Strengthening Institutions and Governance Explanation :
Objective : • Cost push inflation refers to the type in which the rise
of prices is due to rise in costs of factors of production
To know about the international food policy research
– entrepreneurship, land, capital and labour.
institute (IFPRI).
• As the NPAs of banks increase, they will be asking for
stringent regulations and higher credit rate, which
124)SWIFT (Society for Worldwide Inter-bank Financial will make capital costly. Thus it is cost push inflation
Telecommunication) is recently in news,consider the type.
following statements
• If there is shortage of labour, the wages demanded by
1.SWIFT is global financial messaging service that the available will be higher, thus pushing cost of
enables financial institutions worldwide to send and production higher. Hence it is also cost push type.
receive information about financial transactions
Objective :
2.SWIFT does not facilitate funds transfer
• To know about the key concepts like inflation and its
Select the correct statements using code given below: factors
A) 1 only B) 2 only
C) Both 1 and 2 D) Neither 1 nor 2 Science & Technology
Key : C Current affairs on Science and
Explanation : technology
• SWIFT is global financial messaging service that 126)BOLD – QIT stands for Border electronically
enables financial institutions worldwide to send and dominated quick response team interception
receive information about financial transactions in technique, recently inaugurated across India –
secure, standardized and reliable environment. Bangladesh border along Dhubri is a
• It is used to transmit messages relating to cross A) Surveillance system
border financial transactions.
B) Anti Drug and Weapons Smuggling Programme
Functions:
C) The movement against large influxes of illegal
• SWIFT does not facilitate funds transfer: rather, it Bangladeshi Immigrants into India
sends payment orders, which must be settled by
D) “Look East Policy” of India over Bangladesh
correspondent accounts that the institutions have
with each other. Key : A
• The SWIFT is a secure financial message carrier — in Explanation :
other words, it transports messages from one bank to • Union Home Minister Rajnath Singh inaugurated the
its intended bank recipient. BOLD-QIT Surveillance along Dhubri district in Assam
on the India Bangladesh border.

Page No.45
MARCH CURRENT AFFAIRS_2019
BOLD-QIT Surveillance • During the past couple of years, the Jammu sector has
• BOLD-QIT stands for Border electronically dominated witnessed quite a few instances of successful
quick response team interception technique. infiltration by terrorists as a prelude to attacks on
• BOLD-QIT Surveillance is part of the Integrated Border strategic installations — prominent among these
Management System (CIBMS). being the Pathankot and Uri terrorist attacks in 2016.
• BOLD-QIT together with help the Border Security Force • These incidents have not only raised serious concerns
(BSF) in curbing all types of cross border crimes it about the efficacy of the existing border security
will also provide respite to the troops from round the system but also a consequent demand for the
clock human surveillance. deployment of high-tech border surveillance
equipment by the BSF.
• BOLD-QIT project involves the data network generated
by microwave communication along with other What is CIBMS?
paraphernalia, day and night surveillance cameras, • The CIBMS is a more robust and integrated system
and intrusion detection system. that is capable of addressing the gaps in the present
• These modern gadgets under BOLD-QIT provide feeds system of border security by seamlessly integrating
to the BSF control rooms on the border and enable human resources, weapons, and high-tech
BSF Quick Reaction Teams to thwart any possibility of surveillance equipment.
illegal cross border crossing and crimes. • At the moment, BSF is working on a Comprehensive
Why the Dhubri Sector along the India- Integrated Border Management System (CIBMS) which
will be deployed along the International Border with
Bangladesh border is critical?
Pakistan.
• It is in the region of the Dhubri Sector the river • CIBMS is expected to counter infiltration and cross-
Brahmaputra enters into Bangladesh. This region border terror attacks.
comprises of vast char lands and innumerable river
channels. This geography of the region makes the • The system employs latest technology which would
border guarding in the area a daunting task, especially detect infiltration via land, underwater, air and
during the rainy season. tunnels.
• Hence the technological intervention along with the • The concept of CIBMS is the integration of manpower,
physical presence of BSF was envisaged to thwart any sensors and command and control to improve
possibility of illegal cross border crossing and crimes. situational awareness and facilitate quick response
BOLD-QIT Surveillance makes the border management to emerging situations.
system more robust. Objective :
Objective : To know about the CIBMS (Comprehensive Integrated
To know about the different surveillance system across Border Management System).
the borders between India and its border countries.
127)Hand Held Thermal Imagery (HHTI) systems, Long Polity
Range Reconnaissance Observation Systems (LORROS) Miscellaneous
are related to: 128)Which of the following is/are correct provisions
A) Multiple technologies as surveillance system under Article 370 of Indian constitution which grants
across the border between India and Pakistan under special status to Jammu & Kashmir?
Comprehensive Integrated Border Management System 1.Indian citizens from other states cannot purchase
(CIBMS) land or property in Jammu & Kashmir
B) Large Telescopes to view the Galaxies of the Universe 2.The Centre has power to declare financial emergency
C) Technologies developed by ISRO to study the corona under Article 360 in the state
layer of the Sun 3.Indian Parliament cannot increase or reduce the
D) None of the above borders of the state.
Key : A 4.The jurisdiction of the parliament of India in relation
Explanation : to Jammu & Kashmir is confined to the matters
• As cross-border threats increased and the BSF enumerated in the union list, and also the concurrent
embarked on a modernisation process, the list
organisation acquired more sophisticated devices Select the correct answer using the code given below
such as Hand Held Thermal Imagery (HHTI) systems, A) 1, 2 and 3 only B) 2, 3 and 4 only
Long Range Reconnaissance Observation Systems C) 1, 3 and 4 only D) 1, 2, 3 and 4
(LORROS), Battle Field Surveillance Radars (BFSR), etc. Key : C
• This equipment proved to be game changers and force Explanation :
multipliers by enhancing the detection capabilities
What is Article 370?
of BSF personnel.
Need for CIBMS

Page No.46
MARCH CURRENT AFFAIRS_2019
• Article 370 of the Indian Constitution is a ‘temporary Judiciary
provision’ which grants special autonomous status 129)5. Consider the following statements with regards to
to Jammu & Kashmir. National Company Law Tribunal (NCLT):
• Under Part XXI of the Constitution of India, which deals 1.NCLT is a proposed quasi-judicial body in India that
with “Temporary, Transitional and Special provisions”, will govern the companies in India.
the state of Jammu & Kashmir has been accorded 2.It is a successor body of the Company Law Board
special status under Article 370.
3.The principle bench of NCLT is expected to be
• All the provisions of the Constitution which are established at Chennai
applicable to other states are not applicable to J&K.
Select the correct answer using the code given below
Important provisions under the article:
A) 1 and 2 only B) 2 and 3 only
• According to this article, except for defence, foreign
C) 1 and 3 only D) 1, 2 and 3
affairs, finance and communications, Parliament
needs the state government’s concurrence for applying Key : A
all other laws. Thus the state’s residents live under a Explanation :
separate set of laws, including those related to • National Company Law Tribunal (NCLT) is a proposed
citizenship, ownership of property, and fundamental quasi-judicial body in India that will govern the
rights, as compared to other Indians. companies in India. It will be established under the
• Indian citizens from other states cannot purchase Companies Act, 2013 and is a successor body of the
land or property in Jammu & Kashmir. Company Law Board. The principal bench of NCLT is
• Under Article 370, the Centre has no power to declare expected to be established in New Delhi. NCLT will
financial emergency under Article 360 in the state. It have the same powers as assigned to the erstwhile
can declare emergency in the state only in case of war Company Law Board (which are mostly related to
or external aggression. The Union government can dealing with oppression and mismanagement), Board
therefore not declare emergency on grounds of for Industrial and Financial Reconstruction
internal disturbance or imminent danger unless it is (BIFR)(revival of sick companies) and powers related
made at the request or with the concurrence of the to winding up of companies (which was available only
state government. with the High Court’s).
• Under Article 370, the Indian Parliament cannot Objective :
increase or reduce the borders of the state. To know about the National Company Law Tribunal
• The Jurisdiction of the Parliament of India in relation (NCLT)
to Jammu and Kashmir is confined to the matters Environment_Ecology
enumerated in the Union List, and also the concurrent Biodiversity
list. There is no State list for the State of Jammu and 130)The term “Cool spots” is sometimes seen in the news,
Kashmir. is refers to
• At the same time, while in relation to the other States, A) The very low temperature cold desert patches
the residuary power of legislation belongs to
B) The scientific observation spots in Arctic and
Parliament, in the case of Jammu and Kashmir, the
Antarctic
residuary powers belong to the Legislature of the
State, except certain matters to which Parliament has C) The areas with high numbers of threatened species
exclusive powers such as preventing the activities still persist
relating to cession or secession, or disrupting the D) The regions of reduced surface temperature on the
sovereignty or integrity of India. sun
• The power to make laws related to preventive detention Key : C
in Jammu and Kashmir belong to the Legislature of J & Explanation :
K and not the Indian Parliament. Thus, no preventive • ‘Cool-spots’ are the world’s last refuges where high
detention law made in India extends to Jammu & numbers of threatened species still persist. Cool-spots
Kashmir. could be the result of protection or because of intact
• Part IV (Directive Principles of the State Policy) and habitat that has not been cleared yet.
Part IVA (Fundamental Duties) of the Constitution are • Sunspots are temporary phenomena on the Sun’s
not applicable to J&K. photosphere that appear as spots darker than the
Objective : surrounding areas. They are regions of reduced
To know about the provisions of Articles 370 of the surface temperature caused by concentrations of
Indian Constitution to grants special powers to Jammu magnetic field flux that inhibit convection. Sunspots
& Kashmir. usually appear in pairs of opposite magnetic polarity.
Hence Option D is wrong.
Objective :
To know about the concept of ‘Cool-spots.

Page No.47
MARCH CURRENT AFFAIRS_2019
Geography Objective :
World Geography To know about the global actions taken towards
131)Which one of following regions does the Bushmen disaster risk reduction
tribe, belong to? 2019-03-13
A) Kalahari desert B) Andaman and Nicobar Economy
C) Arabian Desert D) Sierra Niveda Growth and Development
Key : A 133)Consider the following statements:
Explanation : 1.GNP = GDP + Net Factor Income from Abroad
• The Bushmen (also known as Khwe, Basarwa, or San) 2.Net National Product at factor cost is called
peoples of South Africa and neighboring Botswana “National Income”
and Namibia, who live in the Kalahari Desert. They 3.National Disposable Income = Net National Product
have a remarkably complex language characterized at market price + other current transfers from the rest
by the use of click sounds. of the world
Objective : Which of the statements given above are correct?
The student has to know about some important A) 1 and 2 only B) 2 and 3 only
indigenous tribes.Sentinellis are the indigenous tribe C) 1 and 3 only D) 1, 2 and 3
of Andaman islands
Key : D
Explanation :
Disaster management
• GNP = GDP + Net factor income from abroad.
132)Yokohoma declaration, Hyogo Framework and Sendai
framework are related to which of the following? • Net National Product at factor cost is called real
National Income as the NNP at factor cost expression
A) Women empowerment B) Trafficking contains all the terms related to National Income as
C) Disasters D) Maritime shown below
Key : C • NNP at factor cost = GDP at market Price + Net Factor
Explanation : Income from abroad – depreciation – Net Indirect
• The first world conference on Disaster Risk Reduction Taxes + Subsidies.
(DRR) held at Yokohama. It is called a Yokohoma • National disposable income = National income + Net
declaration. It recognized 1990-2000 as the Decade indirect taxes + Net current transfers from rest of the
of International Disaster Risk Reduction. The second world simply put.
world conference on Disaster Risk Reduction (DRR) • Net Disposable Income Is the Income which is at the
held at Hyogo resulted in Hyogo framework Action disposal of the nation as a whole for spending or
plan (HFAP) which set three strategic goals: disposal. Current transfers are gifts in cash and kind
1. The more effective integration of disaster risk (like consumer goods and even military equipment).
considerations into sustainable development • The main idea behind NDI is to know what is the
policies, planning and programming at all levels, with amount of goods and services the domestic economy
special emphasis on disaster prevention, mitigation, has at its disposal.
preparedness and vulnerability reduction. Objective :
2. The development and strengthening of Institutions, To know about the National Income, methods to
mechanisms and capacities at all levels, in particular calculate National Income and different terms related
at the community level, that can systematically to National Income.
contribute to building resilience to hazards.
134)Which ministry is responsible for calculating GDP
3. The systematic incorporation of risk reduction in India?
approaches into the design and implementation of
A) Ministry of Finance
emergency preparedness, response and recovery
programmes in the reconstruction of affected B) Ministry of Commerce and Industry
communities. C) Ministry of statistics and Program Implementation
• The Sendai framework for Disaster Risk reduction D) Ministry of Consumer affairs
2015-2030 is a non- binding agreement, which the Key : C
signatory nations, including India, will attempt to Explanation :
comply with on a voluntary basis. The four priorities
• The Central Statistics Office (CSO) under the Ministry
for action under the Sendai Framework are:
of Statistics and Programme calculates the GDP of
Understanding Disaster Risk, Strengthening disaster
India. Central statistics Office is the body that
risk governance to manage disaster risk, investing in
calculates the GDP of the country. It’s a body under
disaster risk reduction for resilience and enhancing
the Ministry of Statistics and program
disaster preparedness for effective response
implementation.

Page No.48
MARCH CURRENT AFFAIRS_2019
Objective : 2.Time Deposits : Fixed Deposit
To know about the agencies or ministries to calculate 3.Legal tenders : Cheques
GDP and other National Income related concepts. 4.Fiat Money : Currency notes
135)Which of the following is/are correct with regards Which of the above pairs are correctly matched?
to GDP calculation in India? A) 1, 2 and 3 only B) 2 and 3 only
1.While calculating GDP, income generated by C) 3 and 4 only D) 1, 2 and 4 only
foreigners in a country is taken into consideration
Key : D
2.While calculating GDP, income generated by
Explanation :
nationals of a country outside the country is taken
into account • Demand Deposits: The sum of money that is given to a
Select the correct answer using the code given below bank but can be withdrawn as per the requirement of
the depositor. Amounts that are lying in the savings
A) 1 only B) 2 only and current accounts are known as demand deposits
C) Both 1 and 2 D) Neither 1 nor 2 because they can be used at any point of time.
Key : A • Time deposits: A time deposit is an interest-bearing
Explanation : bank deposit account that has a specified date of
• While calculating GDP, Income generated by Nationals maturity, such as a certificate of deposit (CD). The
of a country outside the country is not taken into funds in these accounts must be held for a fixed term
account. According to definition GDP is the sum of and include the understanding that the depositor can
total market value of all final goods and services, make a withdrawal only by giving
produced within the domestic territory or a notice.
geographical area counted without duplication. • Legal Tenders: Denomination of a country’s currency
• Hence there is no foreign territory income is indulged that, by law, must be accepted as a medium for
to calculate the GDP in India. commercial exchange and payment for a money debt.
Objective : While usually all denominations of the circulating
paper money are legal tenders, the denomination and
To know about the calculation of GDP in India.
amount in coins acceptable as legal tender varies
136)Select the incorrect from the following with regards from country to country. Checks and postal orders
to real and Nominal GDP are not legal tenders and are accepted only at the
A) Real GDP refers to the current year production of option of the creditor, lender, or seller. Also called
goods and services valued at base year prices. Such lawful money cheques are not a promise to pay by the
base year prices are constant prices bank, but a request to the bank that it pays, out of the
B) Nominal GDP refers to current year production of funds deposited by the customer, an amount to a third
goods and services values at fixed prices. party.
C) Real GDP is a better measure to calculate GDP • Fiat Money or currency: Fiat currency is currency that
D) None of the above a government has declared to be legal tender. It is
different from cryptocurrency or virtual currency like
Key : B
Bitcoin which is not legal tender and not backed by
Explanation : government.
• Real GDP: Refers to the current year production of Objective : To know about the different deposits within
goods and services valued at base year prices. Such the bank in the concept called money and banking.
base year prices are Constant Prices.
Agriculture
• Nominal GDP: Refers to current year production of
138)If the contribution of the agricultural sector is
final goods and services valued at current year prices.
decreasing in a country’s economy, then what
Which one is a better measure? conclusion can be drawn?
• Real GDP is a better measure to calculate the GDP A) The country is growing in the direction of being a
because in a particular year GDP may be inflated developed nation
because of high rate of inflation in the economy.
B) The country is moving towards becoming developing
• Real GDP therefore allows us to determine if nation
production increased or decreased, regardless of
C) The country is moving towards becoming less
changes in the inflation and purchasing power of the
developed nation
currency.
D) The economic growth rate of the country has stopped
Objective : To know about the Real and Nominal GDP.
Key : A
Explanation :
Monetary Policy
• The country is growing in the direction of being a
137)Consider the following pairs with respect to India:
developed nation. Contribution of Agriculture in the
1.Demand Deposits : Saving deposit & Current Account GDP of developed nations decreases and the
Deposits

Page No.49
MARCH CURRENT AFFAIRS_2019
contribution of service and manufacturing sector is • The Totagars’ Cooperative Sale Society (TSS) Ltd, a Sirsi-
very high. based agri cooperative, is the registered proprietor
Objective : of the GI ‘Sirsi Supari’.
To know about the Division of economic sectors or • ‘Sirsi Supari’ is medium sized and round in shape, it
activities and its contribution to the country’s growth. has a somewhat ash coloured hard seed.
139)Which of the following describes the term called • ‘Sirsi Supari’ is unique in taste from areca nuts grown
direct farm credit? in other parts of the country due to the differences in
1. When the borrower is directly responsible for its the chemical composition of different areca nuts.
repayment to the lending agency. The correct order is:
2. FCI and warehouses agencies come under direct Alphonso mango - Maharashtra
creditor to agriculture. Shahi Litchi - Bihar
Select the correct answer using the code given below Kandhamal Haldi - Odisha
A) 1 only B) 2 only Sirsi Supari - Karnataka
C) Both 1 and 2 D) Neither 1 nor 2 Objective :
Key : A To know about recent Geographical Indications
Explanation :
• Any loan to agriculture and allied sector is called External Sector
farm credit. When the borrower is directly responsible 141)The Stockholm International Peace Research Institute
for its repayment to the lending agency, it is direct (SIPRI) has released the “Trends in International Arms
farm credit. It includes short, medium and long term Transfers -2018” Report. In this context, consider the
loans given for agriculture and allied activities (dairy, following statements with respect to trends of India
fishery, piggery, poultry, bee-keeping, etc.) directly to during 2014-18:
individual farmers without limit for taking up 1. Russia accounted for more than half of the total
agriculture / allied activities except nominal members Indian arms imports in 2014-2018.
or to agencies like PACS, primary land development
2. India’s import of arms increased by 24 per cent
banks etc.
between 2009-2013 and 2014-2018.
• Indirect credit refers to, funds in agriculture given
3. India was the world’s largest importer of major
indirectly through some intermediary agency/
arms in 2014-18.
institutions etc. which will be responsible for
repayment. So funds availed by fertilizer dealers, state Which of the statement(s) given above is/are
corporations, FCI, warehouses will come under incorrect?
indirect creditor to agriculture. A) 1 only B) 1 and 2 only
Objective : C) 2 and 3 only D) 1 and 3 only
To know about the Direct and Indirect farm credit, Key : C
and differences. Explanation :
• India was the world’s second largest arms importer
Report and Indices from 2014-18, ceding the long held tag as largest
140)Which of the following pair(s) is /are not correctly importer to Saudi Arabia, which accounted for 12% of
matched with respect to the recent Geographical the total imports during the period.
Indications? • Russia accounted for 58% of Indian arms imports in
1. Alphonso mango - Maharashtra 2014–18, compared with 76% in 2009-13.
2. Shahi Litchi - Bihar • However, Indian imports decreased by 24% between
2009-13 and 2014-18, partly due to delays in
3. Kandhamal Haldi - Karnataka
deliveries of arms produced under licence from foreign
4. Sirsi Supari - Odisha suppliers, such as combat aircraft ordered from
Select the correct answer using the code given below: Russia in 2001 and submarines ordered from France
A) 1 and 2 only B) 2 and 3 only in 2008.
C) 3 and 4 only Objective :
D) All of the above are correct To know about recent global arms trends and trends
Key : C in India.
Explanation : 142)Which of the following is/are correct with regards
• Recently registrar of Geographical Indications to External Commercial Borrowings (ECB)?
accorded a GI tag, ‘Sirsi Supari’. Sirsi Supari is an 1.An ECB is an instrument used in India to facilitate
arecanut grown in Sirsi, Siddpaur and Yellapur taluks the access to foreign money by Indian corporations
of Uttara Kannada district in Karnataka. and PSUs (Public Sector Undertakings).

Page No.50
MARCH CURRENT AFFAIRS_2019
2.ECBs cannot be used for investment in Stock Market treasury bills, state development loans (SDLs) and
or speculation in real estate. corporate bonds, but with certain restrictions and
Select the correct answer using the code given below limits. FPI is part of country’s capital account and
A) 1 only B) 2 only shown on its balance of payments (BOP).
C) Both 1 and 2 D) Neither 1 nor 2 Differences between FPI and FDI
Key : C • FPI lets investor purchase stocks, bonds or other
financial assets in foreign country. In this case,
Explanation :
investor does not actively manage investments or
• An external commercial borrowing (ECB) is an companies that issue investment. It also does not have
instrument used in India to facilitate the access to control over securities or business. In contrast, FDI
foreign money by Indian corporations and PSUs lets investor purchase direct business interest in
(public sector undertakings). ECBs include foreign country. The investor also controls his
commercial banks loans, buyers’ credit, suppliers’ monetary investments and actively manages company
credit, securitised instruments such as floating rate into which he puts money. FPI is more liquid and less
notes and fixed rate bonds etc., credit from official risky than FDI.
export credit agencies and commercial borrowings
Objective :
from the private sector window of multilateral
financial institutions such as International Finance To know about the difference between FDI and FPI.
Corporation (Washington), ADB, AFIC, CDC, etc. ECBs
cannot be used for investment in Stock Market or Science & Technology
speculation in real estate. Defence
Objective : 144)Consider the following statements with respect to
To know about the External Commercial Borrowings “Pinaka Extended Range Rocket”:
(ECBs). 1. Pinaka is a multiple rocket launcher ingenuously
developed by DRDO.
Investment 2. It can fire a salvo of 12 rockets in 44 seconds.
143)In the context of differentiating the Foreign Direct 3. The rocket’s range was increased to 70-80 km with
Investment (FDI) and Foreign Portfolio Investment Pinaka Mark II.
(FPI), which of the following is/are correct? Which of the statement(s) given above is/are correct?
1.FDI aims to take control of the company in which A) 1 and 2 only B) 2 and 3 only
investment is made whereas FPI aims to reap profits C) 1 and 3 only D) 1, 2 and 3
by investing in shares and bonds of the invested entity
Key : D
without controlling the company.
Technique : Unclassified
2.FPI is liquid, volatile and risky whereas FDI is stable.
Explanation :
Select the correct answer using the code given below
• Pinaka is guided extended range rockets which can
A) 1 only B) 2 only
immobilize enemies from a 90Km distance, developed
C) Both 1 and 2 D) Neither 1 nor 2 indigenously by DRDO.
Key : C • Pinaka rocket launcher can fire a salvo of 12 rockets
Explanation : in 44 seconds.
• Foreign Direct Investment (FDI) and Foreign Portfolio • Pinaka Mark I was extensively used by Indian Army in
Investment (FPI) are the two important forms of foreign 1999 Kargil war to neutralize enemies on mountain
capital. The real difference between the two is that tops.
while FDI aims to take control of the company in which • The initial version of the Pinaka rocket had a range of
investment is made; FPI aims to reap profits by 40km before it was developed into Pinaka Mark II,
investing in shares and bonds of the invested entity which had an enhanced range of 70-80 km.
without controlling the company.
• Each Pinaka launcher has its own computer, allowing
• Both FDI and FPI are the most well sought type of it to work independently and able to fire rockets in
foreign capital by the developing world. Usually, both different directions, fire all rockets in one go or select
these are measured in terms of the percentage of the few.
shares they own in a company (ie., 10%, 20% etc.,).
Objective :
Foreign portfolio investment (FPI)
To know about Pinaka Extended Range Rocket.
• FPI consists of securities and other financial assets
passively held by foreign investors. It does not provide
investor with direct ownership of financial assets. It
is relatively liquid depending on volatility of the
market. In India, FPIs are allowed to invest in various
debt market instruments such as government bonds,

Page No.51
MARCH CURRENT AFFAIRS_2019
Governance including its disinvestment in central public sector
Government policies and undertakings. Finance Minister Arun Jaitley had
programs announced renaming of the Department of
Disinvestment in his budget speech for 2016-17.
145)National Knowledge Network (NKN) is a multi- Initially set up as an independent ministry (The
gigabyte pan-India network which facilitates the Ministry of Disinvestment) in December 1999, the
development of India’s communications Department of Disinvestments came into existence in
infrastructure. Recently India has decided to extend May 2004 when the ministry was turned into a
its NKN to which of the following country? department of the Ministry of Finance. The department
A) Bangladesh B) Bhutan took up all the functions of the erstwhile ministry
C) Nepal D) Myanmar which broadly was responsible for systematic policy
Key : A approach to disinvestment and privatisation of Public
Explanation : Sector Units (PSUs).
• India has decided to extend its National Knowledge Objective :
Network to Bangladesh. • To know about the DIPAM (Department of Investment
• NKN is a multi-gigabit pan-India network which and Public Asset Management)
facilitates the development of India’s communications
infrastructure, stimulates research and creates next 147)Which of the following is /are correct with regards
generation applications and services. to “Unnat Jyoti by Affordable LEDs for All (UJALA)”?
• With its multi-gigabit capability, NKN aims to connect 1.UJALA scheme replaced the old scheme called
all universities, research institutions, libraries, “Bachat Lamp Yozana”.
laboratories, healthcare and agricultural institutions 2.The main objective of this scheme is to reduce the
across the country to address such paradigm shift. energy consumption with the help of energy efficient
• It enables collaboration among researchers from bulbs like LEDs.
different educational networks such as TEIN4, 3.UJALA scheme is a subsidized scheme
GARUDA, CERN and Internet2. It also enables sharing 4.The Electricity Distribution Company and Energy
of scientific databases and remote access to advanced Efficiency Services Limited (EESL) are implementing
research facilities. The leading mission oriented the programme.
agencies in the fields of nuclear, space and defence
research are also part of NKN. Select the correct answer using the code given below
Objective : A) 1, 2 and 3 only B) 1, 2 and 4 only
To know about National Knowledge Network C) 2, 3 and 4 only D) 1, 2, 3 and 4
Key : B
146)Which of the following is the nodal agency of Union Explanation :
Finance Ministry mandated to advise the Union • Recently, UJALA scheme was launched by the
Government in the matters of financial restructuring Government of India. “Unnat Jyoti by Affordable LEDs
of PSUs and also for attracting investment through for ALL “is the wordplay for UJALA. Earlier, this scheme
capital market? was announced as “Domestic Efficient Lighting
A) Department of Economic Affairs Programme (DELP)”.
B) Department of Financial services • UJALA scheme replaced the old “Bachat Lamp Yojana”
C) Department of Expenditure • Unnat Jyoti by Affordable Lighting for All (UJALA) is a
Zero subsidy Scheme launched by the Government of
D) Department of Investment and Public Asset
India in an effort to popularize the message of energy
Management
efficiency in the country. The scheme is being
Key : D implemented by a Public Sector Undertaking of the
Explanation : Government of India, Energy Efficiency Services
• DIPAM is the nodal agency of Union Finance Ministry Limited (EESL) under the Union Ministry of Power and
mandated to advise the Union Government in the the Electricity Distribution Company. It is a LED based
matters of financial restructuring of PSUs and also Domestic Efficient Lighting Programme (DELP) targeting
for attracting investment through capital markets. It promotion of reduced energy consumption, energy
will also deal with all matters relating to sale of Union savings and efficient lighting. Under the scheme,
Government’s equity in PSUs through private Electricity Distribution Company would distribute LED
placement or offer for sale or any other mode in the bulbs at subsidized rates to every grid-connected
erstwhile Central PSUs. customer with a metered connection
• The Department of Disinvestment has been renamed Why LED bulbs?
as Department of Investment and Public Asset • LED bulbs have a lifetime of 50 times more than that
Management or ‘DIPAM’, a decision aimed at proper of ordinary bulbs and consume just about 1/10th of
management of Centre’s investments in equity the energy consumed by an ordinary bulb for a similar

Page No.52
MARCH CURRENT AFFAIRS_2019
light output. The replacement of ordinary bulbs with Which of the statements given above is/are correct?
LED bulbs would result in load reduction, reduced A) 1 only B) 2 only
consumer bills, reduction of Greenhouse Gass C) Both 1 and 2 D) Neither 1 nor 2
Emissions and electricity savings. The government has
set a target of 200 million light bulbs to be replaced Key : A
by LED bulbs which is expected to result in a load Explanation :
reduction of 5000 MW and reduce Greenhouse Gas • Sec.89 in the Code of Civil Procedure allows courts
emissions by 79 million tonnes of CO2. UJALA is an for non adjudicatory resolution processes.
effort to contribute to energy security in India. • Procedures of mediation are not controlled by Indian
• UJALA is called a Zero subsidy scheme as the state evidence act. It allows freedom of flexibility to the
governments do not bear the burden of subsidies. The parties in mediation.
total risk coverage and upfront investment is borne Objective :
by EESL and is paid by DISCOM from actual energy To know about the procedure of Mediation which was
savings over 5 years. Therefore, there is no need for recently in news in Ayodhya dispute
the grant of subsidy by the government, and at the
2019-03-12
same time, there is no impact on the electricity bills.
Economy
Objective :
Monetary Policy
To know about the UJALA Scheme.
150)NBFCs
148)Select the correct from the following statements with
regards to Setu Bharatam Project: A) IL&FS B) RBI
A) Setu Bharatam Project aims to construct bridge C) Government of India D) SEBI
across Rama Setu between India and Sri Lanka Key : A
B) Setu Bharatam Project aims to construct highways Explanation :
to gulf of manner from the interiors of India. • The recent debt default by Infrastructure Leasing and
C) Setu Bharatam Project aims to make all National Financial Services (IL&FS), a conglomerate engaged
highways free of railway level crossings. in financing and developing infrastructure projects,
D) Setu Bharatam Project aims to build National has demonstrated the financial risks posed by India’s
Highways along golden quadrilateral network. non-banking financial companies (NBFCs).
Key : C • The crisis has already created a credit crunch,
affecting sectors such as real estate that have been
Explanation :
dependent on NBFC funding for growth. A 17 October
• Ministry/Department: Ministry of Road, Transport and report by Credit Suisse suggests that the exposure of
Highway NBFCs to real-estate developers could be as high as ?
• Objectives: Development of bridges for safe and 2 trillion. The real estate stock index has declined
seamless travel on National Highways 26% since the IL&FS crisis erupted even as the
• Project: benchmark index declined by 12% over the same
• Aims to make all National Highways free of railway period.
level crossings by 2019 • The importance of NBFCs in disbursing credit to the
• This is being done to prevent the frequent accidents overall commercial sector has risen in recent years,
and loss of lives at level crossings as banks have struggled to increase lending amid the
overhang of non-performing assets (NPAs). Thus, the
• The Ministry of Road Transport & Highways has also
share of NBFCs in total credit extended has increased
established an Indian Bridge Management System
from around 9.4% in March 2009 to more than 17%
(IBMS) at the Indian Academy for Highway Engineer in
by March 2018, data from the Reserve Bank of India’s
Noida, U.P.
(RBI’s) latest financial stability report suggests.
• The aim is to carry out conditions survey and
inventorization of all bridges on National Highways Objective :
in India by using Mobile Inspection Units To know about the Shadow banking system, IL&FS and
Objective : Non Banking Financial Companies.
To know about the Setu Bharatam Project
151)“NBFCs (Non Banking Financial Companies) are often
called as Shadow banking system; the financial risks
Polity posed by NBFCs are demonstrated by IL&FS” in this
Judiciary context, which of the following is/are correct which
149)Consider the following statements lead (s) to loss for IL&FS?
1. Section 89 of Code of Civil Procedure (CPC) allows 1.Interest rate mismatch between borrowing and
courts for non adjudicatory resolution processes. lending by IL&FS
2. Procedures of mediation are controlled by Indian
evidence act- 1872

Page No.53
MARCH CURRENT AFFAIRS_2019
2.Stalled projects on which IL&FS was investing for • With effect from January 1 2018, small merchants
longer period will pay a maximum MDR of 0.40 per cent of the bill
Select the correct answer using the code given below value and others will shell out 0.90 per cent. To
A) 1 only B) 2 only prevent those MDR charges from sky-rocketing, RBI
has also set a monetary cap at Rs.200 per bill for
C) Both 1 and 2 D) Neither 1 nor 2 small merchants and ?1,000 for large ones.
Key : C • As per RBI rules, the merchant must cough up the MDR
Explanation : out of his own pocket and cannot pass it on to the
Asset – Liability mismatch issue: customer.
Interest rate mismatch: Why is it important?
• Usually the NBFCs like IL&FS, they borrow from the • To ensure wider adoption of plastic, banks must have
market for shorter period of time say for example 3 to more cards/PoS machines in circulation and more
5 years but they are lending for longer period like 10 merchants need to install PoS terminals. Getting small
years absolutely because they are lending for merchants to install PoS machines has been a
infrastructure projects means it will be 10 years. challenge, as cash transactions entail no extra costs
• Suppose in the year 2010 the IL&FS borrowed for 3 to to them, while cards do. Banks on their part are willing
5 years at 10% interest rate and it lends for 10 years to increase PoS coverage only if their MDR share is
at 12% of interest, here the interest rates are lucrative.
increasing year by year, if interest rates are increasing • The RBI’s latest move seems to be an attempt to resolve
then there is mismatch between borrowing and lending this tug-of-war. It allows banks to make a higher MDR
interest rates, leads to loss for IL&FS. fee off large merchants, while allowing the smaller
Increasing number of stalled projects: fry to pay nominal fees. To calculate MDR, small
• As IL&FS involved in Infrastructure Projects, merchants are defined as those with a turnover of
infrastructure projects are majorly considered as upto ?20 lakh in the previous year. They will pay an
stalled projects as projects getting delayed due to MDR of 0.4 per cent against 0.9 per cent for others.
land pooling, rising of funds etc. as investment on Objective :
infrastructure is a primary object of IL&FS, being To know about the Merchant Discount Rate.
increasing number of stalled projects it leads to loss
for IL&FS.
Objective :
Growth and Development
To know about the Shadow Banking System in India.
153)Consider the following statements with respect to
152)Which one of the following best describes the term Gross Value Added (GVA):
“Merchant Discount Rate” Sometimes seen in news?
1. Gross Value Added is the measure of the value of goods
A) The incentive given by a bank to a merchant for and services produced in an area, industry or sector
accepting payments through debit cards pertaining of an economy.
to that bank
2. Gross Value Added is equal to Gross Domestic Product
B) The amount paid back by banks to their customers (GDP).
when they use debit cards for financial transactions
Which of the above statement (s) is/are correct using
for purchasing goods or services.
the code given below
C) The charge on a merchant by a bank for accepting A) 1 only B) 2 only
payments from his customers through the bank’s debit
cards. C) Both 1 and 2 D) Neither 1 nor 2
D) The incentive given by the government to merchants Key : A
for promoting digital payments by their customers Explanation :
though point of Sale (PoS) machines and debit cards. Gross value added (GVA):
Key : C • Gross value added (GVA) is the measure of the value
Explanation : of goods and services produced in an area, industry
• MDR is the fee that the store accepting your card has or sector of an economy.
to pay to the bank when you swipe it for payments. • GVA is linked as a measurement to gross domestic
The MDR compensates the bank issuing the card, the product (gdp), as both are measures of output. The
bank which puts up the swiping machine (Point-of- relationship is defined as:
Sale or PoS terminal) and network providers such as GVA + taxes on products – subsidies on products =
Mastercard or Visa for their services. MDR charges GDP
are usually shared in a pre-agreed proportion Objective :
between them. In India, the RBI specifies the maximum To know about the Gross Value Added (GVA) and
MDR charges that can be levied on every card comparison between GVA and GDP.
transaction.

Page No.54
MARCH CURRENT AFFAIRS_2019
Polity 1. Nari Shakti Puruskar is the highest civilian honour
Constitutional bodies (Origin, that is awarded exclusively to women.
structure and functions) 2. It is awarded by the Ministry of Women and Child
154)Consider the following statements with respect to Development
Model Code of Conduct (MCC) Which of the statement(s) given above is/are correct?
1. It refers to a set of norms laid down by the Election A) 1 only B) 2 only
Commission of India, with the consensus of political C) Both 1 and 2 D) Neither 1 nor 2
parties. Key : C
2. It comes into force before 48 hours of the polling Explanation :
and remains operational till the process is concluded, • Nari Shakti Puruskar is the highest civilian honour
as provided in the notification. that is awarded exclusively to women. It is awarded
3. It is also applicable to a caretaker government on by the Ministry of Women and Child Development.
premature dissolution. • The awards is being conferred on 8th March on the
Which of the statement(s) given above is/are correct? occasion of International Women’s Day (IWD).
A) 1 and 2 only B) 2 and 3 only • The award would be conferred on eminent women and
C) 1 and 3 only D) 1, 2 and 3 institutions rendering distinguished service to the
Key : C cause of women especially belonging to the vulnerable
Explanation : and marginalized sections of the society. Award in
each category shall carry a certificate and a cash
• The model code refers to a set of norms laid down by
amount.
the Election Commission of India, with the consensus
of political parties. It is not statutory. It spells out the Objective :
dos and don’ts for elections. Political parties, To know about various initiatives of Government to
candidates and polling agents are expected to observe empower women.
the norms, on matters ranging from the content of
election manifestos, speeches and processions, to Science & Technology
general conduct, so that free and fair elections take Defence
place.
156)In the backdrop of recent Pulwama attack, “Combat
• The code comes into force on the announcement of Causality Drugs” which extend the golden hour during
the poll schedule and remains operational till the warfare and terror attacks were recently developed
process is concluded, as provided in the notification. by:
It is also applicable to a “caretaker” government on
A) Council of Scientific & Industrial Research (CSIR)
premature dissolution of a State Assembly, as was
the case in Telangana. B) Defence Research and Development Organisation
(DRDO)
• The EC ensures that ruling parties at the Centre and in
States adhere to the code, as part of its mandate to C) Institute for Defence Studies and Analysis (IDSA)
conduct free and fair elections under Article 324 of D) The Indian Council of Medical Research (ICMR)
the Constitution. In case of electoral offences, Key : B
malpractices and corrupt practices like inducements Explanation :
to voters, bribery, intimidation or any undue influence,
• DRDO develops ‘combat causality drugs’ to reduce
the EC takes action against violators. Anyone can
casualties in Pulwama type attacks and during
report the violations to the EC or approach the court.
warfare. The combat causality drugs can extend the
The EC has devised several mechanisms to take note
golden hour till the trooper is shifted to hospital.
of the offences, which include joint task forces of
enforcement agencies and flying squads. The latest is • These medicines will ensure soldiers do not suffer
the introduction of the cVIGIL mobile app through from unwanted blood loss while being taken to a better
which audio-visual evidence of malpractices can be healthcare from war zones.
reported. • The drugs include bleeding wound sealants, super
Objective : absorptive dressings and glycerinated saline, all of
which can save lives in the event of warfare in the
To know about the Model Code of Conduct.
jungle and high altitude areas as well as in terror
attacks.
Indian Society Objective :
Role of women and women?s To know about the recent development in combat
organization drugs.
155)Consider the following statements with respect to
“Nari Shakti Puruskar”:

Page No.55
MARCH CURRENT AFFAIRS_2019
Basic Science A) 1 only B) 1 and 2 only
157)Which of the following are vector borne diseases C) 2 and 3 only D) 1 and 3 only
(VBD)? Key : D
1.Malaria 2.Filaria Explanation :
3.Kala – azar • The Ministry of Power has announced that two more
4.Japanese Encephalitis electrical appliances microwave ovens and washing
Select the correct answer using the code given below machines will now be assigned star ratings based on
A) 1 and 2 only B) 1, 3 and 4 only their energy efficiency metrics.
C) 1 and 3 only D) 1, 2, 3 and 4 • The Standards & Labelling (Star Rating) Program has
been formulated by Bureau of Energy Efficiency (BEE).
Key : D The BEE is a statutory body under the Ministry of
Explanation : Power, Government of India.
Vector: • Under the Scheme, there are both mandatory and
• Vectors are living organisms that can transmit voluntary forms of labelling.
infectious diseases between humans or from animals Objective :
to humans. Many of these vectors are bloodsucking To know about Bureau of Energy Efficiency (BEE).
insects, which ingest disease-producing
microorganisms during a blood meal from an infected 159)Which of the following is/are correct with regards
host (human or animal) and later inject it into a new to National Health Policy, 2017?
host during their subsequent blood meal. 1.It’s primary objective is to strengthen the trust of
• Mosquitoes are the best known disease vector. Others the common man in public health care system.
include ticks, flies, sandflies, fleas, triatomine bugs 2.It aims to Increase utilization of public health
and some freshwater aquatic snails. facilities by 50% from current levels by 2025.
• Vector-borne diseases are human illnesses caused 3.It targets to provide access to safe water and
by parasites, viruses and bacteria that are sanitation to all by 2020
transmitted by mosquitoes, sandflies, triatomine bugs, Select the correct answer using the code given below
blackflies, ticks, tsetse flies, mites, snails and lice. A) 1 and 2 only B) 2 and 3 only
Every year there are more than 700 000 deaths from C) 1 and 3 only D) 1, 2 and 3
diseases such as malaria, dengue, schistosomiasis,
Key : D
human African trypanosomiasis, leishmaniasis,
Chagas disease, yellow fever, Japanese encephalitis Explanation :
and onchocerciasis, globally. Main Objectives of the Policy:
• The major vector-borne diseases, together, account • The primary objective of the National Health Policy,
for aeround 17% of all infectious diseases. The burden 2017, is to strengthen the trust of the common man in
of these diseases is highest in tropical and subtropical public health care system by making it patient centric,
areas and they disproportionately affect the poorest efficient, effective and affordable, with a
populations. Since 2014, major outbreaks of dengue, comprehensive package of services and products that
malaria, chikungunya yellow fever and Zika have meet immediate health care needs of most people.
afflicted populations, claimed lives and overwhelmed Key Targets of the National Health Policy, 2017,
health systems in many countries. are as follows:
Objective : 1. Increase health expenditure of Government from the
To know about the different vector borne diseases and existing 1.15% to 2.5% of the GDP by 2025.
vectors. 2. Increase Life Expectancy at birth from 67.5 to 70 by
2025.
Governance 3. Reduction of Total Fertility Rate (TFR) to 2.1 at national
Government policies and and sub-national level by 2025. In FY 2016, India had
programs TFR at 2.3 births per woman.
158)Consider the following statements with respect to 4. Reduce under Five Mortality to 23 by 2025 and Maternal
Bureau of Energy Efficiency (BEE) Mortality Ratio (MMR) from currents level’s 167 to
1. Its primary objective is to reduce the energy intensity 100 by 2020. It is worth to mention that Under Five
of the Indian economy. Mortality in India was 29 (per 1000 live births in
2015.
2. It is a non-statutory body under the Ministry of
5. Reduction of 40% in prevalence of stunting of Under-
Power.
Five Children by 2025.
3. Under the Scheme, there are both mandatory and
voluntary forms of labeling. 6. Reduce Infant Mortality Rate (IMR) to 28 by 2019.In
2016 the IMR was 34 per 1000 live births.
Which of the statement(s) given above is/are correct?

Page No.56
MARCH CURRENT AFFAIRS_2019
7. Reduce neo-natal mortality to 16 and birth rate to for the Dam Rehabilitation and Improvement Project
“single digit” by 2025. Neo-Natal Mortality (NMR) was (DRIP) that will help rehabilitate and modernize over
28 per 1000 live births in India in 2013. 220 selected large dams.
9. To reduce the prevalence of blindness to 0.25/ 1000 by • This additional funding of $137 million will be used
2025 and disease burden by one third from current for the construction of an additional spillway for
levels. Hirakud Dam in Odisha and in rehabilitation and
10. Achieve and maintain elimination status of Leprosy improvement of other dams including strengthening
by 2018, Kala-Azar by 2017 and Lymphatic Filariasis the institutional, legal and technical framework for
in endemic pockets by 2017. dam safety assurance within the Government of India
11. To reduce premature mortality from cardiovascular and in the participating States.
diseases, diabetes or chronic respiratory diseases • The Ministry of Water Resources (MoWR), Government
and cancer by 25% by 2025. of India, with assistance from the World Bank, is
12. To achieve and maintain a cure rate of more than 85% implementing the DAM REHABILITATION AND
in new sputum positive patients for TB and reduce IMPROVEMENT PROJECT (DRIP), which would be a six-
incidence of new cases, to reach elimination status year project.
by 2025. • The Central Dam Safety Organisation of Central Water
13. Increase utilization of public health facilities by 50% Commission, assisted by a Consulting firm, is
from current levels by 2025. coordinating and supervising the Project
implementation.
14. More than 90% of the newborn are fully immunized
by one year of age by 2025. Objectives of DRIP:
15. Ensure skilled attendance at birth above 90% by 2025. • Rehabilitation of old dams in country experiencing
distress and are in need of attention for ensuring their
16. Relative reduction in prevalence of current tobacco
structural safety and operational efficiency.
use by 15% by 2020 and 30% by 2025.
• Strengthening institutional capacity and project
17. Access to safe water and sanitation to all by 2020
management in this area.
(Swachh Bharat Mission).
• Bring greater awareness on dam safety issues and
18. Reduction of occupational injury by 50% from current finding novel solutions to address them by pooling
levels of 334/lac agricultural workers by 2020. best knowledge, technologies and experience
19. Increase the share of State on health to more than 8% available around world.
of their budget by 2020. Objective :
20. Decrease in the health expenditure of the households
To know about Dam Rehabilitation & Improvement
from the current level by 25%, by 2025.
Project (DRIP).
Objective :
2019-03-11
To know about the National Health Policy, 2017.
Economy
Fiscal Policy
International Relations
161)Interest on Public Debt is a part of:
International Institutions, A) Transfer payments by the enterprises
agencies and their structure
B) Transfer payments by the Government
and mandate
C) National Income
160)Dam Rehabilitation and Improvement Project (DRIP) D) Interest payment by the House Holds
aims to improve safety and operational performance
of dams. Which of the following statements is/ are Key : B
correct about DRIP? Explanation :
1. The projected is assisted by the newly created Asian • Government debt (also known as public interest, public
Infrastructure Investment Bank. debt, national debt and sovereign debt) is the debt
2. Central water commission is the nodal agency to owed by a government. By contrast, the annual
coordinate and supervise the project. “government deficit” refers to the difference between
government receipts and spending in a single year.
Select the correct answer using the code given below
• Public debt is always related to the government of
A) 1 only B) 2 only
India. Transfer payments to cover the public debt are
C) Both 1 and 2 D) Neither 1 nor 2 done by the Government itself not by any enterprises
Key : B from the above options. Hence answer b is correct.
Explanation : Objective :
• The World Bank, Government of India and To know about the Public debt under public finance
representatives from the states of Karnataka, Kerala, concept.
Odisha, Tamil Nadu and Uttarakhand signed Loan
Agreement for additional financing of $137 Million

Page No.57
MARCH CURRENT AFFAIRS_2019
162)Consider the following statements with regards to A) Economic Development
Goods and Services Tax Network (GSTN): B) Redemption of public debt
1.GSTN is non profit organization formed to provide C) Adjusting the Balance of Payments
IT infrastructure and services to the central and state D) Reducing the foreign debt
governments, tax payers and other stakeholders for
the implementation of GST. Key : A
Explanation :
2.The government of India holds 51% stake in GSTN
• Deficit financing is the budgetary situation where
Which of the statement (s) above is/are correct using
expenditure is higher than the revenue. It is a practice
the code given below?
adopted for financing the excess expenditure with
A) 1 only B) 2 only outside resources. The expenditure revenue gap is
C) Both 1 and 2 D) Neither 1 nor 2 financed by either printing of currency or through
Key : A borrowing.
Explanation : • Nowadays most governments both in the developed
• GSTN was set up as not for profit, non-Government, and developing world are having deficit budgets and
private limited company in 2013. It was established these deficits are often financed through borrowing.
primarily to provide IT infrastructure and services to Hence the fiscal deficit is the ideal indicator of deficit
Central and State Governments, tax payers and other financing.
stakeholders for implementation of Goods and • Deficit financing is very useful in developing countries
Services Tax (GST). like India because of revenue scarcity and
• Currently, Centre and states together hold 49% stake development expenditure needs.
(24.5% each) in GSTN. The remaining 51% is owned by • If government is under the deficit financing, it means
five private financial institutions- ICICI Bank, NSE, it is under the revenue scarcity and it has done more
HDFC Ltd, HDFC Bank and LIC Housing Finance Ltd. Its expenditure on developmental aspects. If expenditure
revenue model after GST was rollout out consisted of levels are increasing, it means each organizational
User Charge to be paid by stakeholders who will use group like House Holds, Government, Firms and
the system and making it self-sustaining organization. Financial institutions etc are spending more in terms
• The Union Cabinet has approved conversion of Goods of expenditures like Consumer expenditure,
and Services Tax Network (GSTN) into government- Government Expenditure, Investment Expenditure and
owned company. Government will now own 100% of giving credits respectively. Hence Expenditure levels
GSTN which is IT backbone to GST regime. Majority of and economic development always are proportional
Goods and Services Tax (GST) processes including hence Deficit financing leads to Economic
registration, filing of returns, payment of taxes, Development.
processing of refunds is IT driven and mainly through Objective :
GSTN. To know about the Deficit Financing, types and
Objective : consequences with that.
• To know about the Goods and Services Tax Network 165)Deficit financing was first used in the area of public
(GSTN) finance in the early 1930s in the USA. Consider the
163)Which of the following taxes is exclusively and following statements regarding the deficit financing:
totally assigned to the Central Government by the 1.The process of financing a deficit budget by a
Constitution? government is deficit financing.
A) Estate tax B) Sales tax 2.In the process of deficit financing, the government
C) Taxes on Railway fares and freights knows well in advance that its total expenditures are
D) Corporation tax going to turn out to be more than its total receipts
and enacts/follows such financial policies so that it
Key : D can sustain the burden of the deficits proposed by it.
Explanation :
Which of the above statement (s) is/are correct using
• Estate duty in respect of agricultural land is under the code given below?
the State Governments.
A) 1 only B) 2 only
• From the above options Corporation Tax is the only C) Both 1 and 2 D) Neither 1 nor 2
tax, which totally assigned to the central government
by the constitution. Key : C
Objective : Explanation :
To know about the assigning of taxes to the central • The act/process of financing/supporting a deficit
government by the constitution. budget by a government is deficit financing. In this
process, the government knows well in advance that
its total expenditures are going to turn out to be more
164)In India deficit financing is used for raising resources than its total receipts and enacts/follows such
for

Page No.58
MARCH CURRENT AFFAIRS_2019
financial policies so that it can sustain the burden of A) Amount of money a taxpayer is able to subtract
the deficits proposed by it. from taxes owed to Government
• First used in the area of public finance in the early B) Amount of money a taxpayer is charged for after a
1930s in the USA, today the term is being used by the threshold limit of income
corporate sector, too and such a financial C) Amount of money collected from a taxpayer for a
management of a firm might be followed by it as part specified purpose, as tax on tax
of its business strategy. Again, a sick firm might need D) Amount of money a taxpayer is able to get refunded
to follow deficit financing route for many years to for better tax compliance
come as required by the firm to make it come out of
the red (i.e., doing away with the losses). Key : A
Explanation :
• The need of Deficit Financing: It was in the late 1920s
that the idea and need of deficit financing were felt. It • A tax credit is a tax incentive which allows certain
is when the government needs to spend more money taxpayers to subtract the amount of the credit they
than it was expected to earn or generate in a particular have accrued from the total they owe the state. It may
period, to go for the desired level of growth and also be a credit granted in recognition of taxes already
development. Had there been some means to go for paid or, as in the United Kingdom, a form of state
more expenditure with less income and receipts, socio- support.
political goals could have been realised as per the Objective : To know about the Tax Credit
aspirations of the public policy! And once the growth
had taken place the extra money spent above the Report and Indices
income would have been reimbursed or repaid! This
168)Recently Marayoor Jaggery has received the
was a good public/government wish which was
Geographical Indication (GI) tag from the Central
fulfilled by the evolution of the idea of deficit
Government. The traditional and handmade product
financing.
is from
Objective :
A) Kerala B) Tamilnadu
To know about the Deficit Financing in general and
C) Karnataka D) Andhra Pradesh
status in India.

Key : A
166)Consider the following taxes:
Explanation :
1.Service tax
• The Marayoor Jaggery, the traditional and handmade
2.Capital gains tax
product from Idukki district of Kerala, has received
3.Securities Transaction tax the Geographical Indication (GI) tag from the Central
Which one of the above is/are indirect tax/taxes? Government.
A) 1 only B) 1 and 3 only • The Jaggery is produced in Marayoor, a town in Idukki
C) 2 and 3 only D) 1, 2 and 3 district of Kerala. It is made from sugarcane and no
Key : A chemicals are added during the manufacturing
Explanation : process.
• It is not produced in modern factories or using modern
• Service tax is levied by Government on Service
equipment. It is prepared in sheds located on the
Providers and Service Providers will in turn levy on
sugarcane farm.
Customers or individuals. So tax burden is not direct
on individuals so it is Indirect Tax So we can eliminate Objective :
the Option (C). To know about recent Geographical Indication
• Capital Gains tax is a Direct Tax; we can’t shift this Products
tax on to any one, as it is directly levied on Capital
gains by individual organisations like individuals or Monetary Policy
companies so it is direct tax. Hence we can eliminate 169)Recently the Reserve Bank Of India (RBI) has come
Options (c) and (d). up with new guidelines for White Label ATMs (WLAs).
• Securities transaction taxes are direct taxes as The White Label ATMs are operated by
securities transactions like bonds etc done by A) Cooperative Banks
individuals so they can’t shift the tax on them Hence
B) Non Banking Financial Companies
Answer is (a) only
C) Payment Banks
Objective :
D) Regional Rural Banks (RRBs)
To know about the differentiation of different taxes
whether they are direct or indirect. Key : B
167)What does tax credit mean? Explanation :

Page No.59
MARCH CURRENT AFFAIRS_2019
• The Reserve Bank Of India (RBI) has come up with a Economic Offender based on the evidence filed by the
review of operations of White Label ATMs (WLAs) in investigating agencies.
the country. • The person who is declared as a Fugitive Economic
• Automated Teller Machines (ATMs) set up, owned and Offender can challenge the proclamation in the High
operated by non-bank entities are called “White Label Court within 30 days of such declaration according
ATMs” (WLAs). They provide the banking services to to the Fugitive Economic Offenders Act, 2018.
the customers of banks in India, based on the cards Objective :
(debit/credit/prepaid) issued by banks. To know about the Fugitive Economic Offenders Act,
• Keeping the fact in view that banks won’t be able to 2018.
provide their ATM facilities in each and every place,
non-bank entities were allowed by the RBI to set up
Governance
White Label ATMs. This was done to increase the
geographical spread of ATMs and enhance the Government policies and
customer service. programs
Objective : 171)The Union Cabinet has approved setting up of a
To know about the details of White Label ATMs. National Mission on Transformative Mobility and
170)Which of the following is/are correct with regards Battery Storage. In this context, consider the following
to Fugitive Economic Offender Act, 2018? statements:
1.A person against whom an arrest warrant has been 1. The Mission will have an Inter-Ministerial Steering
issued for his or her involvement in economic offences Committee chaired by “Union Minister of Road
involving at least Rs. 100 Crore or more and has left Transport and Highways” to promote clean and
sustainable mobility initiatives.
India to avoid prosecution is considered as Fugitive
Economic Offender. 2. The Mission will have a ‘Make in India’ strategy for
2. Cases has to be filed in the special courts under EV components as well as battery technologies.
prevention of Money laundering act,2002. 3. The Mission will also launch the Phased
Select the correct answer using the code given below Manufacturing Programmes (PMP) for Batteries and
for Electric Vehicle components.
A) 1 only B) 2 only
Which of the statement(s) given above is/are correct?
C) Both 1 and 2 D) Neither 1 nor 2
A) 1 and 2 only B) 2 and 3 only
C) 1 and 3 only D) 1 2 and 3
Key : C
Key : B
Explanation :
Explanation :
• The Union Cabinet has approved setting up of a
• The Prevention of Money Laundering (PMLA) court in National Mission on Transformative Mobility and
Mumbai has declared Vijay Mallya as a Fugitive Battery Storage.
Economic Offender. V ijay Mallya is the first
businessman to be charged under the new fugitive
economic offender’s act 2018. • The Mission will have an Inter-Ministerial Steering
Committee chaired by Chief Executive Officer (CEO),
Declaration of the Fugitive Economic Offender
NITI Aayog to promote clean, connected, shared,
Vijay Mallya is declared as the Fugitive Economic sustainable and holistic mobility initiatives.
Offender under the following provisions: • The Mission will also launch the Phased
• According to the Fugitive Economic Offenders Act, Manufacturing Programmes (PMP) for Batteries and
2018 a fugitive economic offender is a person against for Electric Vehicle components.
whom an arrest warrant has been issued for his or • The mission will finalize and implement strategies
her involvement in economic offences involving at for transformative mobility and Phased
least Rs. 100 crore or more and has left India to avoid Manufacturing Programmes (PMP) for electric
prosecution. vehicles, their components and batteries.
• The investigating agencies have to file an application • The Mission will have a ‘Make in India’ strategy for
in a Special Court under the Prevention of Money- EV components as well as battery technologies.
Laundering Act, 2002 containing details of the
Objective :
properties to be confiscated, and any information
about the person’s whereabouts. To know about National Mission on Transformative
Mobility and Battery Storage.
• The Special Court will issue a notice for the person to
appear at a specified place and date at least six weeks
from the issue of notice. 172)Which of the following is/are correct with respect to
• Proceedings will be terminated if the person appears. Universal Basic Income (UBI)?
If not the person would be declared as a Fugitive 1.Universal Basic Income is a targeted scheme

Page No.60
MARCH CURRENT AFFAIRS_2019
2.It is an in- kind transfer scheme. 3. Information Sharing Platform Gateway for South-
3.Universal Basic Income is unconditional in nature. East Asia Regulatory Network (SEARN) is developed by
Select the correct answer using the code given below Centre for Development of Advanced Computing.
A) 1 and 2 only B) 2 and 3 only Which of the above statements is/are correct?
C) 1 and 3 only D) 1, 2 and 3 A) 1 and 2 only B) 2 and 3 only
Key : C C) 1 and 3 only D) 1 2 and 3
Explanation : Key : D
• Universal Basic Income is a periodic, unconditional Explanation :
cash transfer to every citizen in the country. • Information Sharing Platform Gateway for South-East
• Here, social or economic positions of the individual Asia Regulatory Network (SEARN) developed by Centre
are not taken into consideration. for Development of Advanced Computing has been
launched. It will promote regulatory and health
The concept of universal basic income has collaboration among the countries of the South-East
three main features. They are as following: Asia Region.
• UBI is universal in nature. It means UBI is not targeted. • The South East Asia Research Network (SEARN), based
• The second feature of UBI is cash transfer instead of at the London School of Hygiene & Tropical Medicine,
in-kind transfer. is a platform to facilitate research collaboration. It
• The third feature is that UBI is unconditional. That provides a forum to support the communication and
means one need not prove his or her unemployment dissemination of research findings, highlight research
status or socio-economic identity to be eligible for areas and a network connecting people and
UBI. collaborators outside with an interest in South East
Cash Transfers are Unconditional Asia.
• A cash transfer is simply a payment from the • Vision: Healthy populations with timely access to
government to help improve the lives of its citizens. affordable medical products of assured quality, safety
Examples of cash transfer programs in the U.S. include and efficacy in all countries of the South-East Asia
Social Security and unemployment benefits. Cash region and beyond.
transfer payments can be made in a lump sum or in • SEARN includes all ASEAN countries.
many smaller installments. Objective :
• Frequently these are termed unconditional cash To know about South-East Asia Regulatory Network
transfers, because the government imposes no (SEARN)
restrictions on how the aid money may be spent.
Recipients can use the money any way their wish and
History_Modern
have the greatest freedom over what goods and
services they buy. Freedom Struggle Begins
Defining In-Kind Benefits 174)Consider the following statements with respect to
the “Indian Official Secrets Act”.
• An in-kind transfer is also a type of public spending
to help specific populations. Unlike a cash transfer, 1. The Indian Official Secrets Act was enacted during
it takes the form of specific goods and services, which the time of Lord Lytton.
recipients get for free or at a reduced rate. One 2. One of the main purposes of the Act was to muzzle
example of a U.S. in-kind transfer program is Medicare, the voice of nationalist publications.
which subsidizes health care for senior citizens and Which of the statement(s) given above is/are
the disabled. Veterans’ benefits and Stafford student incorrect?
loans are other examples. A) 1 only B) 2 only
Objective : C) Both 1 and 2 D) Neither 1 nor 2
To know about the Universal Basic Income scheme. Key : A
Explanation :
International Relations • The Indian Official Secrets Act, 1904 was enacted
Asia & Pacific during the time of Lord Curzon, Viceroy of India from
173)Consider the following statements about South-East 1899 to 1905.
Asia Regulatory Network (SEARN). • One of the main purposes of the Act was to muzzle the
1. It promotes timely access to affordable medical voice of nationalist publications. However, the Indian
products of assured quality in countries of the South- Official Secrets Act (Act No XIX of 1923) replaced the
East Asia region and beyond. earlier Act, and was extended to all matters of secrecy
2. SEARN includes all ASEAN countries. and confidentiality in governance in the country.
• The secrecy law broadly deals with two aspects —
spying or espionage, which is dealt with in Section 3

Page No.61
MARCH CURRENT AFFAIRS_2019
of the Act, and disclosure of other secret information employment opportunities and promoting cooperation
of the government, which is dealt with in Section 5. with neighbouring countries.
The secret information can be any official code, • The V ision Document has been a focused and
password, sketch, plan, model, article, note, document consultative exercise to develop a common and shared
or information. aspiration for benefiting people of the north east
Objective : region. With involvement and inputs of various
To know about “Indian Official Secrets Act”. stakeholders, industry players and state governments,
the Vision document not only includes the ambition
for the region but also an actionable roadmap. CRISIL
Polity Infrastructure Advisory has been the Knowledge
Judiciary Partner to develop this Vision report.
175)The article which describes that Supreme Court shall • The objectives of the plan are to leverage the region’s
be a court of record and shall have all the powers of hydrocarbon potential, enhance access to clean fuels,
such a court including the power to punish for improve availability of petroleum products, facilitate
contempt of court is: economic development and to link common people to
A) Article 127 B) Article 128 the economic activities in this sector. The states
C) Article 129 D) Article 130 covered include Arunachal Pradesh, Assam, Manipur,
Key : C Meghalaya, Mizoram, Nagaland, Sikkim and Tripura.
The Ministry also undertook series of consultations
Explanation :
with the state governments while drafting the vision
• Under Constitution of India, Articles 129 and 215 document.
empowers the Supreme Court and High Court
• Beyond production, the focus areas include exploring
respectively to punish people for their respective
hydrocarbon linkages and trade opportunities with
contempt. Section 10 of The Contempt of Courts Act of
Bangladesh, Myanmar, Nepal & Bhutan;
1971 defines the power of the High Court to punish
implementation of ‘Make In India’ in the region;
contempt of its subordinate courts.
development of health & medical facilities; industrial
• The law of contempt of court in India is a hangover policy & infrastructure related action points; focus
from British rule. The law originated in England from on skill development; and employment generation
an undelivered judgement by J Wilmot in 1765. The requirement in the region. The vision statement lays
judge said the power of contempt of court was required out a detailed roadmap for the entire hydrocarbons
to uphold the dignity and majesty of judges. value chain, covering upstream, midstream and
• The courts in India are criticised on the ground that downstream segments. This report includes an action
they often invoke the provisions of contempt of court plan – of immediate, medium-term and long-term
to punish expressions of dissent, even when such initiatives – to help achieve the objectives.
expressions are not causing any threat to the Objective :
administration of justice.
To know about the Hydrocarbon Vision 2030.
Objective :
To know about the Contempt of Court
Geography
Geomorphology
Science & Technology
177)Which of the following sequence is correct with
Current affairs on Science regards to Speed or velocity of the Primary waves in
and technology different materials?
176) Select the correct from the following statements with A) Solids > Liquids > Gases
regards to HYDROCARBON VISION 2030: B) Liquids > Solids > Gases
1.The vision aims at doubling Oil & Gas Production C) Solids > Gases > Liquids
by 2030.
D) Gases > liquids > solids
2.CRISIL infrastructure advisory has been the
Key : A
knowledge Partner to develop this vision report
Explanation :
3.The states covered include all North eastern states.
• Primary Waves (P waves)
Select the correct answer using the code given below
• Also called as the longitudinal or compressional
A) 1 and 2 only B) 2 and 3 only
waves.
C) 1and 3 only D) 1, 2 and 3
• Analogous to sound waves.
Key : B
• Particles of the medium vibrate along the direction of
Explanation : propagation of the wave.
• The Hydro Carbon Vision 2030 aims at doubling Oil • P-waves move faster and are the first to arrive at the
& Gas production by 2030, making clean fuels surface.
accessible, fast tracking projects, generating

Page No.62
MARCH CURRENT AFFAIRS_2019
• These waves are of high frequency. • Ilbert Bill is named after Courtenay Peregrine Ilbert,
• They can travel in all mediums. who was appointed as legal adviser to the Council of
• Velocity of P waves in Solids > Liquids > Gases. India.
• Their velocity depends on shear strength or elasticity • The bill was introduced in 1883 by Viceroy Ripon,
of the material. who actually desired to abolish the racial prejudice
from the Indian Penal Code. Ripon had proposed an
• [We usually say that the speed of sound waves amendment for existing laws in the country and to
depends on density. But there are few exceptions. For allow Indian judges and magistrates the jurisdiction
example: Mercury (liquid metal) has density greater to try British offenders in criminal cases at the District
than Iron but speed of sound in mercury is lesser level.
compared to that in iron. This is because the shear
strength of mercury is very low (this is why mercury • It was never allowed before.
is liquid) compared to that of iron.] • So naturally, the Europeans living in India looked it
Objective : as a Humiliation and the introduction of the bill led
to intense opposition in Britain as well as India (by
To know about the Seismic waves and its nature like
the British residents). So it was withdrawn but was
speed, intensity etc.
reintroduced and enacted in 1884 in a severely
compromised state.
2019-03-09 Objective :
History_Modern To know about the Ilbert Bill Controversy in India and
The British Conquest of India Britain.
178)The tendency for increased litigation was visible after
the introduction of the land settlement system of Lord Socio-Religious Reform
Cornwallis in 1793. The reason for this is normally Movements
traced to which of the following provisions??
180)With reference to period of colonial rule in India,
A) Making Zamindar’s position stronger
“Home Charges” formed an important part of drain of
B) Making East India Company an overload of wealth from India. Which of the following funds
Zamindars constituted “Home Charges”?
C) Making Judicial System more efficient 1.Funds used to support the India Office in London
D) None of the above. 2.Funds used to pay salaries and pensions of British
Key : D personnel engaged in India.
Explanation : 3.Funds used for waging wars outside India by the
• Options (c) is irrelevant as there was no case or word British
of Judicial system in the features of Zamindari or Select the correct answer using the code given below
Permanent Settlement system A) 1 only B) 1 and 2 only
• Options (a) and (b) both are not the features of Land C) 2 and 3 only D) 1, 2 and 3
settlement system. British concentrated over the
Key : B
revenue collection from the ryots but not on authority
over the ryots by the Zamindars and East India Explanation :
Company. • Home charges refer to the expenditure incurred in
Objective : England by the Secretary of State on behalf of India.
Before revolt of 1857 the Home Charges varied from
To know about the Zamindari system or Permanent
10% to 13% of the average revenues of India. After the
Settlement system introduced by Lord Cornwallis. revolt the proportion shot up to 24% in the period
1897 to 1901. During 1921-22, the Home Charges
Freedom Struggle Begins sharply increased to 40% of the total revenue of the
179)The Ilbert Bill controversy was related to the: Central Government.
A) Impose restrictions over the Indians to carry arms The Main constituents of the Home charges were:
B) Impositions of restrictions on newspapers and 1.Home charges refer to the interest on public debt
magazines published in Indian languages. raised in England at comparatively higher rates;
C) Removal of disqualifications imposed on the Indian Expenditure incurred in England by the Secretary of
magistrates with regard to the trial of the Europeans State on behalf of India;
D) Removal of duty on imported cotton cloth 2.Annuities on account of railway and irrigation
works;
Key : C
3.Indian office expenses including pensions to retired
Explanation :
officials who had worked in India or England,
pensions to army and Navals etc.

Page No.63
MARCH CURRENT AFFAIRS_2019
4.Remittances to England by Europeans to their Important Personalities in
families National Movement
5.Remittances for purchase of British goods for 182)Which of the following statements is/are correct
consumption of British employees in India. regarding Brahmo Samaj?
6.Interest charges on public debt held in Britain 1.It Opposed idolatry
7.Also trade as well as Indian labour was deeply 2.It denied the need for a priestly class for interpreting
undervalued. the religious texts
8.Civil and military charges include payments 3.It popularized the doctrine that the Vedas are
towards pensions and furloughs of Britain officers in infallible
the civil and military departments in India, expenses Select the correct answer using the code given below
on India office establishment in London, payments to
A) 1 only B) 1 and 2 only
the British war office etc. All these charges were solely
due to India’s subjection to foreign rule. C) 3 only D) 1, 2 and 3
Objective : Key : B
To know about the Home charges which are the parts Explanation :
of expenditure made in Britain under the part of Drain • Brahmo Samaj began as a reformist movement within
of wealth. Hinduism and began the Bengal Renaissance. It was
started at Calcutta in 1828 by Raja Ram Mohan Roy
181)Which of the following contribution (s) is/are made and Debendranath Tagore as a reform movement and
by Lord Cornwallis? proved to be one of the most influential religious
reform movements in India, and made significant
1.Purification of civil services contribution to the making of modern India.
2.Pay handsome salaries to the company servants • The Brahmo Samaj made an effort to reform Hindu
3.Separation of 3 branches of service, namely religion by removing abuses and by abusing it on the
commercial, judicial and revenue worship of one god and on the teaching of the Vedas
4.Revenue reforms and Upanishads even though it repudiated the
Select the correct answer using the code given below doctrine of the “infallibility of the Vedas”.
A) 1, 2 and 3 only B) 2, 3 and 4 only The various principles of Brahmo Samaj:
C) 1, 3 and 4 only D) 1, 2, 3 and 4 • God – Brahmo Samaj advocates the existence of an
Key : A ‘Infinite Singularity.’ It does not believe in Avatars,
reincarnation, or rebirths. It also denounces idol-
Explanation :
worship and polytheism. The ‘Infinite Singularity’ is
Administrative reforms: considered limitless, indivisible, imperceivable, and
1. Greatest work: Purification of the civil service by the undefinable.
employment of capable and honest public servants. • Love – While it urges its followers to treat all creations
2. Found that the servants of the company were with respect, Brahmo Samaj never encourages anyone
underpaid. to worship anything or anybody, as only the
3. They have received very high commissions on Singularity or ‘Brahman’ can be worshipped and
revenues. They conducted forbidden and profitable adored.
private trade in the names of relatives and friends. • Scripture – The followers of Brahmo Samaj, who are
4. He persuaded the Directors of the Company to pay called as ‘Brahmo Samajists’ do not believe in any
handsome salaries to the company servants. scriptures, prophets and mediators between Brahman
5. Cornwallis inaugurated the policy of making and the human soul. Not surprisingly, they don’t
appointments mainly on the basis of merit. believe in the doctrines of Karma as well. However,
Brahmo Samajists can believe in Karma and rebirth,
6. Laying foundation of the Civil Service
depending upon their personal preference.
7. The major reform was Separation of the three
• Liberation – Though Brahmo Samaj believes in the
branches of service, namely commercial, judicial and
concept of soul and that the soul is immortal and is
revenue.
poised to become a part of Brahman, it does not
Objective : advocate the concept of salvation or ‘Mukthi.’ Also, it
To know about the contributions made by different doesn’t believe in the concepts of hell and heaven.
governor generals, governors and viceroys during • Knowledge – The followers embrace knowledge, truth,
British rule in India. and free will. They also consider righteousness as
their only way of life. Though the Samaj embraces
secular principles, it opposes sectarianism and the
act of imposing religious beliefs onto someone.
• Superstition – Brahmo Samaj condemns superstition
and dogma. In fact, superstitious practices like ‘Sati’

Page No.64
MARCH CURRENT AFFAIRS_2019
were one of the primary reasons why Brahmo Samaj • Officially known as the Anarchical and Revolutionary
came into existence. It also validates the supposed Crimes Act, 1919
insignificance of unscientific rituals, priests, and • Passed in March 1919 by the Imperial Legislative
places of worship like temple, church, and mosque. Council
• Totalitarianism – Brahmo Samaj condemns all forms • This act authorised the British government to arrest
of totalitarianism. It also rejects irrational and bigotry anybody suspected of terrorist activities.
distinctions like religion, color, creed, caste, and race. • It also authorised the government to detain such
These distinctions were considered evil as they often people arrested for up to 2 years without trial.
formed the reasons that created rift between human
beings. • It empowered the police to search a place without a
warrant.
Objective :
• It also placed severe restrictions on the freedom of
To know about the Brahmo Samaj ideology. the press.
• The act was passed as per recommendations of the
183)During the nationalist movements, series of “Praja Rowlatt Committee chaired by a judge, Sir Sidney
Mandals” were established Rowlatt.
1.To promote the nationalist creed in the princely • The act was widely condemned by Indian leaders and
states. the public. The bills came to be known as ‘black bills’.
2.To promote the tradition of wearing dhoti-kurta for • The act was passed despite unanimous opposition
all the nationalists. from the Indian members of the council, all of whom
3.To oppose Simon Commission of 1927 which was to resigned in protest. These included Mohammed Ali
report on the working of the Indian constitution Jinnah, Madan Mohan Malviya and Mazhar Ul Haq.
established by the Government of India Act of 1919 Objective :
Which of the following statement(s) is/are correct? To know about the Rowlatt act (1919 feb)
A) 1 only B) 1 and 2 only
C) 1 and 3 only D) 1, 2 and 3 185) The Partition of Bengal made by Lord Curzon in 1905
Key : A lasted until:
Explanation : A) The first World war when Indian troops were needed
• Whi le Mahatma Gandhi’s mass appeal was by the British and the partition was ended.
undoubtedly genuine – and in the context of Indian B) King George V abrogated Curzon’s Act at the Royal
politics, without precedent – it must also be stressed Darbar in Delhi in 1911
that his success in broadening the basis of C) Gandhiji launched his civil Disobedience
nationalism was based on careful organisation. New Movement
branches of the Congress were set up in various parts D) The Partition of India, in 1947 when East Bengal
of India. A series of “Praja Mandals” were established became East Pakistan
to promote the nationalist creed in the princely states.
Gandhiji encouraged the communication of the Key : B
nationalist message in the mother tongue, rather than Explanation :
in the language of the rulers, English. Thus the 1. The idea of using the Bengal partition as a political
provincial committees of the Congress were based on tool to undermine the growing nationalism in Bengal
linguistic regions, rather than on the artificial and other parts of India occurred later.
boundaries of British India. 2. As per Curzon, after the partition the two provinces
Objective : would be Bengal (including modern West Bengal,
To know about the “Praja Mandals” during nationalist Odisha and Bihar) and Eastern Bengal and Assam.
movements. 3. Bengal would also lose five Hindi-speaking states to
the Central Provinces. It would gain Odia-speaking
National movement 1905-1919 states from the Central Provinces.
184)The Rowlatt Act aimed at: 4. Eastern Bengal would consist of Hi ll Tripura,
Chittagong, Rajshahi and Dhaka divisions. Its capital
A) Compulsory economic support to war efforts would be Dhaka.
B) Imprisonment without trial and summary 5. Bengal would have a Hindu majority and Eastern
procedures for trial Bengal and Assam would have a Muslim majority
C) Suppression of the Khilafat Movement population. Its capital would remain Calcutta.
D) Imposition of restrictions on freedom of the press Partition annulled:
• Owing to mass political protests, the partition was
Key : B annulled in 1911.
Explanation :

Page No.65
MARCH CURRENT AFFAIRS_2019
• New provinces were created based on linguistic lines The beginning of European
rather than religious lines. Bihar and Orissa Province Settlements
was carved out of Bengal. (Bihar and Orissa became 187)Which of the following is/are introduced during the
separate provinces in 1936) rule of first Governor – General of Bengal?
• A separate Assam province was created 1.Abolition of the Dual System
• The capital of British India was moved to Delhi from 2.Reorganisation of the Judicial System
Calcutta in 1911.
3.Trade regulations and other reforms
• Despite the annulment, the partition did create a
4.The regulating act of 1773
communal divide among the Hindus and Muslims of
Bengal. 5.Pitt’s India Act, 1784
Objective : Select the correct answer using the code given below
To know about the Partition of Bengal. A) 1, 2, 3 and 4 only B) 1 and 3 only
C) 1, 3 and 4 only D) 1, 2, 3, 4 and 5
Emergence of Gandhi Key : D
186)The Lahore session of the Indian National Congress Explanation :
(INC) (1929) is very important in History, because: • The first Governor – General of India is Warren
1.The congress passed a resolution demanding Hasting (1774 – 1785). The following are the reforms
complete independence were introduced in India during the rule of Warrant
2.The rift between the extremists and moderates was Hasting:
resolved in that session •. Abolition of the Dual System
3.A resolution was passed rejecting the two – nation •. Dual System introduced by Robert Clive was abolished.
theory in that session •. To improve the finances of the Company, Warren
Which of the statements given above is/are correct? Hastings reduced the Nawab’s allowances of 32lakhs
of rupees to half that amount.
A) 1 only B) 2 and 3 only
2. Revenue reforms
C) 1 and 3 only D) 1, 2 and 3
•. A Board of Revenue was established at Calcutta to
Key : A
supervise the collection of revenue
Explanation :
•. Board of Revenue framed out the lands by auction for
• The 1929 Lahore session under the presidency of a period of five years instead of one year in order to
Jawaharlal Nehru holds special significance as in this find out their real value. Zamindars were given priority
session “Purna Swaraj” (complete independence) was in the auction.
declared as the goal of the INC. 26 January, 1930 was
•. Calcutta thus became the capital of Bengal in 1772.
declared as “Purna Swaraj Diwas”.
English Collectors were appointed in each district.
• The rift between moderates and extremists was
•. Accountant General was appointed
resolved through Lucknow Pact (December 1916) was
signed after both the Congress and the Muslim League 3. Reorganisation of the Judicial System
realised that they will have to come together and •. Store house of abuses (like Nawab was misused his
jointly demand self rule for India from British rulers. powers judgement were careless, where as Zamindars,
... Congress was then led by Bal Gangadhar Tilak, who acted as judges at lower levels within their own
while Mohammed Ali Jinnah represented the League areas were highly corrupt and prejudiced). Therefore,
• The two nation theory was not the proposal or demand Warren Hastings reorganized the judiciary.
during the INC Lahore session in 1929. It became a •. Each district was provided with a civil court under
word in 1940’s. the Collector and a criminal court.
• In 1940 in Lahore Muhammad Ali Jinnah, the man •. To hear appeals from the district courts two appellate
who founded Pakistan gave a seminal speech setting courts, one for civil cases and another for criminal
out the need for a separate state for Muslims on the cases, were established at Calcutta.
subcontinent. Prior to the division of India in 1947, •. Highest civil court of appeal was called Sadar Diwani
Hindus and Muslims had lived together across the Adalat presided over by the governor and two judges
country. But Jinnah described them as two separate recruited from among the members of his council.
nations. •. Highest appellate criminal court was known as Sadar
Objective : Nizamat Adalat function under an Indian judge
To know about the importance of Lahore session of appointed by the Governor –in-council
Indian National Congress 1929. 4. Trade regulations and other reforms
5. The Regulating act of 1773
6. Expansion of Policy of Warren Hastings
7. Pitt’s India Act, 1784

Page No.66
MARCH CURRENT AFFAIRS_2019
Objective : Sun Kosi Projects in Nepal which would benefit both
To know about the reforms taken in India by the British countries.
East India Company during the rule of different •.The funding pattern for FM Component for works in
Governor’s, Governor – General’s and Viceroys. general category States will continue to be 50%
(Centre): 50% (State) and for projects of North Eastern
Governance States, Sikkim, J&K, Himachal Pradesh and
Uttarakhand, the funding pattern will continue to be
Government policies and 70% (Centre): 30% (State).
programs
•. RMBA component being specific to activities in border
188)The Union Cabinet has approved the “Flood areas with neighbouring countries and in accordance
Management and Border Areas Programme (FMBAP)” with bilateral mechanisms, the projects / works will
for Flood Management Works. Consider the following continue to be funded as 100% grant-in-aid / central
statements about the scheme: assistance.
1. The FMBAP Scheme will be implemented throughout Objective :
the country for effective flood management, erosion
To know about “Flood Management and Border Areas
control and anti-sea erosion.
Programme (FMBAP)” scheme
2. The aim of the Scheme is to assist the State
Governments to provide reasonable degree of
protection against floods in critical areas by adopting Polity
optimum combination of structural and non- Constitutional bodies (Origin,
structural measures. structure and functions)
3. The funding pattern for FM Component for works in 189)Consider the following statements with respect to
general category States will continue to be 70:30 and Model Code of Conduct (MCC)
for projects of North Eastern States 90:10 between 1. It refers to a set of norms laid down by the Election
centre and state. Commission of India, with the consensus of political
Which of the statement(s) given above is/are correct? parties.
A) 1 and 2 only B) 2 and 3 only 2.The code comes into force on the announcement of
C) 1 and 3 only D) 1 2 and 3 the poll schedule and remains operational till the
Key : A process is concluded, as provided in the notification.
Explanation : 3. It is also applicable to a caretaker government on
premature dissolution.
•. The FMBAP Scheme will be implemented throughout
the country for effective flood management, erosion Which of the statement(s) given above is/are correct?
control and anti-sea erosion. A) 1 and 2 only B) 2 and 3 only
•. The Scheme “FMBAP” has been framed by merging the C) 1 and 3 only D) 1, 2 and 3
components of two continuing XII Plan schemes titled Key : D
“Flood Management Programme (FMP)” and “River Explanation :
Management Activities and Works related to Border •. The model code refers to a set of norms laid down by
Areas (RMBA)”. the Election Commission of India, with the consensus
•. The aim of the Scheme is to assist the State of political parties. It is not statutory. It spells out the
Governments to provide reasonable degree of dos and don’ts for elections. Political parties,
protection against floods in critical areas by adopting candidates and polling agents are expected to observe
optimum combination of structural and non- the norms, on matters ranging from the content of
structural measures and enhancing capabilities of election manifestos, speeches and processions, to
State/ Central Government officials in related fields. general conduct, so that free and fair elections take
•. The works under the scheme will protect valuable land place.
from erosion and flooding and help in maintaining •. The code comes into force on the announcement of
peace along the border. the poll schedule and remains operational till the
•. The Scheme aims at completion of the on-going process is concluded, as provided in the notification.
projects already approved under FMP. Further, the It is also applicable to a “caretaker” government on
scheme also caters to Hydro-meteorological premature dissolution of a State Assembly, as was
observations and Flood Forecasting on common rivers the case in Telangana.
with the neighbouring countries. •. The EC ensures that ruling parties at the Centre and in
•. The Scheme also includes survey and investigations, States adhere to the code, as part of its mandate to
preparation of DPR etc. of water resources projects conduct free and fair elections under Article 324 of
on the common rivers with neighbouring countries the Constitution. In case of electoral offences,
like Pancheshwar Multipurpose Project, Sapta Kosi- malpractices and corrupt practices like inducements
to voters, bribery, intimidation or any undue influence,

Page No.67
MARCH CURRENT AFFAIRS_2019
the EC takes action against violators. Anyone can •. SAT is a statutory body established under the
report the violations to the EC or approach the court. provisions of Section 15K of the Securities and
The EC has devised several mechanisms to take note Exchange Board of India Act, 1992.
of the offences, which include joint task forces of •. The Tribunal is a three-member body composed of a
enforcement agencies and flying squads. The latest is Presiding Officer and two other members who are to
the introduction of the cVIGIL mobile app through be nominated via a notification by the Central
which audio-visual evidence of malpractices can be Government. The Union Government also reserves the
reported. right to notify as many SAT’s as is needed.
Objective : •. The Securities Appellate Tribunal has only one bench
To know about the Model Code of Conduct. that sits at Mumbai and has jurisdiction over all of
India.
Indian Society •. The Securities Appellate Tribunal is not be bound by
Effects of globalization on the procedure laid down by the Code of Civil
Procedure, 1908, but is be guided by the principles of
Indian society
natural justice and, subject to the other provisions of
190)Recently ‘Bolo’ application is sometimes seen in the Depositories Act, 1996.
news which aims to help children in primary school
•. The Securities Appellate Tribunal has powers to
to read in Hindi and English. The app was launched
regulate its own procedure including the places at
by
which it shall have its sittings.
A) Google B) Microsoft Objective :
C) Facebook D) Twitter To know about the details of ‘Securities Appellate
Key : A Tribunal (SAT)’.
Explanation :
•. Google has launched a new application called ‘Bolo’ International Relations
that aims to help children in primary school to read
International Institutions,
in Hindi and English.
agencies and their structure
•. The app, which is being launched in India first, uses
Google’s speech recognition and text-to-speech and mandate
technology. 192)Recently IBBI signs a Cooperation Agreement with
•. It comes with a built-in fun and helpful reading buddy, the International Finance Corporation (IFC) to further
an animated character called ‘Diya’. The reading build the capacity of the Insolvency Professionals. In
material available on the app will be completely free this context, consider the following statements with
of cost. respect to IFC:
•. The app aims to help improve the reading ability of 1. The IFC is an international financial institution that
children, as the lack of it can significantly impact offers investment, advisory, and asset management
further education and ultimately children’s ability to services to encourage private sector development in
realise their full potential. developing countries.
Objective : 2. It is a member of the World Bank Group.
To know about the recent Google initiative- Bolo App. 3. It is a corporation whose shareholders are member
governments that provide paid-in capital and which
have the right to vote on its matters.
sEconomy
Which of the statement(s) given above is/are correct?
Capital Market A) 1 only B) 1 and 2 only
191)Consider the following statements with respect to C) 2 and 3 only D) 1 2 and 3
‘Securities Appellate Tribunal (SAT)’:
Key : D
1. SAT is a statutory body established under the
provisions of the Securities and Exchange Board of Explanation :
India Act, 1992. •. The International Finance Corporation (IFC) is an
2. The Tribunal is a three-member body composed of international financial institution that offers
a Presiding Officer and two other members who are investment, advisory, and asset management services
to be nominated via a notification by the Central to encourage private sector development in developing
Government. countries.
Which of the statement(s) given above is/are correct? •. It is a member of the World Bank Group and is
headquartered in Washington, D.C., United States.
A) 1 only B) 2 only
•. It was established in 1956 as the private sector arm
C) Both 1 and 2 D) Neither 1 nor 2
of the World Bank Group to advance economic
Key : C development by investing in strictly for-profit and
Explanation :

Page No.68
MARCH CURRENT AFFAIRS_2019
commercial projects that purport to reduce poverty Key : A
and promote development. Explanation :
•. The IFC is owned and governed by its member •. Ministry: Ministry of Statistics and Programme
countries, but has its own executive leadership and Implementation
staff that conduct its normal business operations. About MPLAD scheme:
•. It is a corporation whose shareholders are member Launched in December, 1993
governments that provide paid-in capital and which
•. It provides a mechanism for the Members of
have the right to vote on its matters.
Parliament to recommend works of developmental
Objective : nature for creation of durable community assets and
To know about the details of International Finance for provision of basic facilities including community
Corporation (IFC). infrastructure, based on locally felt needs.
Salient features:
2019-03-08 • MPLADS is a centrally-sponsored plan scheme fully
Governance funded by the government of India under which funds
Government policies and are released in the form of grants in-aid directly to
programs the district authorities.
193)Consider the following statements with respect to ‘e- • Works, developmental in nature, based on locally felt
Dharti Geo Portal’: needs and always available for the use of the public
at large, are eligible under the scheme.
1. Through e-Dharti Geo Portal the lessee of the
property will be able to see the basic details of • Preference under the scheme is given to works relating
property along with map showing its location. to national priorities, such as provision of drinking
water, public health, education, sanitation, roads, etc.
2. The portal was launched by Ministry of Earth
Sciences. • The funds released under the scheme are non-
lapsable. Funds not released in a particular year is
Which of the statement(s) given above is/are
carried forward to the subsequent years, subject to
incorrect?
eligibility.
A) 1 only B) 2 only
• The MPs have a recommendatory role under the
C) Both 1 and 2 D) Neither 1 nor 2 scheme. They recommend their choice of works to the
Key : B concerned district authorities who implement these
Explanation : works by following the established procedures of the
•. Union ministry of Housing and Urban Affairs has concerned state government.
recently launched e-Dharti Geo Portal, through which • The district authority is empowered to examine the
the lessee of the property will be able to see the basic eligibility of works sanction funds and select the
details of property along with map showing its implementing agencies, prioritise works, supervise
location. overall execution, and monitor the scheme at the
•. The Ministry also launched e-Dharti app which is a ground level.
new online system where all the three main modules • The district authorities get the works executed through
under the jurisdiction of Land and Development Office the line departments, local self-governments or other
(L&DO) i.e. Conversion, Substitution and Mutation government agencies. In some cases, the district
have been made online for the citizens. authorities get the works executed through reputed
Objective : non-government organisations.
To know about e-Dharti Geo Portal. • The Lok Sabha Members can recommend works in their
respective constituencies.
• The elected members of the Rajya Sabha can
194)Which of the following is/are correct with regards
recommend works anywhere in the state from which
to Members of Parliament Local Area Development
they are elected.
Scheme (MPLAD)?
• Nominated members of the Lok Sabha and Rajya Sabha
1.MPLAD Scheme comes under Ministry of Statistics
may select works for implementation anywhere in the
and Programme Implementation.
country.
2.MPLADs are a centrally sponsored scheme fully
Objective :
funded by government of India under which funds are
released in the form of grants in-aid directly to the To know about the MPLADs scheme by Government of
district authorities. India.
3.The funds under the scheme are lapsable.
Select the correct answer using the code given below 195)Which of the following is/are correct with regards
to National Mineral Policy 2019?
A) 1 and 2 only B) 2 and 3 only
C) 1 and 3 only D) 1, 2 and 3

Page No.69
MARCH CURRENT AFFAIRS_2019
1.It proposes to grant status of industry to mining exclusive outlets known as Jan Aushadhi Medical
activity to boost financing of mining for private sector Stores.
in India . 2.The market share of generic medicines has grown
2.The new policy focuses on use coastal water ways over three fold from 2% to 7% in last 3 years.
and inland shipping for evacuation and 3.The scheme was launched by Ministry of Health and
transportation of minerals and encourages dedicated Family Welfare.
mineral corridors to the facilitate the transportation Which of the statement(s) given above is/are correct?
of minerals.
A) 1 and 2 only B) 2 and 3 only
Select the correct answer using the code given below
C) 1 and 3 only D) 1, 2 and 3
A) 1 only B) 2 only
Key : A
C) Both 1 and 2 D) Neither 1 nor 2
Explanation :
Key : C
•. Ministry of Chemicals and fertilizers has decided to
Explanation : celebrate 7th March 2019 as ‘Janaushadhi Diwas’
• The 2019 Policy proposes to grant status of industry across India. It is to provide further impetus & create
to mining activity to boost financing of mining for awareness about the use of generic medicines.
private sector and for acquisitions of mineral assets •. Under Pradhan Mantri Bhartiya Janaushadhi
in other countries by private sector. Pariyojana around 5050 Janaushadhi stores were
• It also mentions that Long term import export policy opened. It has provided the awareness about the
for mineral will help private sector in better planning availability of the high quality affordable generic
and stability in business. medicines in the country.
• The Policy also mentions rationalize reserved areas •. About 10-15 lakh people benefit from Janaushadhi
given to PSUs which have not been used and to put medicines per day.
these areas to auction, which will give more •. The market share of generic medicines has grown over
opportunity to private sector for participation. 3 fold from 2% to 7% in last 3 years.
• It also mentions to make efforts to harmonize taxes, Objective : To know about Government initiatives to
levies & royalty with world benchmarks to help private provide generic medicines covering all therapeutic
sector. groups.
• NMP 2019 proposes a long term export import policy
for the mineral sector to provide stability and as an
197) ‘Web Wonder Women campaign’ is sometimes seen
incentive for investing in large scale commercial
in the news. The campaign was launched by
mining activity.
A) United Nations
• Regarding the role of state in mineral development
online public portal with provision for generating B) Women for Women International
triggers at higher level in the event of delay of C) Ministry of Women and Child Development
clearances has been put in place. D) Centre for reproductive rights
• NMP 2019 aims to attract private investment through Key : C
incentives while the efforts would be made to maintain Explanation :
a database of mineral resources and tenements under
•. The Ministry of Women and Child Development hosted
mining tenement systems.
a felicitation event for Web Wonder Women. The
• The new policy focuses on use coastal waterways and Campaign has been launched by the Ministry of
inland shipping for evacuation and transportation of Women and Child Development, Government of India
minerals and encourages dedicated mineral corridors in association with the NGO Breakthrough and Twitter
to facilitate the transportation of minerals. India.
• The utilization of the district mineral fund for •. Through the campaign, the Ministry aims to recognize
equitable development of project affected persons and the fortitude of Indian women stalwarts from across
areas. the globe who have used the power of social media to
Objective : run positive & niche campaigns to steer a change in
To know about the National Mineral Policy 2019. society.
•. The Campaign is aimed at encouraging, recognizing
196) The government has decided to celebrate 7th March and acknowledging the efforts of these meritorious
2019 as ‘Janaushadhi Diwas’ across India.In this Women.
context, consider the following statements with Objective :
respect to ‘Pradhan Mantri Bhartiya Janaushadhi To know about the details of Web Wonder Women
Pariyojana’: campaign
1.The scheme was launched to provide quality
medicines at affordable prices to masses through

Page No.70
MARCH CURRENT AFFAIRS_2019
Science & Technology to a person and laws that states enact must conform
Current affairs on Science to the laws of the land like – fairness, fundamental
and technology rights, liberty etc. It also gives the judiciary to access
fundamental fairness, justice, and liberty of any
198) Recently ‘FASER’ Experiment is sometimes seen in legislation.
the news. Consider the following statements with
respect to the experiment: •. The difference in layman’s terms is as below: Due
Process of Law = Procedure Established by Law + The
1. It is a new experiment announced by European procedure should be fair and just and not arbitrary.
Organization for Nuclear Research (CERN).
•. Here Supremacy of Court over Constitution is favoured,
2. It will search for a suite of hypothesized particles
but in country like India Constitution is supreme.
including dark photons, particles which are
Hence statement 1 is a limitation why India didn’t
associated with dark matter, neutralinos and others.
adopt the due process of law.
Which of the statement(s) given above is/are correct?
•. Due Process of law can be subjective and Political, it
A) 1 only B) 2 only might not be work in India and hence it is dropped
C) Both 1 and 2 D) Neither 1 nor 2 and taken as limitation.
Key : C Procedure established by law:
Explanation : •. It means that a law that is duly enacted by the
•. Forward Search Experiment (FASER) is a new legislature or the concerned body is valid if it has
experiment announced by European Organization for followed the correct procedure. Following this
Nuclear Research (CERN). doctrine means that, a person can be deprived of his
•. It will complement CERN’s ongoing physics program, life or personal liberty according to the procedure
extending its discovery potential to several new established by law.
particles. It will look for light and weakly interacting Objective :
particles at the Large Hadron Collider (LHC). To know about the differences between Due Process
•. It will search for a suite of hypothesized particles of law and Procedure established by law.
including dark photons, particles which are
associated with dark matter, neutralinos and others. Legislature
•. Dark matter is a hypothesized kind of matter that does 200) Which of the following session of the parliament
not interact with the electromagnetic force and can terminate a sitting of a parliament for a specified
consequently cannot be directly detected using time?
emitted light. The mysterious dark matter makes up
A) Adjournment B) Adjournment Sine die
about 27% of the universe.
C) Prorogation D) Dissolution
Objective :
Key : A
To know about Forward Search Experiment (FASER)
Explanation :
•. A sitting of Parliament can be terminated by
Polity
adjournment, adjournment sine die, prorogation or
Judiciary dissolution. Technically, a session of Indian
199)Which of the following is/are the limitations of “Due Parliament is the period between the first sitting of a
process of law” which was dropped, and adopted the House and its prorogation or dissolution. The period
“Procedure established by law” by the framers of between the prorogation of a House and its
Indian Constitution in India? reassembly in a new session is called ‘recess’.
1.It favours supremacy of Courts over constitution Summoning:
2.Justice could become subjective and political •. Summoning is the process of calling all members of
Select the correct answer using the code given below the Parliament to meet. It is the duty of Indian
A) 1 only B) 2 only President to summon each House of the Parliament
from time to time. The maximum gap between two
C) Both 1 and 2 D) Neither 1 nor 2
sessions of Parliament cannot be more than six
Key : C months. In other words, the Parliament should meet
Explanation : at least twice a year.
Due process of law: Adjournment:
•. Due process of law doctrine not only checks if there •. An adjournment suspends the work in a sitting for a
is a law to deprive the life and personal liberty of a specified time, which may be hours, days or weeks. In
person but also see if the law made is fair, just and this case, the time of reassembly is specified. An
not arbitrary. adjournment only terminates a sitting and not a
•. Under due process, it is the legal requirement that the session of the House. The power of adjournment lies
state must respect all of the legal rights that are owed with the presiding officer of the House.

Page No.71
MARCH CURRENT AFFAIRS_2019
Adjournment sine die: assembly of the respective state and it acquires 2
•. Adjournment sine die means terminating a sitting of seats in the State Assembly.
Parliament for an indefinite period. In other words, • If the registered party secures 6% of the valid votes
when the House is adjourned without naming a day cast in the state at the general elections of the lower
for reassembly, it is called adjournment sine die. The house of Parliament, i.e. Lok Sabha, from the
power of adjournment sine die lies with the presiding respective states and it acquires one seat in the Lok
officer of the House. Sabha of the respective state.
Note: The presiding officer of a House can call a sitting • If the party acquires 3% of the seats in the State
of the House before the date or time to which it has Assembly at the general elections to the assembly of
been adjourned or at any time after the House has the respective state or three seats in the legislative
been adjourned sine die. assembly, whichever is greater.
Prorogation: • If the registered party acquires one seat in the lower
•. Prorogation means the termination of a session of house of the Parliament for every 25 seats or any part
the House by an order made by the President under thereof assigned to the state at the general elections
article 85(2)(a) of the Constitution. Prorogation to the Lok Sabha from the respective state.
terminates both the sitting and session of the House. • If the registered party obtains 8% of the total valid
Usually, within a few days after the House is adjourned votes cast in the state general elections to the lower
sine die by the presiding officer, the President issues house of the Parliament from the state or to the State
a notification for the prorogation of the session. Assembly.
However, the President can also prorogue the House To achieve the status of a national party, the
while in session. registered party must satisfy any of the following
Note: All pending notices (other than those for conditions:
introducing bills) lapse on prorogation and fresh • If the party occupies a minimum of 6% valid votes
notices have to be given for the next session. cast in any four or more than four states, in the general
Dissolution: elections of the Lok Sabha or the State Legislative
•. A dissolution ends the very life of the existing House, Assembly and it acquires a minimum of 4 seats in the
and a new House is constituted after general elections lower house from any state.
are held. Rajya Sabha, being a permanent House, is • If the party occupies a minimum of 2% seats in the Lok
not subject to dissolution. Only the Lok Sabha is Sabha, which are elected from a minimum of three
subject to dissolution. different states.
Objective : • If the political party is acknowledged as a state party
To know about the Sessions of the Parliament. in a minimum of four states.

Elections and representation Objective :


of people act To know about the Regional Parties or National Parties.
201) Which of the following is the eligibility to a registered
party to become a Regional party and National Party? Environment_Ecology
1.If the registered party acquires 3% of the seats in National Environment Laws &
the state assembly at the general elections to the Policies
assembly of the respective state or three seats in the 202) Consider the following statements with respect to
legislative assembly, whichever is greater Hazardous and Other Wastes Amendment Rules, 2019
2.If the party occupies a minimum of 6% of valid votes 1. It totally bans the import of solid plastic wastes in
cast in any four or more than four states, in the general to India, except when they are carried out by agencies
elections of the Lok Sabha or the state legislative situated in Special Economic Zones (SEZ).
assembly and it acquires a minimum of 4 seats in the
2. It mandates the exporters of silk waste to get prior
lower house from any state.
permission from the Ministry of Environment, Forest
Select the correct answer using the code given below and Climate Change.
A) 1 only B) 2 only
C) Both 1 and 2 D) Neither 1 nor 2 Which of the statement(s) given above is/are correct?
Key : C A) 1 only B) 2 only
Explanation : C) Both 1 and 2 only D) Neither 1 nor 2
The following conditions are to be satisfied by the Key : D
registered party to achieve the status of a regional
Explanation :
party:
• If the registered party occupies 6% of the valid votes •. Recently the Ministry of Environment, Forest and
cast in the states during the elections of the legislative Climate Change has amended the Hazardous and Other

Page No.72
MARCH CURRENT AFFAIRS_2019
Wastes (Management & Trans-boundary Movement) To know about the affected regions under the revolt of
Rules, 2016. 1857.
•. Some of the salient features of the Hazardous and
Other Wastes (Management & Trans-boundary 205) “In this instance we could not play off the
Movement) Amendment Rules, 2019 are as follows: Mohammedans against the Hindus” to which one of
•. Solid plastic waste has been prohibited from import the following events did this remark of Aitcheson
into the country including in Special Economic Zones relates?
(SEZ) and by Export Oriented Units (EOU). A) Revolt of 1857 B)Champaran Satyagraha
•. Exporters of silk waste have now been given exemption C) Khilafat and Non Cooperation Movement
from requiring permission from the Ministry of (1919-22)
Environment, Forest and Climate Change.
D) August Movement of 1942
• Electrical and electronic assemblies and components
manufactured in and exported from India, if found
defective can now be imported back into the country, Key : A
within a year of export, without obtaining permission Explanation :
from the Ministry of Environment, Forest and Climate •. Hindus and Muslim fought like brothers in arms in
Change. the revolt of 1857 only from the above options. This
•. Industries which do not require consent under Water unity was found again during Swadeshi Movement in
(Prevention and Control of Pollution) Act 1974 and 1905.
Air (Prevention and Control of Pollution) Act 1981, Objective :
are now exempted from requiring authorization also To know about the unity between Hindus and Muslim
under the Hazardous and Other Wastes (Management in the revolt of 1857 and later like in Indian Freedom
& Transboundary Movement) Rules, 2016, provided Struggle.
that hazardous and other wastes generated by such
industries are handed over to the authorized actual
users, waste collectors or disposal facilities. 206) The educated middle class in India:
Objective : A) Opposed the revolt of 1857
To know about Hazardous and Other Wastes B) Supported the revolt of 1857
Amendment Rules, 2019. C) Remained neutral to the revolt of 1857
D) Fought against native rulers.
2019-03-07 Key : A
History Modern Explanation :
The Revolt of 1857 •. They clearly opposed the revolt of 1857 because they
203) With reference to the revolt of the year 1857, who of believed that the revolt was backward looking.
the following was betrayed by a friend; captured and Objective :
put to death by the British? To know about the aftermath of revolt of 1857 like
A) Nana Sahib B) Kunwar Singh searching of the reasons how revolt of 1857 got failed.
C) Khan Bahadur Khan D) Tantia Tope
Key : D Emergence of Gandhi
Explanation : 207)Mahatma Gandhi said that some of his deepest
•. He was betrayed by Man Singh, raja of Narwar, a convictions were reflected in a book titled, “Unto the
feudatory of Scindia. last” and the book transformed his life. What was the
message from the book that transformed Mahatma
Objective :
Gandhi?
To know about the course of 1857 Sepoy Mutiny.
A) Uplifting the oppressed and poor is the moral
responsibility of an educated man
204) Which one of the following territories was not B) The good of individual is contained in the good of
affected by the revolt of 1857? al l
A) Jhansi B) Chittor C) The life of celibacy and spiritual pursuit are
C) Jagdishpur D) Lucknow essential for a noble life
Key : B D) All the above
Explanation : Key : B
•. (a), (c) and (d) were epicentres of the revolt under Explanation :
Rani Lakshmi Bai, Kunwar Singh, and Begum Hazrat •. During a long train journey in South Africa, Gandhi
Mahal respectively. was given a book of John Ruskin (Unto This Last) by
Objective : one of his friends.

Page No.73
MARCH CURRENT AFFAIRS_2019
•. About this book, he has written that it brought an 2.Interpreted the ancient Indian texts and restored
instantaneous change in his life. Gandhi derived the the self – confidence of Indians
following 3 messages from this book: 3.Stressed the need for eradication of all the social
• “The good of the individual is contained in the good evils before anything else
of all”. The concept of “Sarvodaya” and “Antyodaya” Which of the statements given above is/are correct?
were the products of this influence of Ruskin on A) 1 only B) 2 and 3 only
Gandhi. Here we note the following talisman of
Gandhi, which is inspired from the ideal of Antyodaya: C) 1 and 3 only D) 1, 2 and 3
Key : A
•. Whenever you are in doubt or when the self becomes
too much with you, apply the following test: ‘Recall Explanation :
the face of the poorest and the weakest man whom •. Dadabhai naoroji was parsi Indian political and
you may have seen and ask yourself if the step you social leader. He was the first Asian to be a member
contemplate is going to be of any use to him. Will he of British parliament. Naoroji was a prominent figure
gain anything by it? Will it restore him to a control and play a significant role in indian national
over his own life and destiny? In other words, will it movement.
lead to Swaraj for the hungry and spiritually starving •. He was the founder of, Indian national congress, along
millions?’ then you will find your doubts and yourself with A.O.hume and dinshaw wacha. His unearthing of
melting away. colonial economic exploitation is the most important
•. Thus for Gandhi, ‘Unto The Last’ would mean only the contribution to Indian national movement.
uplift of the last (Antyodaya). Ruskin’s Unto the Last •. In his book, poverty and unbritish rule in India, he
had directly or indirectly had a profound influence exposes the economic exploitation of India under
on Gandhi in adopting the ideal of Sarvodaya as his British rule. He present, Drain of wealth theory, which
life’s mission. refers to unilateral transfer of wealth from India to
Objective : Britain resulting in widespread poverty and severe
To know about the books from which mahatma Gandhi famines.
inspired a lot. •. The use of word, unbritish, in title of book refers to
208)What was the reason for Mahatma Gandhi to exploitive nature of British rule in India which was
organize a Satyagraha on behalf of the peasants of not the case in Britain, where it was progressive.
Kheda? Naoroji was an economic critic and his work on
1.The Administration did not support the land revenue economic exploitation gave a boost to Indian national
collection in spite of a drought movement and unmasked the real face of imperial
rule.
2.The Administration proposed to introduce
Permanent Settlement in Gujarat. •. Some historians use the term, economic colonialism,
for the first phase of indian national movement (1885-
Which of the statement (s) given above is/are correct? 1905) on the basis on revelations by dadabhai
A) 1 only B) 2 only naoroji.
C) Both 1 and 2 D) Neither 1 nor 2 Objective :
Key : A To know about the contribution made by different
Explanation : Indian leaders to Indian freedom struggle
•. Option 1 is correct as british administration did not
support the peasants in spite of a drought in Science & Technology
collecting land revenue. The government said that if Nuclear Technology
the taxes are not paid, the property was seized.
210) ‘HL-2M Tokamak’ is sometimes seen in the news
•. Option 2 is incorrect as no where it mentioned the recently. Consider the following statements with
introduction of permanent settlement system in respect to HL-2M Tokamak:
Kheda, Gujarat.
1.The State Atomic Energy Corporation ROSATOM of
Objective : Russia is planning to construct the artificial sun
To know about the Kheda Satyagraha device HL-2M Tokamak.
2.It is designed to replicate the nuclear fission
Freedom Struggle Begins process that occurs naturally in the sun and stars.
209) Consider the following statements: Which of the statement(s) given above is/are correct?
The Most effective contribution made by Dadabhai A) 1 only B) 2 only
Naoroji to the cause of Indian National Movement C) Both 1 and 2 D) Neither 1 nor 2
was that he Key : D
1.Exposed the economic exploitation of India by the Explanation :
British

Page No.74
MARCH CURRENT AFFAIRS_2019
•. China plans to complete the construction of the Which of the statement(s) given above is/are correct?
artificial sun device (HL-2M Tokamak ). It is designed A) 1 and 2 only B) 2 and 3 only
to replicate the nuclear fusion process that occurs C) 1 and 3 only D) 1, 2 and 3
naturally in the sun and stars.
Key : A
•. The device provides almost infinite clean energy
through controlled nuclear fusion. Explanation :
•. An agreement was signed between World Bank and
•. Its plasma is mainly composed of electrons and ions.
Government of India to provide a $250-million loan
•. The country’s existing Tokamak devices have achieved for the National Rural Economic Transformation
an electron temperature of over 100 million degrees Project (NRETP).
Celsius in its core plasma, and an ion temperature of
•. The key focus of the project is to promote women-
50 million degrees Celsius.
owned and women-led farm and non-farm enterprises
•. The new HL-2M device shall provide main technical across value chains. It will also enable them to build
support for China’s participation in the experiment businesses, access finance, markets and networks and
and operation of the International Thermonuclear generate employment.
Experimental Reactor (ITER).
•.It would support enterprise development programs for
Objective : rural poor women and youth. It creates a platform to
To know about the recent global developments in access finance including start-up financing options
nuclear fusion Tokamaks. to build their individual and/or collectively owned
and managed enterprises.
International Relations •. It is an additional financing to the $500-million
India-West Asia National Rural Livelihoods Project (NRLP) approved
211) Exercise Al Nagah III is recently seen in news, it is by the World Bank.
related to which among the following? Objective :
A) A joint naval exercise between India & UAE To know about the details of National Rural Economic
B) A joint military exercise between India & UAE Transformation Project (NRETP) scheme
C) A joint military exercise between India & Oman 213) Consider the following statements with respect to
Transport and Marketing Assistance (TMA):
D) A joint naval exercise between India & Oman
1. It aims to provide assistance for the international
Key : C
component of freight and marketing of agricultural
Explanation : produce.
•. Exercise Al Nagah III is a bilateral joint exercise 2. The scheme is applicable to all Agriculture
between India and Oman. It is scheduled at Jabel Al products.
Akhdar Mountains in Oman
3. The scheme shall be admissible for exports made
•. It aims to enhance interoperability in counter-terrorist through EDI (Electronic Data Interchange) ports only.
operations in semi urban mountainous terrain.
4. It was launched by the Ministry of Commerce and
•. It is the third in the series, while the first 2 joint Industry.
exercises were held in Oman in January 2015 and
Which of the statement(s) given above is/are correct?
India in March 2017 respectively.
A) 1,2and 3 only B) 2,3 and 4 only
Objective :
C) 1,3 and 4 only D) 1,2,3 and4
To know about bilateral military exercise Al Nagah
Key : C
Explanation :
Governance
•. The Centre has notified a scheme for Transport and
Government policies and
Marketing Assistance (TMA) for specified agriculture
programs products that will provide assistance for the
212) Consider the following statements with respect to international component of freight and marketing of
‘National Rural Economic Transformation Project agricultural produce. The scheme will be available
(NRETP)’: for exports effected from March 1, 2019 to March 31,
1. It is a new rural Livelihood Scheme under the 2020.
Deendayal Antyodaya Yojana – National Rural •. Eligibility: All exporters, duly registered with relevant
Livelihoods Mission (DAY-NRLM). Export Promotion Council as per Foreign Trade Policy,
2. The key focus of the project is to promote women- of eligible agriculture products shall be covered
owned and women-led farm and non-farm enterprises under this scheme.
across value chains. •. Exceptions: The assistance is available for most
3. The project would be undertaken with loan agricultural product exports with some exceptions
assistance from the New Development Bank. such as live animals, products of animal origin, milk,

Page No.75
MARCH CURRENT AFFAIRS_2019
cream, curd, butter, buttermilk, whey, rice, wheat, •. In case of death before the age of 60 years: If a
tobacco and garlic. beneficiary has given regular contribution and dies
•. The assistance, at notified rates, will be available for before attaining the age of 60 years, his or her spouse
export of eligible agriculture products to the will be entitled to continue the scheme subsequently
permissible countries, as specified from time to time. by payment of regular contribution or may even exit
The assistance shall be admissible only if payments the scheme.
for the exports are received in Free Foreign Exchange Contribution to the scheme:
through normal banking channels. •. Contribution by the Subscriber: The subscriber is
required to contribute the prescribed contribution
•. The scheme shall be admissible for exports made amount from the age of joining the scheme till the age
through EDI (Electronic Data Interchange) ports only. of 60 years.
•. The scheme covers freight and marketing assistance •. Medium of contribution: The subscriber can
for export by air as well as by sea contribute to the PM-SYM through ‘auto-debit’ facility
•. It was launched by the Ministry of Commerce and from his or her savings bank account or from his or
Industry.. her Jan- Dhan account.
Objective : •. Equal contribution by the Central Government: Under
the PM-SYM, the prescribed age-specific contribution
To know about the recent scheme Transport and
by the beneficiary and the matching contribution by
Marketing Assistance (TMA) for specified agriculture
the Central Government will be made on a ‘50:50
products
basis’.
Objective :
214) Consider the following statements regarding recently
To know about the details of Pradhan Mantri Shram
launched Pradhan Mantri Shram Yogi Maan-dhan
Yogi Maan-dhan (PM-SYM) Yojana
(PM-SYM) Yojana:
1. The scheme is open for all government employees.
Governance and Good
2. Central government will contribute an equal amount
irrespective of the entry age of subscriber. Governance
Which of the statements given above is/are correct? 215) Which among the following publishes the ‘Corruption
Perception Index’?
A) 1 only B) 2 only
A) Financial Action Task Force
C) Both 1 and 2 D) Neither 1 nor 2
B) NITI Aayog
Key : B
C) Transparency International
Explanation :
D) Amnesty International
•. Prime Minister Narendra Modi recently launched a
pension scheme called Pradhan Mantri Shram-Yogi Key : C
Maandhan Yojana (PM-SYM). The scheme was Explanation :
announced in the Union Budget in February this year •. It is a composite index released by Transparency
•. PM-SYM is a voluntary and contributory pension International that draws from 12 surveys to rank
scheme that will engage as many as 42 crore workers nations around the globe. It has become a benchmark
in the unorganised sector. gauge of perceptions of corruption and is used by
Eligibility: analysts and investors.
•. The unorganised sector workers, with income of less •. The index is also based on expert opinions of public
than Rs 15,000 per month and who belong to the entry sector corruption and takes note of range of factors
age group of 18-40 years, will be eligible for the like whether governmental leaders are held to account
scheme. or go unpunished for corruption, the perceived
prevalence of bribery, and whether public institutions
•. Those workers should not be covered under New respond to citizens’ needs.
Pension Scheme (NPS), Employees’ State Insurance
Corporation (ESIC) scheme or Employees’ Provident Objective :
Fund Organisation (EPFO). To know about the Corruption Perception Index
•. He or she should not be an income tax payer.
•. Minimum Assured Pension: Each subscriber under the Environment_Ecology
scheme will receive minimum assured pension of Rs Environmental Pollution and
3000 per month after attaining the age of 60 years. Degradation
•. In case of death during receipt of pension: If the 216) The terms ‘SWAS, SAFAL and STAR’, is related to which
subscriber dies during the receipt of pension, his or among the following?
her spouse will be entitled to receive 50 percent of A) Financial Inclusion
the pension as family pension. This family pension is
applicable only to spouse. B) Non-Performing assets

Page No.76
MARCH CURRENT AFFAIRS_2019
C) Green Crackers A) 1 and 2 only B) 2 and 3 only
D) Cybercrime C) 1 and 3 only D) 1, 2 and 3
Key : C Key : C
Explanation : Explanation :
•. The Council of Scientific and Industrial Research •. Prime Minister Narendra Modi launched a National
(CSIR) scientists has developed less polluting green Common Mobility Card (NCMC), which can be used to
firecrackers named SWAS,SAFAL,STAR. Green crackers make payments across public transport anywhere in
are so named because they do not contain harmful the country, including Metro, bus and suburban
chemicals that would cause air pollution. railways, for toll and parking, and for retail like any
•. The crackers have been named as Safe Water Releaser credit or debit card.
(SWAS), Safe Thermite Cracker (STAR) and Safe Minimal •. Referred to as “One Nation One Card”, this inter-
Aluminium (SAFAL) operable transport card would allow the holders to
Objective : pay for their bus travel, toll taxes, parking charges,
To know about green crackers retail shopping and even withdraw money.
2019-03-06 •. It runs as on RuPay card and automates the fare
collection system. This eliminates the need to carry
Economy change and cash while travelling in the metro, bus or
External Sector train, or for toll and parking.
217) A trade programme ‘Generalized System of •. The gate and reader prototype for the NCMC has been
Preferences (GSP)’ is sometimes seen in the news developed by government-owned Bharat Electronics
recently with its contentious issues with Indian Limited (BEL) in compliance with the Make in India
exports. The Trade programme is related to which of initiative.
the following country? •. These cards would be issued by all public and private
A) China B) USA banks the same way credit, debit, and prepaid cards
C) Pakistan D) Maldives are issued.
Key : B •. The first Indigenously Developed Payment Eco-system
Explanation : for transport consists of”: NCMC Card, SWEEKAR
•. The Generalized System of Preferences (GSP) is a U.S. (Swachalit Kiraya: Automatic Fare Collection System
trade program designed to promote economic growth (AFC)) and SWAGAT (Swachalit Gate). AFC System (gates,
in the developing world by providing preferential readers/validators, backend infrastructure etc.) is the
duty-free entry for up to 4,800 products from 129 core of any transit operator to automate the fare
designated beneficiary countries and territories. collection process.
•. USA has decided to withdraw India’s name from Objective :
Generalized System of Preferences or GSP list. India To know about the details of ‘National Common
exported an estimated $5.6 billion worth of goods to Mobility Card (NCMC)’
the US under this program in 2017.
•. The reason behind this move is that India has not Monetary Policy
assured the United States that it will provide equitable 219) “Bank of Baroda Announces SWAP ratio for merger
and reasonable access to its markets. with Dena Bank, Vijaya Bank” in this Context SWAP
Objective : ratio means:
To know about the recent trade war between India A) Exchanging of Shares between acquiring Company
and USA; And the details about Generalized System of and target Company
Preferences (GSP) B) Exchanging of Profits between acquiring Company
Infrastructure and Target Company
218)Consider the following statements with respect to C) Exchanging of risk between acquiring Company and
the ‘National Common Mobility Card (NCMC)’: Target Company
1. The first Indigenously Developed Payment Eco- D) Exchanging of long term debts between acquiring
system for transport consists of NCMC Card, SWEEKAR Company and Target Company
and SWAGAT. Key : A
2. SWAGAT is an Automatic Fare Collection System Explanation :
which is the core of any transit operator to automate
•. Swap ratio is an exchange ratio used in case of
the fare collection process.
mergers and acquisitions. It is the ratio in which the
3. These cards would be issued by all public and acquiring company offers its own shares in exchange
private banks the same way credit, debit, and prepaid for the target company’s shares. To calculate the swap
cards are issued. ratio, companies analyze financial ratios such as
Which of the statement(s) given above is/are correct? book value, earnings per share, profits after tax as

Page No.77
MARCH CURRENT AFFAIRS_2019
well as other factors, such as size of company, long- Internal Security
term debts, and strategic reasons for the merger or Security challenges and their
acquisition and so on. management in border areas
•. For example, if company A is acquiring company B 221) Consider the following statements with respect to
and offers a swap ratio of 1:5, it will issue one share BOLD-QIT Project:
of its own company (company A) for every 5 shares of
the company B being acquired. In other words, if 1. It is a boundary monitoring system that installs
company B has 10 crore outstanding equity shares technical systems under the Comprehensive
and 100% of it is being acquired by company A, and Integrated Border Management System (CIBMS)
then company A will issue 2 crore new equity shares programme.
of company A to the shareholders of company B, 2. It helps Border Security Forces (BSF) to monitor
proportionately. Indo-Pakistan borders.
Objective : Which of the statement(s) given above is/are
correct?
To know about the SWAP Ratio for amalgamation of
Banks or any Companies. A) 1 only B) 2 only
C) Both 1 and 2 D) Neither 1 nor 2
Science & Technology Key : A
Space Explanation :
220) Consider the following statements with respect to •. BOLD-QIT (Border Electronically Dominated QRT
“Yuva Vigyani Karyakram (YUVIKA)”: Interception Technique) is a boundary monitoring
1. It is primarily aimed at imparting basic knowledge system that installs technical systems under the
on Space Technology, Space Science and Space CIBMS (Comprehensive Integrated Border
Applications to the students in order to develop their Management System) programme.
interest in the emerging areas of Space activities. •. It was scheduled to be established on India-
Bangladesh border in Dhubri district of Assam.
2. The programme was launched by Ministry of Human
Since, Border Security Forces (BSF) is responsible for
Resource Development in order to scientific temper
safeguarding 4,096 km long International Border
in the county.
with Bangladesh, it enables BSF to equip Indo-Bangla
Which of the statement(s) given above is/are correct? borders with different kinds of sensors in the
A) 1 only B) 2 only unfenced riverine area of Brahmaputra and its
C) Both 1 and 2 D) Neither 1 nor 2 tributaries.
Key : A •. It is built on the basis of security on the border
Explanation : similar to that of Israel.
•. Indian Space Research Organization (ISRO) has Objective :
launched a special program for School Children called To know about the details of BOLD-QIT (Border
“Young Scientist Program” or “Indian Space Research Electronically Dominated QRT Interception
Organization (ISRO) has launched a special program Technique)
for School Children called “Young Scientist Program”
or “YUva VIgyani KAryakram”. Environment_Ecology
•. It is primarily aimed at imparting basic knowledge INTERNATIONAL
on Space Technology, Space Science and Space
ENVIRONMENT
Applications to the younger in order to develop their
interest in the emerging areas of Space activities. CONVENTIONS AND
•. The students who have finished 8th standard and PROTOCOLS
currently studying in 9th standard will be eligible for 222) Consider the following statements with respect to
the program.”. World Wildlife day:
• It is primarily aimed at imparting basic knowledge 1. The day of signature of Convention on Biological
on Space Technology, Space Science and Space diversity is celebrated as World Wildlife Day.
Applications to the younger in order to develop their 2.. It aims to celebrate and raise awareness of the
interest in the emerging areas of Space activities. world’s wild animals and plants.
•. The students who have finished 8th standard and 3. This year theme aligns with goal 14 of UN
currently studying in 9th standard will be eligible for Sustainable Development Goals.
the program. Which of the statement(s) given above is/are
Objective : incorrect?
To know about the details of “YUva VIgyani KAryakram” A) 1 only B) 1 and 2 only
C) 2 and 3 only D) 1 and 3 only
Key : A

Page No.78
MARCH CURRENT AFFAIRS_2019
Explanation : originates in the Wayanad District of Kerala state by
•. The day of signature of the Convention on the confluence of the Panamaram River and the
International Trade in Endangered Species of Wild Mananthavady River. It flows eastward to join the
Fauna and Flora (CITES) is celebrated as UN World Kaveri River at Tirumakudalu Narasipura in Karnataka
Wildlife Day. Objective :
•. It aims to celebrate and raise awareness of the world’s To know about the Bandipur National Park and Tiger
wild animals and plants. Reserve.
•. The theme for 2019 is “Life below water: for people International Relations
and planet”. International Institutions,
•. The theme aligns with goal 14 of UN Sustainable agencies and their structure
Development Goals. and mandate
•. This is the first World Wildlife Day to focus on life 224) Which of the following is/are correct with regards
below water. to Geneva Convention?
•. The oceans have an estimated market value of $3 1.The Geneva Convention defines the standard of
trillion, which makes up nearly 5% of world’s gross international law for humanitarian treatment in war.
domestic product.
2.The Fourth Geneva Convention is associated to the
•. Nearly 40% of global livelihoods depend on the oceans treatment of Prisoners of War (PoWs).
for sustenance and craftwork as well as for their
Select the correct answer using the code given below:
cultural and spiritual value.
Objective : A) 1 only B) 2 only
C) Both 1 and 2 D) Neither 1 nor 2
To know about the details of World Wildlife Day
Key : A
Explanation :
Environment & Ecology
•. The Geneva Conventions of 1949 comprise four treaties
223) Consider the following statements regarding
and three additional protocols that define the
“Bandipur Tiger Reserve and National Park”:
standards of international law for humanitarian
1.The Bandipur tiger reserve was established under treatment in war. The convention mandates the parties
project tiger in 1973. to the war to remain humane to non-combatants such
2.The National Park forms part of Nilgiri Biosphere as civilians and medical personnel, as well as to
Reserve. combatants who are no longer participating in
3.The Bandipur National Park is surrounded by the hostilities, such as prisoners of war, or wounded in
river Kabini in the north which is the tributary of river action. A major part of international humanitarian
Krishna. law is contained in the first four Geneva Conventions.
Which of the above statement (s) is/are correct using • First Geneva Convention: This deals with improving
the code given below the condition of the wounded and sick soldiers in the
A) 1 and 2 only B) 2 and 3 only field during the war. This convention contains 64
C) 1 and 3 only D) 1, 2 and 3 articles.
Key : A • Second Geneva Convention: This protects wounded
and sick combatants while on board ship or at sea
Explanation : during the war. This convention contains 63 articles.
•. Bandipur Tiger Reserve and National Park spread over
• Third Geneva Convention: This is associated to the
an area of about 874.2 sq km is situated in the
treatment of Prisoners of War (POWs). It states that
Chamarajanagar District of Karnataka. The National prisoners of war (POWs) must be treated humanely,
Park forms part of Nilgiri Biosphere Reserve. provided adequate houses, receive sufficient food,
•. Originally the Maharaja of Mysore created a clothing and medical care. This convention contains
sanctuary of 90 km2 (35 sq mi) in 1931 and named it 143 articles.
the Venugopala Wildlife Park. Later in the year 1973 • Fourth Geneva Convention: This is related to the
Came into existence with an addition of nearly 800 protection of civilians in areas of armed conflict
km2 (310 sq mi) to the Venugopala Wildlife Park. during the time of war. It contains 159 articles.
•. The Bandipur National Park is surrounded by the river
Objective :
Kabini in the north, River Moyar in the south and the
River Nugu runs through the park. The Gopalaswamy To know about the Geneva Convention.
Betta which is a Hindu pilgrimage centre and it also
forms part of the National Park. the park has a variety
of biomes including dry deciduous forests, moist
deciduous forests and shrublands.
•. The Kabini, also called Kapila, is one of the major
tributaries of the river Cauvery in southern India. It

Page No.79
MARCH CURRENT AFFAIRS_2019
Geography •. The Security Council consists of fifteen members.
Map & current affairs based Russia, the United Kingdom, France, China, and the
questions United States—serve as the body’s five permanent
members. These permanent members can veto any
225) INCOIS abbreviates for Indian National Centre for substantive Security Council resolution, including
Ocean Information Services is an autonomous those on the admission of new member states or
organization of Government of India, under the candidates for Secretary-General.
Ministry of:
•. The Security Council also has 10 non-permanent
A) Ministry of Defence members, elected on a regional basis to serve two-
B) Ministry of Earth Sciences year terms. The body’s presidency rotates monthly
C) Ministry of science and Technology among its members.
D) Ministry of information and Broadcasting Objective :
Key : B To know about the United Nations Security Council.
Explanation :
•. Indian National Center for Ocean Information Services 2019-03-05
(INCOIS) is an autonomous organization of the Economy
Government of India, under the Ministry of Earth
Monetary Policy
Sciences, located in Pragathi Nagar, Hyderabad. ESSO-
INCOIS was established as an autonomous body in 227) As per the SARFAESI Act which of the following
1999 under the Ministry of Earth Sciences (MoES) and measures can be undertaken by ARCs for asset
is a unit of the Earth System Science Organization reconstruction?
(ESSO). ESSO- INCOIS is mandated to provide the best 1.Taking over or changing the management of the
possible ocean information and advisory services to business of the borrower,
society, industry, government agencies and the 2.The sale or lease of the business of the borrower
scientific community through sustained ocean 3.Restructuring or rescheduling of debt.
observations and constant improvements through Select the correct answer using the code below:
systematic and focussed research.
A) 2 only B) 1 and 2 only
Objective :
C) 1,2 and 3 D) None of the above
To know about the INCOIS.
Key : C
Explanation :
Governance
•. SARFAESI Act stipulates various measures that can be
Government policies and undertaken by ARCs for asset reconstruction. These
programs include:
226) Select the incorrect from the following statements a) taking over or changing the management of the
with regards to United Nations Security Council business of the borrower,
(UNSC): b) the sale or lease of the business of the borrower
A) The United Nations Security Council (UNSC) is one c) entering into settlements and
of the 6 principal organs of the UN and is charged
d) restructuring or rescheduling of debt.
with the maintenance of International Peace and
security e) enforcement of security interest
B) The Security Council has fifteen members, five are The last step of ‘enforcement of security interest’
permanent and ten are non permanent. means ARCs can take possession/sell/lease the
supported asset like land, building etc.
C) India is a Permanent member of UNSC.
Objective :
D) None of the above
Key : C To know about the Asset Reconstruction Company
228) Which among the following refers to the financial
Explanation :
intermediaries involved in facilitating the creation
•. The United Nations Security Council (UNSC) is one of of credit across the global financial system but whose
the six principal organs of the United Nations and is members are not subject to regulatory oversight?
charged with the maintenance of international peace
A) Commercial banking B) Narrow banking
and security.
•. Its powers include the establishment of peacekeeping C) Shadow banking D) Virtual bank
operations, the establishment of international Key : C
sanctions, and the authorization of military action Explanation :
through Security Council resolutions; it is the only •. Shadow banking is that part of the financial system
UN body with the authority to issue binding where ‘credit intermediation involving entities and
resolutions to member states. activities remains outside the regular banking system’.

Page No.80
MARCH CURRENT AFFAIRS_2019
•. Shadow banks are those institutions that do not Minimum Support Price (MSP) and development of
collect deposits but still provide loans. value chain for MFP’ was planned as a social safety
•. These include a variety of institutions, ranging from initiative for MFP gatherers.
trusts, investment funds etc. •. The MSP for MFP scheme was first launched in 2013
Objective : but the severe gaps in its implementation led to the
to know about the concept of Shadow banking programme remaining dormant. To revitalize the
programme the revised guidelines have been issued.
The revised guidelines:
229) Merchant Discount Rate is a fee charged from a
Provide for a revised MSP in the range of 30-40% on
merchant by a bank for accepting payments from
average.
customers through credit and debit cards in their
establishments. Who among the following will be The procurement of these MFPs will commence in haat
compensated by the Merchant Discount Rate? bazars, where tribals bring their produce with the
facilitation provided by state government agencies
1. Bank that issued the credit/debit card
and district collectors.
2. Bank that puts up swiping machine
Lay down a decentralized implementation framework.
3. Payment gateways Objective : To know about Minimum Support Price (MSP)
Select the correct answer using the code below: for Minor Forest Produce (MFP) scheme
A) 1 only B) 2 only
C) 2 and 3 only D) 1,2 and 3 International Relations
Key : D International Institutions,
Explanation : agencies and their
•. MDR is a fee charged from a merchant by a bank for structure and mandate
accepting payments from customers through credit
231) With regard to United Nations Security Council,
and debit cards in their establishments.
consider the following:
•. MDR compensates the card issuing bank, the lender
1. UNSC can authorize military action
which puts the PoS terminal and payment gateways
such as Mastercard or Visa for their services. 2. India is one among the Coffee club nations bidding
for UNSC Permenent membership
Objective :
3. The non permanent members of UNSC are elected
To know about the Merchant Discount Rate
by the permanent members.
Select the correct answer using the code below:
Agriculture
A) 1 only B) 1 and 2 only
230) Consider the following statements Minimum Support
C) 2 and 3 only D) 1,2 and 3
Price (MSP) for Minor Forest Produce (MFP) scheme:
Key : A
1.MSP for MFP scheme is a central sector scheme
aimed to ensure fair and remunerative price to MFP Explanation :
gatherers. •. Statement 2 is incorrect because, India is one among
2. The scheme is applicable in the States which have the G4 nations bidding for UNSC Permenent
atleast 15% of tribal population. membership.
3. Ministry of Tribal Affairs is the Nodal Ministry for •. Uniting for Consensus is a movement nicknamed as
implementation of the scheme. Coffee Club is a group of nations which counter the
bids for permanent seats by countries such as Brazil,
Which of the above statements is/are incorrect?
Germany, India, and Japan or G4.
A) 1 and 2 only B) 2 and 3 only •. Statement 3 is incorrect because, non permanent
C) 1 and 3 only D) 1, 2 and 3 members of UNSC are elected by the United Nations
Key : A General Assembly.
Explanation : Objective :
•. The Union Tribal Affairs Minister launched the revised To know about the United Nations Security Council
MSP for MFP for the welfare of Tribals at the National
Workshop organized by TRIFED under Ministry of Geography
Tribal Affairs.
World Geography
•. Minimum Support Price Scheme (MSP) for Minor Forest
Produce (MFP) is a centrally-sponsored scheme aimed 232) Which among the following nations is/are not
to ensure fair and remunerative price to MFP bordered with Mediterranean Sea?
gatherers. 1. Tunisia
•. The scheme is officially known as ‘Mechanism for 2. Morocco
marketing of Minor Forest Produce (MFP) through 3. Nigeria

Page No.81
MARCH CURRENT AFFAIRS_2019
4. Chad D) It is the gram sabha that passes initial resolution
Select the correct answer using the code below: to recognize the rights of claimants.
A) 3 only B) 3 and 4 only Key : A
C) 1,3 and 4 only D) 1,2,3 and 4 Explanation :
Key : B •. Statement A is incorrect because, FRA, 2006 grants
Explanation : rights only to eligible forest dwellers.
•. There are 21 countries which border the Eligibility:
Mediterranean Sea. They are Spain, France, Monaco, •. Eligibility to get rights under the Act is confined to
Italy, Malta, Slovenia, Croatia, Bosnia and those who “primarily reside in forests” and who
Herzegovina, Montenegro, Albania, Greece, Turkey, depend on forests and forest land for a livelihood.
Cyprus, Syria, Lebanon, Israel, Egypt, Libya, Tunisia, Further, either the claimant must be a member of the
Algeria and Morocco. Scheduled Tribes scheduled in that area or must have
Objective : been residing in the forest for 75 years.
To know the geographical features of various countries Objective :
that are in news To know the features of Forest Rights Act, 2006

Map & current affairs based Biodiversity


questions 235) Consider the following statements with respect to
233) Consider the following statements: International Coral Reef Initiative (ICRI)
1. The International Border is the official border 1. It is an informal partnership among governments,
between India and Pakistan, whereas Line of Control International Organisations, and NGO’s and its
(LOC) is the military control line between India and decisions are binding on members.
POK 2. India is a member country to ICRI.
2. The Indian controlled Jammu and Kashmir shares Which of the statement(s) given above is/are correct?
boundary with the International Border A) 1 only B) 2 only
Select the correct answer using the code below: C) Both 1 and 2 D) Neither 1 nor 2
A) 1 only B) 2 only Key : B
C) Both 1 and 2 D) Neither 1 nor 2 Explanation :
Key : C •. The International Coral Reef Initiative (ICRI) is an
informal partnership between Nations and
Explanation : organizations which strives to preserve coral reefs
and related ecosystems around the world.
•. The India–Pakistan Border, known locally as the
International Border (IB), is an international border •. ICRI’s decisions are not binding on its members.
running between India and Pakistan that demarcates •. It was founded in 1994 by eight governments:
the Indian states and the Pakistani four provinces. Australia, France, Japan, Jamaica, the Philippines,
•. The border runs from the Line of Control (LoC), which Sweden, the United Kingdom, and the United States of
separates the Indian state of Jammu and Kashmir from America.
Pakistani-controlled Kashmir, in the north, to the Zero •. ICRI now counts more than 60 members including
Point between the Indian state of Gujarat and the Sindh India (Ministry of Environment and Forests)
province of Pakistan, in the south. Objective :
Objective : To know about the International Coral Reef Initiative
To know the difference between Line of Control (LOC) (ICRI).
and the International Border (IB) Science & Technology
Computer, IT and Tele
EnvironmentEcology Communications
CONSERVATION EFFORTS 236) Consider the following statements with respect to
234) With regard to Forest Rights Act, 2006 which of the ‘Smart India Hackathon 2019’:
following is not correct? 1. Smart India Hackathon 2019 is an initiative by
A) It grants legal recognition to the rights of all forest Ministry of Science & Technology.
dwellers 2. In this initiative, Technology Students across India
B) The claimants will be provided with ownership to can compete to creatively solve problems and offer
land being cultivated by the tribals technical solutions.
C) It includes the forest management rights

Page No.82
MARCH CURRENT AFFAIRS_2019
3. In SIH 2019, the students would also have the
opportunity to work on challenges faced within the 238) Recently Ministry of Commerce and Industry has
private sector organisations. revised its export policy to permit the export of red
Which of the statement(s) given above is/are correct? sanders if it is obtained from cultivated land. Red
A) 1 and 2 only B) 2 and 3 only sanders was classified under which of the following
C) 1 and 3 only D) 1, 2 and 3 category of IUCN?
Key : B A) Endangered B) Near threatened
Explanation : C) Critically endangered D) Least concerned
•. Smart India Hackathon 2019 is a nationwide initiative Key : B
to provide students a platform to solve some of Explanation :
pressing problems we face in our daily lives, and thus •. The International Union for Conservation of Nature
inculcate a culture of product innovation and a (IUCN) has now reclassified red sander as ‘near
mindset of problem solving. In SIH 2019, the students threatened’ from the earlier ‘endangered’.
would also have the opportunity to work on challenges •. After being classified as ‘endangered’ in 1997 and
faced within the private sector organisations and added to the Red List, this is the first time that red
create world class solutions for some of the top sanders have been shifted to a better-conserved
companies in the world, thus helping the Private category.
sector hire the best minds from across the nation. •. Red sanders wood fetches huge prices in the
•. SIH 2019 IS an initiative by Ministry of HRD, AICTE, international market for its use in making luxury
Persistent Systems, i4c and Rambhau Mhalgi products, musical instruments and medicine.
Prabodhini •. As the species is endemic to India and had been
•. It is World’s biggest Software and Hardware hackathon considered near threatened, the Foreign Trade Policy
and involves 1 Lakh+ technical students, 3000+ of India doesn’t allow its export. However recently
technical institutions, 200+ organizations from The Directorate General of Foreign Trade (DGFT), an
across India. agency of the Ministry of Commerce and Industry has
Objective : revised its export policy to permit the export of red
To know about the details of Smart India Hackathon sanders if it is obtained from cultivated land.
2019 Objective :
To know about government’s export policy on near
Governance threatened Red Sanders.
Government policies and 2019-03-04
programs History Modern
237)Consider the following statements with respect to The British Conquest of India
National anti doping Agency (NADA):
1. It was set up with a mandate for Dope free sports in 239) Which one among the following was not an outcome
India. of the battle of plassey in 1757?
2. It was set up as a registered society under the A) The Mughals were cut to size
Societies Registration Act of 1890. B) The battle paved the way for British rule in India
Which of the statement(s) given above is/are correct? C) The battle enabled the British to use the revenue of
A) 1 only B) 2 only Bengal to raise a strong army
C) Both 1 and 2 D) Neither 1 nor 2 D) None of the above
Key : C Key : A
Explanation : Explanation :
•. It was set up as a registered society under the Societies •. Battle of plassey was not thrown the effect like
Registration Act of 1890 on 2005. Mughals were cut to size. Hence statement (a) is
•. It was set up with a mandate for Dope free sports in wrong.
India. •. Remaining statements (b) and (c) are correct with
The primary objectives are: regards to raise of british rule politically and
To implement anti-doping rules as per WADA code economically.
Regulate dope control program Objective :
To promote education and research and creating To know about the Effects of the Battle of Plasseyv
awareness about doping and its ill effect
Objective : 240) Which of the following is/are the causes of the First
To know about National anti doping Agency (NADA) Anglo – Mysore War (1767 – 1769)?

Page No.83
MARCH CURRENT AFFAIRS_2019
1. Hyder ali built a strong and modern army and of 1773, where the government only sought to
trained them along European lines ‘regulate’ matters and not take over.
2.He took support from French in training his army 5. This act established the British Crown’s authority in
Select the correct answer using the code given below civil and military administration of its Indian
A) 1 only B) 2 only territories. Commercial activities were still a
monopoly of the Company.
C) Both 1 and 2 D) Neither 1 nor 2
Objective :
Key : C
To know about the Pitts India effect and its importance
Explanation :
and the reasons to provocation of this act.
1. He set up a modern army and trained them along
European lines.
Socio-Religious Reform
2. Hyder Ali built a strong army and annexed many
regions in the South including Bidnur, Canara, Sera, Movements
Malabar and Sunda. 242) Match the following List I with List II
3. He also took French support in training his army. (List I – outbreak or revolt) (List II – place)
4. This alarmed the British. A. Moppila outbreak 1. Maharashtra
Objective : B. Ramosi peasant force 2. Bengal
To know about the Causes, course and effects of any C. Kuka revolt 3. Malabar
war. D. Pabna revolt 4. Punjab
Code:
241)Consider the following with respect to Pitt’s India A BCD ABCD
Act, 1784: A) 3 1 4 2 B) 3 4 1 2
1.The Pitt’s India Act, 1784 also called the East India C) 2 4 1 3 D) 2 1 4 3
Company Act. Key : A
2.It was passed by the British Parliament to correct Explanation :
the defects of the Regulating Act of 1773.
•. The Malabar rebellion (also known as the Moplah
3.This act resulted in dual control of British rebellion and M?ppila Laha?a in Malayalam) was an
possessions in India by the British government and armed uprising in 1921 against British authority in
the Company with the final authority resting with the the Malabar region of Southern India by Mappilas
government. and the culmination of a series of Mappila revolts
4. This effect continued in effect till 1858. that recurred throughout the 19th century and early
Which of the above statement (s) is/are correct using 20th century. The 1921 rebellion began as a reaction
the code given below against a heavy-handed crackdown on the Khilafat
A) 1, 2 and 4 only B) 1, 2 and 3 only Movement, a campaign in defense of the Ottoman
C) 3 and 4 only D) 1, 2, 3 and 4 Caliphate,[3] by the British authorities in the Eranad
and Valluvanad taluks of Malabar.
Key : D
Ramosis peasant force:
Explanation :
•. Reason: Ramosis of Maharashtra were the inferior
•. The Pitt’s India Act, 1784 also called the East India
ranks of police in Maratha administration. After the
Company Act, 1784 was passed by the British
fall of the Maratha kingdom, they became farmers
Parliament to correct the defects of the Regulating
=>heavy land Revenue demands by British.
Act of 1773. This act resulted in dual control of British
possessions in India by the British government and Area: Satara, Maharashtra, Deccan
the Company with the final authority resting with the Leader: Chittur Singh (1822), Vasudev Balwant Phadke
government. This act continued in effect till 1858. (1877-87)
Hence above all statements are correct. Result: Government gave them land grants and
Features of this act: recruited them as hill police.
1. This act made a distinction between the commercial •. There has been a doubtful history about the initial
and political activities of the East India Company. days of Kuka Movement. There are two names
associated with the start of this movement i.e. Baba
2. For the first time, the term ‘British possessions in Balak Singh and Bhagat Jawar Mal.
India’ was used.
•. Balak Singh was born in village Sarvala, in District
3. This act gave the British government direct control Attock, in 1799. He started preaching very early in his
over Indian administration. life and the objective of his preachings was to uphold
4. The Company became subordinate to the British the religious purity of Sikhism.
government unlike as in the previous Regulating Act
•. Pabna Peasant Uprising (1873–76) was a resistance
movement by the peasants (“Ryots”) against the lords

Page No.84
MARCH CURRENT AFFAIRS_2019
of the lands in Bengal (“zamindars”) in the Yusufshahi Governance
pargana (now the Sirajganj District, Bangladesh) in Government policies and
Pabna.It was led by Keshab Chandra Roy. programs
Objective : 246) Which among the following is/are components of
To know about the various revolts or movements and “National Policy on Software Products - 2019”?
places associated with them 1. Generation of employment opportunities for 3.5
million people by 2025.
The Revolt of 1857 2. Setting up of National Software Products Mission
243) What was/were the object/objects of Queen to evolve and monitor the implementation of the
Victoria’s proclamation (1858)? policy.
1.To disclaim any intention to annex Indian states 3. To increase India’s share of the Global Software product
2.To place the Indian administration under the British market by ten times by 2025.
Crown Select the correct answer using the code given below:
3.To regulate East India Company’s trade with India A) 1 and 2 only B) 1 and 3 only
Select the correct answer using the code given below C) 2 and 3 only D) 1, 2 and 3
A) 1 and 2 only B) 2 and 3 only Key : D
C) 1 and 3 only D) 1, 2 and 3 Explanation :
Key : A •. Components and objectives of the Policy
Explanation : a) An outlay of Rs.1500 Crore is involved to implement
•. Option 3 is totally irrelevant. Queen V ictoria the schemes.
proclamation reversed the policy of annexation after b) Rs1500 Crore is divided into Software Product
the revolt of 1857, so option 1 is true. Development Fund (SPDF) and Research & Innovation
•. Option 2 is definitely true as the Indian states were fund.
henceforth to recognise the paramountcy of the British c) Software Product Development Fund will participate
crown and were to be treated as parts of single charge. in venture funds having objectives aligned to this
Objective : policy
To know about the Queen Victoria’s Proclamation d) The fund can leverage private investments targeted to
1858. promote software product ecosystem.
e) The SPDF will be financially managed by a professional
244) The ruler of which one of the following states was financial institution.
removed from power by the British on the pretext of f) To promote the creation of a sustainable Indian
mis governance? software product industry, driven by intellectual
A) Awadh B) Jhansi property (IP)
C) Oudh D) Satara g) A ten-fold increase in India share of the Global Software
product market by 2025.
Key : A
h) Nurture 10,000 technology startups in software product
Explanation : industry
•. Jhansi, Oudh and Satara were annexed via the
i) Generate employment for 3.5 million people by 2025.
Doctrine of Lapse.
j) Create a talent pool for software product industry
Objective : through up-skilling of 1 lakh IT professionals and
To know about the annexation and removing of princely generating 10000 specialized professionals that can
states on the grounds of misgovernance by the British. provide leadership.
245) Who was the governor general of India during the k) Build a cluster-based innovation driven ecosystem by
Sepoy Mutiny? developing software product development clusters.
A) Lord Canning B) Lord Dalhousie l) National Software Products Mission will be set up to
C) Lord Hardinge D) Lord Lytton evolve and monitor the implementation of this policy.
Key : A Objective :
Explanation : To know about the steps taken by the government to
•. Lord Canning was the Governor General from 1856 to improve India’s position in Global Software Industry.
1860 and became the first viceroy of India in 1858.
Objective : 247) Which of the following is/are correct with regards
To know about the different Governor’s, Governor to Van Dhan Scheme?
Generals and Viceroy during British rule in India. 1.Van Dhan Scheme aims at economic development of
tribal’s involved in collection of Minor Food produces
(MFPs) by helping them in optimum utilization of

Page No.85
MARCH CURRENT AFFAIRS_2019
natural resources and provide them sustainable D) None of the above.
livelihood. Key : A
2.It will be implemented through Ministry of Tribal Explanation :
Affairs and Tribal Cooperative Marketing Development What is Compensatory Afforestation?
Federation of India (TRIFED).
•. Compensatory afforestation means afforestation
3.To create infrastructure and providing enabling done in lieu of diversion of forest land for non-forest
environment for undertaking value addition of use.
systematic scientific lines, the central government
What does CAF Bill seek?
only provides necessary support not the states.
•. The Compensatory Afforestation Fund Bill seeks to
Select the correct answer using the code given below
govern how forests will be raised, cut and
A) 1 and 2 only B) 2 and 3 only resurrected across India.
C) 1 and 3 only D) 1, 2 and 3 Background (Why we need legislation)?
Key : A • The Forest (Conservation) Act, 1980 (FCA) governs
Explanation : diversion or use of forest land for non-forest
•. Prime Minister Narendra Modi launched Van Dhan purposes such as industrial or infrastructure
Scheme of Ministry of Tribal Affairs and Tribal projects.
Cooperative Marketing Development Federation of • A company diverting forest land must provide
India (TRIFED) during the celebrations of Ambedkar alternative land for taking up compensatory
Jayanti (April 14) at Bijapur, Chattisgarh. PM also afforestation.
inaugurated first model Van Dhan Vikas Kendra under • For the afforestation purpose, the company should
the scheme in Bijapur. pay for planting new trees in the alternative land
Van Dhan Scheme provided to the state. The loss of forest ecosystem
•. The scheme aims at economic development of tribals must also be compensated by paying.
involved in collection of Minor Food Produces (MFPs) • In 2002, the Supreme Court of India observed that
by helping them in optimum utilization of natural collected funds for afforestation were underutilised
resources and provide them sustainable livelihood. by the states and it ordered for centrally pooling of
Under it, 10 Self Help Groups (SHGs) of 30 Tribal funds under ad hoc Compensatory Afforestation
gatherers will be constituted. Fund.
•. The SHGs will then be trained and provided with • The court had set up the ad hoc National
working capital to add value to products they collect Compensatory Afforestation Fund Management and
from forest. They will be able to market their products, Planning Authority (National CAMPA) to manage the
by working under leadership of collector, not only Fund.
within states but also outside states. • In 2009, states also had set up State CAMPAs that
•. TRIFED will be provide all required training and receive 10% of funds form National CAMPA to use for
technical support to SHGs. They will be trained on afforestation and forest conservation.
sustainable harvesting, collection, primary • However, in 2013, a CAG report identified that the
processing and value addition. They will be formed funds continued to be underutilised.
into clusters to aggregate their stock in tradable
quantity and link them with facility of primary What are the salient features of the passed bill?
processing in Van Dhan Vikas Kendra • It seeks to establish the National Compensatory
Afforestation Fund under the Public Account of India,
Objective :
and a State Compensatory Afforestation Fund under
To know about the Van Dhan scheme and various the Public Account of each state.
other like initiatives or schemes
• The payments into the funds include compensatory
afforestation, net present value of forest (NPV), and
248)The “National Compensatory Afforestation Fund any project specific payments.
Management and Planning Authority (National • The National Fund will get 10% of funds collected
CAMPA) is mechanism setup for: and the remaining 90% will go to respective State
A) Utilization of collected funds for Afforestation, Fund.
regeneration of forest ecosystem, wildlife protection • The collected funds will be utilised for afforestation,
and forest related infrastructure development regeneration of forest ecosystem, wild life protection
B) Diverse the funds for use of forest land for non – and forest related infrastructure development.
forest purposes such as industrial or infrastructure • The bill also seeks to establish National and State
projects. Compensatory Afforestation Fund Management and
C) Allocating the 10% of funds to states CAMPA for Planning Authorities to manage the funds.
Afforestation, regeneration of forest ecosystem, Objective :
wildlife protection and forest related infrastructure
development. To know about the National CAMPA.

Page No.86
MARCH CURRENT AFFAIRS_2019
Issues relating to Health, Key : D
education and Human Explanation :
resources •. NASA has selected a new mission that will help
249) Consider the following statements about ‘ Mental scientists understand and, ultimately, forecast the
Health ‘ in India. vast space weather system around our planet.The
1. National Mental Health Survey ( NMHS) was Atmospheric Waves Experiment (AWE) mission will
conducted in the year 2014-16 in all the states. cost $42 million and is planned to launch in August
2022, attached to the exterior of the Earth-orbiting
2. As per the census 2011, there are nearly 15 lakh
International Space Station (ISS),.From its space
mentally challenged persons in the country.
station perch, AWE will focus on colourful bands of
3. National Mental Health policy aims to promote de- light in Earth’s atmosphere, called airglow, to
stigmatization and desegregation of persons affected determine what combination of forces drive space
by mental illness. weather in the upper atmosphere. Space weather is
Select the correct answer using the code given below important because it can have profound impacts -
A) 1 and 2 only B) 2 and 3 only affecting technology and astronauts in space,
C) 1 and 3 only D) 1, 2 and 3 disrupting radio communications and, at its most
severe, overwhelming power grids.
Key : B
•. It was earlier thought that only the Sun’s constant
Explanation :
outflow of ultraviolet light and particles, the solar
•. National Mental Health survey was conducted in the wind, could affect the region. However, researchers
year 2014-16 and its report released in the year 2016. now have learned that solar variability is not enough
the survey was done in 12 states. the pilot survey was to drive the changes observed, and Earth’s weather
conducted in the state of Karnataka. also must be having an effect.
•. As per the census-2011, there are 15 lakh persons
who are mentally ill.
•. AWE wi ll investigate how waves in the lower
•. Government formulated a policy, to address the above atmosphere, caused by variations in the densities of
problem in the year 2016. some of the objectives of different packets of air, impact the upper atmosphere.
the policy are :
Objective :
•. provide universal access to mental health care.
•. To know about latest experiments which helps in better
To increase access to and utilization of comprehensive understanding of Earth.
mental health services by persons with mental health
problems.
251) Which of the following missions constituted by NASA
To increase access to mental health care especially to
to study and forecast the vast space weather system
vulnerable groups including homeless persons,
around the Earth?
persons in remote areas, educationally, socially and
deprived sections. A) ARTEMIS B) THEMIS
To reduce the prevalence and impact of risk factors C) New Horizons
associated with mental health problems. D) Atmospheric Waves Experiment
To reduce the risk and incidence of suicide and attempted Key : D
suicide. Explanation :
To ensure respect for rights and protection from harm of •. NASA has selected a new mission that will help
persons with mental health problems. scientists understand and, ultimately, forecast the
To reduce the stigma associated with mental health vast space weather system around our planet.The
problems. Atmospheric Waves Experiment (AWE) mission will
To enhance the availability and equitable distribution of cost $42 million and is planned to launch in August
skilled human resources for mental health. 2022, attached to the exterior of the Earth-orbiting
International Space Station (ISS),.From its space
Objective :
station perch, AWE will focus on colourful bands of
To know about the status of mental health in India. light in Earth’s atmosphere, called airglow, to
Science & Technology determine what combination of forces drive space
Space weather in the upper atmosphere. Space weather is
250) Which of the following missions constituted by NASA important because it can have profound impacts -
to study and forecast the vast space weather system affecting technology and astronauts in space,
around the Earth? disrupting radio communications and, at its most
severe, overwhelming power grids.
A) ARTEMIS B) THEMIS
•. It was earlier thought that only the Sun’s constant
C) New Horizons
outflow of ultraviolet light and particles, the solar
D) Atmospheric Waves Experiment wind, could affect the region. However, researchers

Page No.87
MARCH CURRENT AFFAIRS_2019
now have learned that solar variability is not enough 3. The appointment of governor by the centre is the
to drive the changes observed, and Earth’s weather most significant feature of federal system of
also must be having an effect. government.
•. AWE wi ll investigate how waves in the lower Select the correct answer using the code given below.
atmosphere, caused by variations in the densities of A) 1 and 3 only B) 1 and 2 only
different packets of air, impact the upper atmosphere. C) 2 and 3 only D) 1, 2 and 3
Key : C
Objective : Explanation :
To know about latest experiments which helps in better •. Statement 1 is incorrect. Article 1 of the constitution
understanding of Earth. describes India as a “Union of States” there is no word
252) Which of the following missions constituted by NASA called “Federation” mentioned in constitution as such.
to study and forecast the vast space weather system •. Statement 2 is correct. The phrase “Union of States”
around the Earth? implies that no state has right to secede from the
A) ARTEMIS B) THEMIS Union.
C) New Horizons •. Statement 3 is correct The post of Governor, a strong
D) Atmospheric Waves Experiment centre etc are considered as non-federal features.
Key : D Hence, the appointme.nt of governor is the most
Explanation : significant feature of non-federal / unitary form of
government.
•. NASA has selected a new mission that will help
scientists understand and, ultimately, forecast the Objective :
vast space weather system around our planet.The •. To know about the federal provisions mentioned in
Atmospheric Waves Experiment (AWE) mission will the constitution.(Indian Constitution - Features)
cost $42 million and is planned to launch in August 254) Which among the following about Indian
2022, attached to the exterior of the Earth-orbiting constitution is true?
International Space Station (ISS),.From its space A) Communal representation in Executive.
station perch, AWE will focus on colourful bands of B) Provision of Fundamental Duties in the original
light in Earth’s atmosphere, called airglow, to constitution.
determine what combination of forces drive space
C) Flexible nature of Indian constitution.
weather in the upper atmosphere. Space weather is
important because it can have profound impacts - D) The control of states by centre at specific times.
affecting technology and astronauts in space, Key : D
disrupting radio communications and, at its most Explanation :
severe, overwhelming power grids. •. Fundamental duties are not provided in the original
•. It was earlier thought that only the Sun’s constant document, but are later added by 42nd constitutional
outflow of ultraviolet light and particles, the solar amendment Act of 1976. Indian constitution is both
wind, could affect the region. However, researchers flexible & rigid; some bills needing simple majority &
now have learned that solar variability is not enough some needing special majority make it clear that
to drive the changes observed, and Earth’s weather Indian constitution is neither fully rigid nor fully
also must be having an effect. flexible. Communal representation & reservation in
•. AWE wi ll investigate how waves in the lower executive is not mentioned in the constitution. But it
atmosphere, caused by variations in the densities of is the discretion of legislature whether to provide or
different packets of air, impact the upper atmosphere. to vary the percentage of reservation on communal
Objective : basis in the executive. However, though the communal
representation in legislature is abolished by
. To know about latest experiments which helps in better
constitution; reservation for SCs & STs is provided.
understanding of Earth.
•. The “Emergency situation” is the best example of
Polity
centre controlling the states. During Emergency, the
History underpinnings centre takes the control of whole country, including
Evolution, Features of Indian the powers of states which is more like the federal
Constitution system of government turning into an unitary one
253) Consider the following statements about Federal without a formal amendment to the constitution if
system of government. not absolutely. This is a unique feature of Indian
1. Article 1 of the constitution describes India as a constitution.
“Federation of states” Objective :
2. The constitution allowed no state to secede from To know about salient & unique features of the Indian
the federation. constitution.

Page No.88
MARCH CURRENT AFFAIRS_2019
Preamble 4. Bank or post office passbooks carrying photographs
255) Consider the following statements about Preamble. of voters,
1. Preamble is not a part of constitution. 5. PAN card,
2. The basic features preamble of cannot be amended 6. Smart cards issued under the National Population
under Article 368. Register,
3. The Preamble can be amended under Article 368. 7. MNREGA job card
4. The Original Preamble document had no mention about 8. Health insurance smart card
the terms Socialist & Secular. 9. Pension documents with photograph,
Select the correct answer using the code given below. 10.Official identity cards issued to the legislators and
A) 1, 2 and 3 only B) 2, 3 and 4 only 11.Aadhaar card
C) 1, 3 and 4 only D) 1, 2 and 4 only Photo Voter Slip is no longer an identity proof for
Key : B voters to exercise their franchise.
Explanation : Objective :
•. As per the directives issued by Supreme Court of India To know the documents allowed by Election
in BeruBari Union Case of 1960, it held that Preamble Commission as proof of voters identity
is not a part of the Constitution. But, in Keshavananda
Bharati Case of 1973, it revised its own judgment and International Relations
held that the Preamble is an integral Part of the India-South Asia(India and its
constitution and can be amended as per Article 368.
neighbhors)
However, the “Basic features” of the Preamble Cannot
be amended.The original Preamble document had no 257) Consider the following statements about India -
mention of the terms “Socialist” “Secular”, “Integrity” Bangladesh relations:
they were added by the 42rd constitutional 1. Sampritee military exercise is aimed at counter-
amendment Act of 1976. Hence statements 2,3 and 4 terrorism and to aid civil authorities for disaster
are correct. management in a UN peacekeeping scenario.
•. This shows that the preamble can be amended but the 2. Both the countries are part of SAARC, BIMSTEC regional
basic features of the constitution cannot be. groupings and also Commonwealth of Independent
Objective : States.
To know about the amendability of the preamble of Select the incorrect statement(s) using the code given
the Indian constitution. below:
Elections and representation A) 1 only B) 2 only
of people act C) Both 1 and 2 D) Neither 1 nor 2
256) Which among the following documents are allowed Key : B
by Election Commission as voter ’s identity for Explanation :
exercising their franchise? •. The eighth edition of joint military exercise between
1. MGNREGA job card the armies of India and Bangladesh began yesterday
2. Post office pass book with an impressive ceremony at Bongobondhu
3. Photo Voter Slip Cantonment in Tangail, Bangladesh. The joint military
exercise ‘Sampriti’ is aiming to increase mutual
4. Service identity card of public limited cooperation, bonhomie and camaraderie between the
ompany armies of two countries.
5. Passport •. It is a two-week long exercise that will see them hone
Select the correct answer using the code below: their tactical and technical skills in countering the
A) 2 and 3 only B) 1,3 and 5 only insurgency, counter-terrorism and aid to civil
C) 2,3 and 5 only D) 1,2,4 and 5 only authorities for disaster management in a UN
Key : D peacekeeping scenario involving a combined
deployment at a battalion level.
Explanation :
•. Both India and Bangladesh are part of SAARC, BIMSTEC
11 documents allowed by Election Commission as proof and Commonwealth, But both are not part in
of voters identity: Commonwealth of Independent States grouping.
1. passport, Objective :
2. driving licence, To know about India’s relations with neighbourhood
3. service identity cards of the Central and State countries.
governments, public sector undertakings and public
limited companies,

Page No.89
MARCH CURRENT AFFAIRS_2019
Bilateral and Regional every four years to reassess the need for joint actions
groupings/agreements to secure their future. The twelve point strategic
involving India and its actions thus outlined the following actions-
interests To collaborate to strengthen protection regimes, strategic
information gathering, and real time sharing of
258) Consider the following statements about RCEP actionable information on rhino crime and its horn
(Regional Comprehensive Economic Partnership): trade to secure the rhino population within and
1. Both Timor Leste and Papua New Guinea are not a between range countries;
part of this grouping.
To initiate research on various habitat parameters
2. The RCEP negotiations includes SMEs (Small and including invasive species threatening the suitable
Medium Enterprises). habitats of Asian rhinos and take appropriate steps
Select the correct statement(s) using the code given to optimally manage the habitats;
below: To explore possibilities of expanding rhino ranges within
A) 1 only B) 2 only country or between rhino range countries for optimal
C) Both 1 and 2 D) Neither 1 nor 2 population management;
Key : C To strengthen transboundary collaboration among India,
Explanation : Nepal, and Bhutan for the greater one-horned rhino
conservation and protection;
•. The RCEP negotiations were launched by Leaders from
10 ASEAN Member States (Brunei Darussalam, Objective :
Cambodia, Indonesia, Lao PDR, Malaysia, Myanmar, To know about the conservation efforts of Asian
the Philippines, Singapore, Thailand and Viet Nam) Rhinos.
and six ASEAN FTA partners (Australia, People’s
Republic of China, India, Japan, Republic of Korea, 260) Consider the following pairs:
and New Zealand) during the 21st ASEAN Summit and
Rhino Conservation areas Location
Related Summits in Phnom Penh, Cambodia in
November 2012. 1. Manas National Park Assam
•. The RCEP negotiation includes: trade in goods, trade 2. Buxa Tiger Reserve West Bengal
in services, investment, economic and technical 3. Valmiki Tiger Reserve Bihar
cooperation, intellectual property, competition, Select which of the above is/are correctly paired:
dispute settlement, e-commerce, small and medium A) 1 Only B) 2 and 3 Only
enterprises (SMEs) and other issues. C) 1 and 2 Only D) 1,2 and 3
Objective : Key : D
To know about the importance of regional and global Explanation :
groupings with respect to India.
•. All the given conservation sites are correctly paired
Objective :
Environment Ecology
To know about various Rhino conservation areas in
CONSERVATION EFFORTS India
259) The “New Delhi Declaration” which emphasizes the
“Rhinos without Borders” movement majorly
concentrates on movement and conservation of Asian Geography
Rhinos between which three countries among the Geographical features of India
following? Physiographyhy of India
A) India - Nepal - Bhutan 261) In the context of Difference between International
B) India - Bangladesh - Myanmar Border (IB) and Line of Control (LoC) consider the
C) India - Bhutan - Bangladesh following statements:
1.IB is de facto (cease fire) border Shared by India
D) India - Nepal - Bangladesh
and Pakistan, Where as LoC is internationally
Key : A recognised and legal.
Explanation : 2.Red Cliff Line between India and Pakistan is an
•. With an objective to increase the population of three example for Line of Control.
Asian rhino species (Greater one-horned rhinoceros, Which of the above is/are correct using the code given
Javan rhino, and Sumatran rhino), the New Delhi below?
Declaration on Asian Rhinos 2019 was signed by
representatives of the governments of India, Bhutan, A) 1 only B) 2 only
Indonesia, Malaysia, and Nepal. As part of the C) Both 1 and 2 D) Neither 1 nor 2
declaration, the rhino range countries also agreed to Key : D
review the population of the three Asian rhino species Explanation :

Page No.90
MARCH CURRENT AFFAIRS_2019
•. The India-Pakistan international border is the • Following this US Military Base was setup in Diego
internationally recognized boundary that separates Garcia, the largest of the islands.
the provinces of Pakistan from the states of India. • 2,000 inhabitants were resettled in Mauritius and the
•. It is based upon the Radcliffe Line, which was an Seychelles.
imaginary line to divide the Punjab, recommended by • The UN resolutions banned the dismemberment of
a committee chaired by the English barrister Sir Cyril colonial territories before independence.
Radcliffe, continuing to divide Gujarat from Sindh, • Therefore Mauritius claims sovereignty over the
and Kashmir from the North West Frontier Provinces islands and states that Britain’s claim is a violation
of Pakistan. of law and of UN resolutions.
•. The Line of Control (LoC), is the de facto border • The resettled inhabitants now number around 10,000
established after conflict, which means it is not including their descendants and they wish to resettle.
internationally deemed as a legitimate boundary.
• UK declared Marine Protected Area around Chagos in
•. The LoC is the result of the solidification of fronts 2010, which prohibits fishing and extractive industry
after the ceasefire in the Kashmir war of 1947, when and has the effect of preventing any resettlement.
raiders backed by Pakistan entered the Kashmir valley,
• In 2015, the Permanent Court of Arbitration ruled this
repelled by Indian forces.
move as illegal under the United Nations Convention
•. The LoC divides the previous princely state of Jammu on the Law of the Sea (UNCLOS).
and Kashmir. The provinces of Azad Kashmir and
• According to PCA, Mauritius holds legally binding
Gilgit-Baltistan are segregated from the rest of the
rights to fish in the waters surrounding the Chagos.
Indian state by it. Both of these constitute disputed
to the eventual return of the Chagos to Mauritius when
territory, and are currently administered by Pakistan.
no longer needed for defence purposes to the
•. Therefore, the LoC is not an official border, but a line preservation of the benefit of any minerals or oil
of military control separating disputed areas. discovered in the Chagos Archipelago pending its
Objective : eventual return
To know about the difference between International • In November 2016, the U.K. ruled out the resettlement
Border and Line of Control. of the islanders on the grounds of feasibility, defence,
security interests and the cost.
Geomorphology • UK also renewed the lease for Diego Garcia with US,
262) Which of the following is/are correct with respect up until 2036.
to geographical location/feature of The Chagos • Subsequently, Mauritius warned that it would push
Archipelago? to take the matter to the International Court of Justice
1.The Chagos Archipelago is a group of islands which (ICJ).
are formed through Volcanic Erruptions from the Objective :
ocean bed. To know about the different Archipelago locatitona
2.Chagos Archipelago islands are located in Northern and its importance geographically, politically and
hemisphere i.e. above the Equator. economically.
3.The largest island in this group is Deigo Garcia
Select the correct answer using the code given below Economy
A) 2 and 3 only B) 3 only International Organisations
C) 1 and 3 only D) 1, 2 and 3 263) Consider the following statements about
Key : B “International Court of Justice” (ICJ):
Explanation : 1.The International Court of Justice is composed by
• The Chagos Archipelago is a group of seven atolls the United Nations General Assembly only
comprising more than 60 islands in the Indian Ocean 2.The retiring judges of ICJ are not re elected
about 500 km south of the Maldives. 3.The Judgement given by ICJ is an advisory opinion
• It had been part of Mauritius since the 18th century only.
when the French first settled the islands. Which of the above statement (s) is/are incorrect
• All of the islands of French colonial territory in the using the code given below?
region were ceded to the British in 1810. A) 1 and 2 only B) 3 only
• Before Mauritian independence, in 1965, the UK split C) 2 and 3 only D) 1, 2 and 3
the archipelago from the territory of Mauritius to form Key : A
the British Indian Ocean Territory and permitted the Explanation :
US to use it for defence purposes for 50 years (until
December 2016) followed by a 20-year optional •. The International Court of Justice is the principal
extension. judicial organ of the United Nations. Its seat is at the
Peace Palace in The Hague (Netherlands). It began

Page No.91
MARCH CURRENT AFFAIRS_2019
work in 1946, when it replaced the Permanent Court •. PMKVY is the flagship scheme of the Ministry of Skill
of International Justice which had functioned in the Development & Entrepreneurship. The objective is to
Peace Palace since 1922. It operates under a Statute enable a large number of Indian youth to take up
largely similar to that of its predecessor, which is an industry-relevant skill training that will help them in
integral part of the Charter of the United Nations. securing a better livelihood. Individuals with prior
Functions of the Court learning experience or skills will also be assessed
•. The Court has a dual role: to settle in accordance and certified under Recognition of Prior Learning
with international law the legal disputes submitted (RPL). Under this Scheme, Training and Assessment
to it by States, and to give advisory opinions on legal fees are completely paid by the Government.[2]
questions referred to it by duly authorized •. PMKVY is applicable to any candidate of Indian
international organs and agencies. nationality who is unemployed, school or college
Composition dropout, or as identified by the Sector Skill Council
(SSC) for their respective job roles.
•. The Court is composed of 15 judges elected to nine-
year terms of office by the United Nations General Objective :
Assembly and Security Council sitting independently To know about the PM Kaushal Vikas Yojana
of each other. It may not include more than one judge
of any nationality. Elections are held every three years Infrastructure
for one-third of the seats, and retiring judges may be 265) Which of the following is/are correct with regard to
re-elected. The Members of the Court do not represent National mineral policy 2019?
their governments but are independent magistrates.
1. It encourages private sector to take up exploration
The judges must possess the qualifications required
in their respective countries for appointment to the 2. It proposed to grant industry status to mining activity
highest judicial offices, or be jurists of recognized 3. It introduced the concept of inter-generational equity
competence in international law. The composition of Select the correct answer using the code below:
the Court has also to reflect the main forms of A) 1 and 2 only B) 2 and 3 only
civilization and the principal legal systems of the
C) 1 and 3 only D) 1,2 and 3
world. When the Court does not include a judge
possessing the nationality of a State party to a case, Key : D
that State may appoint person to sit as a judge ad hoc Explanation :
for the purpose of the case. •. The National Mineral Policy 2019 includes provisions
•. ICJ is principal judicial body of UN. It was established which will give boost to mining sector. Some of the
in 1945 via UN Charter and its jurisdiction is provisions are:
worldwide. It is located in “The Hague” in Netherlands 1. Introduction of Right of First Refusal for RP/PL holders.
and has 193 state parties. Current President is Ronny 2. Encouraging the private sector to take up exploration.
Abraham.
3. Encouragement of merger and acquisition of mining
•. The major functions of ICJ are to settle legal disputes entities.
submitted to it by states and to provide advisory
4. Creation of dedicated mineral corridors to boost
opinions on legal questions submitted to it by duly
private sector mining areas.
authorized international organs, agencies, and UN
General Assembly. 5. The 2019 Policy proposes to grant status of industry
to mining activity to boost financing of mining for
private sector and for acquisitions of mineral assets
Objective : in other countries by private sector.
To know about the International Court of Justice (ICJ). 6. It also mentions that Long term import export policy
for mineral will help private sector in better planning
Employment & Unemployment and stability in business.
264) Which of the following statements is/are correct 7. The Policy also mentions rationalize reserved areas
about Pradhan Mantri Kaushal Vikas Yojana (PMKVY)? given to PSUs which have not been used and to put
1. Individuals with prior learning skills are recognized these areas to auction, which will give more
under the scheme. opportunity to private sector for participation.
2. unemployed, school and college dropouts are 8. The Policy also mentions to make efforts to harmonize
covered. taxes, levies & royalty with world benchmarks to help
private sector.
Select the correct answer using the code given below
9. The Policy also introduces the concept of Inter-
A) 1 only B) 2 only
Generational Equity that deals with the well-being not
C) Both 1 and 2 D) Neither 1 nor 2 only of the present generation but also of the
Key : C generations to come and also proposes to constitute
Explanation : an inter-ministerial body to institutionalize the

Page No.92
MARCH CURRENT AFFAIRS_2019
mechanism for ensuring sustainable development in •. Pair 2 is not matched correctly: Rosetta is a space
mining. probe built by the European Space Agency to perform
Objective : a detai led study of comet 67P/Churyumov-
To know about National Mineral Policy, 2019 Gerasimenko (67P). Juno is a NASA space probe
orbiting the planet Jupiter. Its mission is to measure
2019-03-02 Jupiter’s composition, gravity field, magnetic field,
Science & Technology and polar magnetosphere.
Space •. Pair 3 is matched correctly: Dawn is a space probe
266) Consider the following events in space launched by NASA with the mission of studying two of
1. Explosion of star. the three known protoplanets of the asteroid belt,
2. Two big stars moving at very high speed. Vesta and Ceres.
3. Two black holes orbit each other and merge. Objective :
Which of the above can cause gravitational waves? To know about the developments in Space science and
technology.
A) 1 only B) 3 only
C) 2 and 3 only D) 1, 2 and 3
268) Consider the following statements with reference to
Key : D
Giant Meterwave Radio Telescope (GMRT) India
Explanation :
1. It is located in the state of Andhra Pradesh.
•. All three statements are correct.Gravitational waves
2. It aims to determine the event of galaxy formation
are produced when masses accelerate changing
in the universe.
distortion of space. So, everything with mass can make
gravitational waves. A supernova star or black hole Which of the statements given above is/are correct?
has mass. Gravitational waves are small ripples in A) 1 only B) 2 only
space-time that are believed to travel across the C) Both 1 and 2 D) Neither 1 nor 2
universe at the speed of light. Key : B
•. The most powerful gravitational waves are created Explanation :
when objects move at very high speeds. Some examples
•. Statement 1 is incorrect. India‘s GMRT-It is operated
of events that could cause a gravitational wave are:
by the National Centre for Radio Astrophysics, a part
• when a star explodes asymmetrically (called a of the Tata Institute of Fundamental Research,
supernova) Mumbai. The Giant Meterwave Radio Telescope
• when two big stars orbit each other (GMRT) is being set up near Pune in India consisting
• when two black holes orbit each other and merge of 30 numbers of 45-m diameter parabolic dishes. At
•. But these types of objects that create gravitational the time it was built, it was the world’s largest
waves are far away. And sometimes, these events only interferometric array offering a baseline of up to 25 -
cause small, weak gravitational waves. The waves are kilometres.
then very weak by the time they reach Earth. This makes •. Statement 2 is correct. One of the aims for the
gravitational waves hard to detect. telescope during its development was to search for
Objective : the highly redshifted 21-cm line radiation from
primordial neutral hydrogen clouds in order to
Know about the Gravitational wave and its origin.
determine the epoch of galaxy formation in the
universe.
267) Which of the following pairs of Spacecraft and their Objective :
purpose is/are correct?
•. To know about the Giant Meterwave Radio Telescope
1. Kepler : To discover Earth-size planets orbiting other (GMRT) of India and its main Objective.
stars
2. Rosetta : To orbit Jupiter and studying its
269) With respect to space science, the ‘redshift’
composition
phenomenon provides evidence for?
3. Dawn : To study protoplanets of the asteroid belt
A) expansion of the Universe
Select the correct answer using the code given below:
B) presence of dark energy
A) 1 and 2 only B) 1 and 3 only
C) presence of gravitational waves
C) 2 only D) 1, 2 and 3
D) formation of black holes
Key : B
Key : A
Explanation :
Explanation :
•. Pair 1 is matched correctly: Kepler is a space
•. Redshift and blueshift describe how light changes as
observatory launched by NASA to discover Earth-size
objects in space (such as stars or galaxies) move
planets orbiting other stars.
closer or farther away from us. The concept is key to

Page No.93
MARCH CURRENT AFFAIRS_2019
charting the universe’s expansion. (Option 1 is Select correct answer using given code below:
correct) A) 1 and 2 Only B) 2 and 3 Only
•. Visible light is a spectrum of colors, which is clear to C) 1 and 3 Only D) 1,2 and 3
anyone who has looked at a rainbow. When an object Key : D
moves away from us, the light is shifted to the red end
of the spectrum, as its wavelengths get longer. If an Explanation :
object moves closer, the light moves to the blue end of •. Gravitational waves are ripples in the curvature of
the spectrum, as its wavelengths get shorter. spacetime that propagate at the speed of light,
generated in certain gravitational interactions .
Objective :
Predicted in 1916 by Albert Einstein on the basis of
To know about some of the key terms associated with his theory of general relativity, gravitational waves
Space science and technology transport energy as gravitational radiation, a form of
radiant energy similar to electromagnetic radiation.
270) ISRO carried out successful testing of scramjet Objective :
engine from the Satish Dhawan Space Centre. Which To know about the Gravitational wave and its
of the following statements is/are true about Scramjet properties.
Engine used in space rockets?
1. It is a special category of ramjet air breathing jet
engine in which combustion takes place in supersonic Current affairs on Science
airflow. and technology
2. It uses Hydrogen as fuel and the Oxygen from the 272) Consider the following statements related to the
atmospheric air as the oxidiser. recent initiatives launched by the government for the
Select correct answer using given code below: growth of Science and Technology:
1. IMPRINT: A roadmap for research to solve the major
A) 1 only B) 2 only
engineering and technology challenges relevant to
C) Both 1 and 2 D) Neither 1 nor 2 India.
Key : C 2. KIRAN: Aimed at tapping the talent pool of scientists
Explanation : and entrepreneurs, internationally to encourage their
•. A scramjet is a variant of a ramjet air breathing jet engagement with the institutes of Higher Education in
engine in which combustion takes place in supersonic India.
airflow. As in ramjets, a scramjet relies on high vehicle 3. GIAN: To address issues related with women scientists
speed to forcefully compress the incoming air before and aimed to provide opportunities in research,
combustion (hence ramjet), but a ramjet decelerates technology development, etc.
the air to subsonic velocities before combustion, while Select correct answer using given code below:
airflow in a scramjet is supersonic throughout the
A) 1 Only B) 2 Only
entire engine. This allows the scramjet to operate
efficiently at extremely high speeds. C) 2 and 3 Only D) 1,2 and 3
Advantages Key : A
• Does not have to carry oxygen Technique : Layered
• No rotating parts makes it easier to manufacture than a Explanation :
turbojet •. “KIRAN” (Knowledge Involvement in Research
• Has a higher specific impulse (change in momentum Advancement through Nurturing) is addressing various
per unit of propellant) than a rocket engine issues related with women scientists (e.g.
• Higher speed could mean cheaper access to outer space unemployment, relocation etc.) and aimed to provide
in the future opportunities in research (WOS¬A), technology
development/demonstration (WOS¬B), and
Objective : self¬employment (WOS¬C) etc.
To know about the advantage of scramjet and its usage •. KIRAN is also actively involved in taking proactive
in ISRO measures, under the name CURIE (Consolidation of
271) Which of the following is/are true about University Research for Innovation and Excellence in
Gravitational waves? Women Universities) to develop tate¬of¬the¬art
1. Gravitational waves are ripples in the curvature of •. Infrastructure in women universities in order to
spacetime that propagate at the speed of light, attract, train and retain promising girls students in
generated in certain gravitational interactions. S&T domain. Global Initiative of Academic Networks
2. Gravitational waves can penetrate regions of space (GIAN) in Higher Education aimed at tapping the talent
that Electromagnetic radiations has no reach. pool of scientists and entrepreneurs, internationally
3. Gravitational waves travel at the speed of light and to encourage their engagement with the institutes of
distort space¬time on their path. Higher Education in India so as to augment the
country’s existing academic resources, accelerate the

Page No.94
MARCH CURRENT AFFAIRS_2019
pace of quality reform, and elevate India’s scientific A) Acidic lavas are hotter than basic lavas.
and technological capacity to global excellence. B) Acidic lavas contain higher percentage of silica
Objective : than basic lavas.
To know about various initiatives of dept.of science C) Basic lavas flow slowly and are more viscous than
and technology acidic lavas.
D) Both (a) and (c) are correct
Energy Key : B
273) Ministry of Petroleum & Natural Gas, Government Explanation :
of India joined IEA (International Energy Agency) Bio- •. Statement 1 is incorrect : Basic lavas are the hottest
energy Technology Collaboration Program (TCP) as lavas, which are about 1000 degree C. Statement 2 is
its 25th member recently. In this context, consider the correct: Acidic lavas contain higher percentage of
following statements: Silica than basic lavas. Basic lavas are dark coloured,
1. The primary goal of joining is to facilitate the market rich in iron and magnesium but poor in
introduction of advanced bio-fuels with an aim to silica.Statement 3 is incorrect: Basic lavas have very
bring down emissions and reduce crude imports. high fluidity and flow readily. The resultant volcano
2. International Energy Agency is an autonomous is gently sloping with a wide diameter and forms a
intergovernmental organization established in the flattened shield or dome. Acidic lavas flow slowly
framework of OECD. and are more viscous and resultant cone is steep-
3. India is a member country of the International sided.
Energy Agency. Objective :
Which of the statement(s) given above is/are correct? Student has to know the basics about the acidic and
A) 1 only B) 2 and 3 only basic lavas.
C) 1 and 2 only D) 1 and 3 only
Key : C 275) Consider the following pairs:
Explanation : Type of fault Description
•. Ministry of Petroleum & Natural Gas, Government of 1. Normal Fault : When a section of land slides down
India joined IEA (International Energy Agency) Bio- along the fault line
energy TCP as its 25th member recently. It is an 2. Reverse Fault : When a section of land thrusts up
international platform for co-operation among along the fault line
countries. It aims to improve cooperation and 3. Transform Fault : When a section of land moves
information exchange between countries that have horizontally along the fault line
national programs in bio-energy research, Which of the pairs given above is/are correctly
development and deployment. The primary goal of matched?
joining is to facilitate the market introduction of A) 1 only B) 2 only
advanced bio-fuels with an aim to bring down
emissions and reduce crude imports. C) 2 and 3 only D) 1, 2 and 3
Key : D
•. It works under the framework of International Energy
Agency (IEA). India is an Associate member of IEA. Explanation :
•. IEA is a Paris-based autonomous intergovernmental •. Statement 1 is correct. A normal fault is when a
organization established in the framework of the section of land slides down along the fault line.
Organisation for Economic Co-operation and statement 2 is correct. A reverse fault is when a
Development (OECD). Only OECD member states can section of land thrusts up along the fault line.
become members of the IEA. Except for Chile, Iceland, Statement 3 is correct. A tear fault is when a section
Israel, Latvia, Slovenia all OECD member states are of land displaces horizontally.
members of the IEA. Mexico has recently joined IEA as Objective :
30th member. Students some times get confusion regarding different
•. Brazil, China, India, Indonesia, Morocco, Singapore faults. He has to get the clarity about A normal fault,
and Thailand are the associate members of IEA. reverse fault and Transform Fault
Objective :
To know about IEA’s Bio-energy Technology 276) Arrange the following elements in decreasing order
Collaboration Program (TCP) of their weight in composition of earth’s crust.
Geography 1. Silicon 2. Oxygen
Origin And Evolution Of Earth, 3. Iron 4. Aluminium
Interiors Of The Earth Select the answer using the code given below
274) Which of the following statements with reference to A) 1-2-3-4 B) 2-1-3-4
acidic and basic lavas is correct? C) 2-1-4-3 D) 1-2-4-3

Page No.95
MARCH CURRENT AFFAIRS_2019
Key : C or producing bioethanol through fermentation. Crops
Explanation : such as wheat and sugar are the most widely used
•. The Major Elements of the Earth’s crust:Elements By feedstock for bioethanol while oil seed rape has
Weight (%) proved a very effective crop for use in biodiesel.
1. Oxygen 46.60 •. Second Generation biofuels have been developed to
overcome the limitations of first generation biofuels.
2. Silicon 27.72 They are produced from non-food crops such as wood,
3. Aluminium 8.13 organic waste, food crop waste and specific biomass
4. Iron 5.00 crops, therefore eliminating the main problem with
5. Calcium 3.63 first generation biofuels.
Objective : •. The Third Generation of biofuels is based on
Multiple answer type questions are frequently coming improvements in the production of biomass. It takes
in upsc prelims.So student has to practice more advantage of specially engineered energy crops such
questions in this regard. as algae as its energy source. The algae are cultured
to act as a low-cost, high-energy and entirely
renewable feedstock.
Governance
•. Fourth Generation Bio-fuels are aimed at not only
Government policies and producing sustainable energy but also a way of
programs capturing and storing CO2. Biomass materials, which
277) Consider the following statements with respect to have absorbed CO2 while growing, are converted into
the “Pradhan Mantri JI-VAN (Jaiv Indhan- Vatavaran fuel using the same processes as second generation
Anukool fasal awashesh Nivaran) Yojana” biofuels.
1. The scheme provides financial support to Integrated Objective :
Bio ethanol Projects using lignocellulosic biomass To know about the details of Pradhan Mantri Jl-VAN
and other renewable feedstock. yojana scheme and its objectives
2. The scheme is mainly focuses on the boosting of
Third generation(3G) Bio fuel sector. 278) Which one of the following statements is not correct
3. The ethanol produced by the scheme beneficiaries with respect to the newly approved ‘National Mineral
will be mandatorily supplied to O il Marketing Policy, 2019’?
Companies to further enhance the blending percentage A) Introduces the concept of Inter-Generational Equity
under EBP Programme.
B) cancelled the Right of First Refusal for RP
Which of the statement(s) given above is/are correct? (Reconnaissance Permits)/PL (Prospecting Licenses)
A) 1 and 2 only B) 2 and 3 only holders
C) 1 and 3 only D) 1, 2 and 3 C) Encouraging the private sector to take up
Key : C exploration
Explanation : D) Encourages dedicated mineral corridors to
•. The Cabinet Committee on Economic Affairs has facilitate the transportation of minerals
approved the “Pradhan Mantri JI-VAN (Jaiv Indhan- Key : B
Vatavaran Anukool fasal awashesh Nivaran) Yojana”. Explanation :
•. The scheme provides financial support to Integrated •. National Mineral Policy, 2019 was recently approved
Bioethanol Projects using lignocellulosic biomass by Cabinet.
and other renewable feedstock. •. The aim of National Mineral Policy 2019 is to have a
•. The scheme focuses to incentivise Second Generation more effective, meaningful and implementable policy
(2G) Ethanol sector and support this nascent industry that brings in further transparency, better regulation
by creating a suitable ecosystem for setting up and enforcement, balanced social and economic
commercial projects and increasing Research & growth as well as sustainable mining practices.
Development in this area. •. The National Mineral Policy 2019 includes provisions
•. The ethanol produced by the scheme beneficiaries will which will give boost to mining sector. Some of the
be mandatorily supplied to Oil Marketing Companies provisions are:
(OMCs) to further enhance the blending percentage Introduction of Right of First Refusal for RP/PL holders.
under EBP Programme.
Encouraging the private sector to take up exploration.
•. Centre for High Technology (CHT), a technical body
Encouragement of merger and acquisition of mining
under the aegis of MoP&NG, will be the
entities.
implementation Agency for the scheme.
Creation of dedicated mineral corridors to boost
Additional Facts:
private sector mining areas.
•. First Generation biofuels are produced directly from
food crops by abstracting the oils for use in biodiesel

Page No.96
MARCH CURRENT AFFAIRS_2019
The 2019 Policy proposes to grant status of industry assemble peaceably without arms and to form
to mining activity to boost financing of mining for associations.
private sector and for acquisitions of mineral assets •. The Act came after the 16th CAA, 1963 which placed
in other countries by private sector. reasonable restrictions on [Ar.19(1)], [Ar.19(2)],
It also mentions that Long term import export policy [Ar.19(3)].
for mineral will help private sector in better planning •. List of Banned Organizations in India is made under
and stability in business. Section 35 of the Unlawful Activities (Prevention) Act,
The Policy also mentions rationalize reserved areas 1967 – the period for which an association can be
given to PSUs which have not been used and to put declared as unlawful is five years.
these areas to auction, which will give more •. The Unlawful Activities (Prevention) Act, 1967 helps
opportunity to private sector for participation. meet the commitments to the Financial Action Task
The Policy also mentions to make efforts to harmonize Force.
taxes, levies & royalty with world benchmarks to help Objective :
private sector. To know about Unlawful Activities (Prevention) Act,
The Policy also introduces the concept of Inter- 1967.
Generational Equity that deals with the well-being not
only of the present generation but also of the
International Relations
generations to come and also proposes to constitute
an inter-ministerial body to institutionalize the Asia & Pacific
mechanism for ensuring sustainable development in 280) Hanoi summit sometimes seen in the news recently,
mining. is associated with which of the following?
Objective : A) USA and North Korea
To know about the details and objective of the National B) Israel and Palestine
Mineral Policy 2019 C) Afghanistan and Taliban group
D) Saudi Arabia and Houthi rebels
Internal Security Key : A
Role of external state and non- Explanation :
state actors in creating •. The United States and North Korea have failed to reach
challenges to internal security a formal deal at the Hanoi Summit. The talks ended
279) The Home Ministry has banned the Jamaat-i-Islami with no agreement after USA refused North Korean
as an unlawful association under Section 3 of the demand to lift all international sanctions.
Unlawful Activities (Prevention) Act, 1967. Consider •. The two-day meeting in the Vietnamese capital Hanoi
the following statements with respect to Unlawful had come eight months after their historic summit in
Activities (Prevention) Act, 1967: Singapore in June last year.
1. Its main objective is to make powers available for Objective :
dealing with activities directed against integrity and To know about power shifting politics in Asian region.
sovereignty of India. 2019-03-01
2. The Act helps to meet the commitments to the Economy
Financial Action Task Force (FATF).
Growth and Development
Which of the statement(s) given above is/are correct?
281) Activities which contribute to the gross national
A) 1 only B) 2 only product are called economic activities. Consider the
C) Both 1 and 2 D) Neither 1 nor 2 following statements about activities which
Key : C contribute to GNP?
Explanation : 1.Income of Ambassadors of other countries in India
•. UAPA is anti-terrorist law aimed at effective prevention 2.Women working at home also contribute to gross
of unlawful activities associations in India. national product.
•. Its main objective is to make powers available for 3.Income of people working abroad also contributes
dealing with activities directed against integrity and to gross national product.
sovereignty of India. Select the correct answer from the code given below:
•. It bans certain terrorist associations, punishes A) 1 and 2 only B) 2 and 3 only
membership and association with such organizations C) 3 only D) 1, 2 and 3
and punishes terrorist activities. The law been
Key : C
legislated to impose reasonable restrictions in
interests of sovereignty and integrity of India on Explanation :
exercise of freedom of speech and expression, to •. The simplified version of the official GNP formula can
be written as the sum of consumption by nationals,

Page No.97
MARCH CURRENT AFFAIRS_2019
government expenditures, investments by nationals, The law also contains cross border insolvency
exports to foreign consumers and foreign production provisions.
by domestic firms minus the domestic production by Four pillars of institutional infrastructure.
foreign firms. Insolvency Professionals:
•. Another way to calculate GNP is to take the GDP figure, a) They would play a key role in the efficient working of
plus net factor income from abroad. the bankruptcy process.
Objective : b) They regulated by “Insolvency Professional agencies”
To know about the Gross National Product (GNP) in Information Utilities:
India.
a) these would store facts about lenders and terms of
lending in electronic databases
Inflation b) This would eliminate delays and disputes about facts
282) Which of the following statements is/ are correct when default takes place
about consumer Price Index for Industrial workers ( Adjudication:
CPI- IW)?
a) The NCLT will be the forum where firm insolvency will
1. The base year for CPI - IW is 2001-02. be heard and DRTs will be the forum where individual
2. It is used to determine the dearness allowance of insolvencies will be heard.
the employees in both the public and private sector. Regulator:
Select the correct answer using the code given below a) “The Insolvency and Bankruptcy Board of India? will
A) 1 only B) 2 only regulate the Insolvency Professional, Insolvency
C) Both 1 and 2 D) Neither 1 nor 2 Professional agencies and information utilities.
Key : C Its impact:
Explanation : • With the addition of the Bankruptcy and Insolvency
•. Statement 1 is correct: The base year for CPI - IW is Code, the Indian lending system has gotten teeth in
2001-02. dealing with stressed borrowers and a difficult
•. It is generally argued that WPI inflation is not an macroeconomic situation.
appropriate index to determine the impact of price • Insolvency resolution is important for investments,
rise on the cost of living of the common man. Rather, whether Indian or foreign capital.
the Consumer Price Index for Industrial Workers (CPI- • Impact of the bankruptcy code will be widely positive,
IW), which also includes selected services and is leading to a better management of stressed Companies
measured on the basis of retail prices, and is used to in India.
determine the dearness allowance of employees in • There will be better financial discipline among
both the public and private sectors, is the appropriate companies, which will work to ensure that all their
indicator of general inflation creditors are paid on time. Code bestows significant
Objective : powers on the legal system to deal with stressed
To know about the CPI - Industrial workers (IW) companies;
• The powers have once again come into the hands of
Monetary Policy lenders as they are far above equity holders in the
payout structure that the code mandates in a
283) With reference to Insolvency and bankruptcy code, liquidation situation
consider the following statements
Objective :
1.National Company Law Tri bunal (NCLT) will
adjudicate insolvency resolution for individuals This question is intended to make student learn about
Insolvency and Bankruptcy Code, its institutional
2.Debt Recovery Tribunal (DRT) will adjudicate infrastructure (i.e. four pillars) and its impact.
insolvency resolution for companies.
Select the correct answer using the code below:
Infrastructure
A) 1 only B) 2 only
284) Consider the statements about city and Gas
C) Both 1 and 2 D) Neither 1 nor 2 Distribution (CGD) network?
Key : D
1. Petroleum and Natural Gas Regulatory Board
Explanation : (PNGRB) is the authority recognised for development
•. The Insolvency and Bankruptcy Code, 2016 is a major of City Gas Distribution network.
economic reform aimed at consolidating the existing 2. One of the objective of National Gas Grid was to
laws related to the insolvency of partnerships with develop CGD network.
unlimited liability, entities with limited liabilities Select the correct answer using the code given below
(including limited liability partnerships), and
individuals into a single legislation in order to rule A) 1 only B) 2 only
out the ambiguity in the insolvency resolution process. C) Both 1 and 2 D) Neither 1 nor 2

Page No.98
MARCH CURRENT AFFAIRS_2019
Key : C • cyanide fishing
Explanation : •. The Largest Coral Reef Island in the world in The Great
•. Under the Petroleum and Natural Gas Regulatory Barrier Reef acts like wall to Eastern Australia.
Board Act, 2006 Petroleum and Natural Gas Regulatory Objective :
Board (PNGRB) has been recognised as the authority To know about the Coral Reefs and their bleaching
to grant authorization to the entities for the triggers.
development of City Gas Distribution (CGD) network
in Geographical areas.
International Relations
•. Petroleum and Natural Gas Regulatory Board
International Institutions, agencies and their structure
identifies the geographical areas for authorizing the
and mandate
development of the CGD network in synchronization
with the development of natural gas pipeline 286) Consider the following statements with respect to
connectivity. ‘Geneva Conventions’:
Objective : 1. Geneva Conventions are a set of international treaties
To know about the city and Gas Distribution Network. that ensure that warring parties conduct themselves
in a humane way with non-combatants as well as
with combatants no longer actively engaged in
Environment Ecology fighting.
Environmental Pollution and 2. civilians, medical personnel, prisoners of war, and
Degradation wounded or sick soldiers etc.. will come under Geneva
285) With reference to Coral reefs and their bleaching Conventions.
select the correct answer. Which of the statement(s) given above is/are correct?
1. Coral bleaching occurs when Coral polyps expel A) 1 only B) 2 only
algae that live inside their tissue. C) Both 1 and 2 D) Neither 1 nor 2
2. The largest coral reef island is located in Key : C
Lakshadweep Islands group. Explanation :
3. Cyanide fishing is one of the trigger for bleaching •. The 1949 Geneva Conventions are a set of
of the coral reefs international treaties that ensure that warring parties
Select the correct answer using the code given below conduct themselves in a humane way with non-
A) 1 only B) 1 and 3 only combatants such as civilians and medical personnel,
C) 2 and 3 only D) 1, 2 and 3 as well as with combatants no longer actively engaged
Key : B in fighting, such as prisoners of war, and wounded or
sick soldiers. All countries are signatories to the
Explanation :
Geneva Conventions. There are four conventions, with
Coral bleaching occurs when coral polyps expel three protocols added on since 1949.
algae that live inside their tissues. Normally, coral
Objective :
polyps live in an endosymbioticrelationship with this
algae crucial for the health of the coral and the reef.[1] To know about the details of Geneva Conventions
The algae provides up to 90% of the coral’s energy.
Bleached corals continue to live but begin to starve Geography
after bleaching. Some corals recover. Map & current affairs based
Above-average sea water temperatures caused by questions
global warming is the leading cause of coral 287) Recently International Court of Justice (ICJ)’s verdict
bleaching. on ‘Diego Garcia’ is in news. It is geographically
List of triggers: located in which of the following islands of Indian
• increased water temperature (most commonly due to Ocean?
global warming), or reduced water temperatures. A) Maldives B) Seychelles
• oxygen starvation caused by an increase in C) Mauritus D) Kerguelen Islands
zooplankton levels as a result of overfishing.. Key : C
• increased solar irradiance (photosynthetically active Explanation :
radiation and ultraviolet light)
•. Mauritius was the colony of erstwhile of British
• increased sedimentation (due to silt runoff). Empire. With the calls of decolonization grew
• bacterial infections. stronger, the British granted independence to the
• changes in salinity. Island nation Mauritius.
• herbicides. •. But the British cut off the Chagos Archipelago from
• extreme low tide and exposure. Mauritius in 1965 before granting it independence in
1968.

Page No.99
MARCH CURRENT AFFAIRS_2019
•. Also, the residents of the Chagos Archipelago were 289) Which among the following is a part of Government
forcibly removed and the Chagos Archipelago was of India’s Digital India initiative, which aims at
handed over to the US. empowering the citizen regarding pharmaceuticals
•. The US established its strategic Indian Ocean military through a transparent, accountable and responsive
base at the Chagos Archipelago. governance system?
•. Both the UK and the US had vehemently opposed the A) Scheme for Development of Pharmaceutical Industry
UN resolution which referred the matter to ICJ saying B) Pradhan Mantri Bhartiya Janaushadhi Pariyojana
it was a bilateral matter with Mauritius. UK is not C) Pradhan Mantri Jan Aushadi Samadhan Yojana
represented in the ICJ Bench and the sole dissent D) Pharma Jan Samadhan
judgement was by the American Judge. Hence it is
Key : D
unlikely that the UK would accept the advisory. The
possibility of any challenge to the US Diego Garcia Explanation :
base from Mauritius is also unlikely. Hence by and •. The Pharma Jan Samadhan (PJS) initiative is part of
large status quo would be maintained. Goverment of India’s Digital India initiative, which
Objective : aims at empowering the citizen through a transparent,
accountable and responsive governance system.
To know about recent controversy around Chagos
Archipelago •. The PJS provides the consumer with an effective and
time bound grievance redressal system to effectively
deal with complaints related to pricing, shortage and
Governance non-availability of medicines. Apart from the internet-
Government policies and based online facility, there is a consumer Help Line
programs also, which can be used to lodge complaints.
288) Which among the following are the components of Objective :
“Scheme for Development of Knitting and Knitwear To know about initiatives taken by the government in
Sector” launched by the Union Textile Minister? the betterment of the pharma industry in India
1. Modernization and upgradation of existing power
loom service Centers (PSCs). 290) Which among the following statements about Scheme
2. Yarn bank scheme. for Higher Education Youth in Apprenticeship and
3. Solar Energy Scheme. Skills (SHREYAS) is incorrect?
Select the correct answer using the code given below: A) Ministry of Skill Development and
A) 1 and 2 only B) 1 and 3 only Entrepreneurship(MSDE) operates the National
C) 2 and 3 only D) 1, 2 and 3 Apprenticeship Promotion Scheme(NAPS) programme
through National Skill Development Council (NSDC).
Key : D
B) One of the primary objective of this scheme is to
Explanation :
establish an ‘earn while you learn’ system into higher
•. The main components of the Scheme for Development education.
of Knitting and Knitwear Sector are:
C) This scheme is mainly targeted at higher school
students to train them in different skills.
1. Creation of new service centers on Public Private D) The primary scheme will be operated in conjunction
Partnership (PPP) model by industry and association with National Apprenticeship Promotion Scheme
in the knitting and knitwear clusters. (NAPS).
2. Modernization and upgradation of existing power loom Key : C
service Centers (PSCs) and institution run by Textile Explanation :
Research Associations (TRAs) and Export Promotion
Councils (EPCs) Association in knitting and knitwear •. Role of MSDE (NSDC): MSDE operates the NAPS
clusters. programme through NSDC. They would not only
monitor the programme, progress of the apprentices,
3. Group work shed scheme. but would finance the programme by disbursing the
4. Yarn bank scheme. claims from the business enterprises towards stipend
5. Common facility centerscheme. reimbursement as per the NAPS.The ongoing efforts of
6. Pradhan mantra Credit Scheme. the SSCs would be monitored by MoSDE, which would
7. Solar Energy Scheme. also periodically introduce new SSCs into the SHREYAS
fold. The entire programme would progress with
8. Facilitation, IT, awareness,studies, surveys, market
dynamic interface & information sharing between
development and publicity for knitting and knitwear
MHRD and MoSDE.
units.
•. The primary scheme will be operated in conjunction
Objective :
with National Apprenticeship Promotion Scheme
To know about the initiatives taken by government to (NAPS) which provides for placing of apprentices upto
promote textile industry in India.

Page No.100
MARCH CURRENT AFFAIRS_2019
10% of the total work force in every business/industry. early intervention, development of daily living skills,
The scheme will be implemented by the Sector Skill education, skill-development oriented towards
Councils (SSCs) , initially the Banking Finance employability, training and awareness generation.
Insurance Services (BFSI), Retail, Health care, Telecom, With a view to inclusion of persons with disabilities
Logistics, Media, Management services, ITeS and in the mainstream of society and actualizing their
Apparel. More sectors would be added over time with potential, the thrust would be on education and
emerging apprenticeship demand and curriculum training programmes.
adjustments. Objective :
Following are the objectives of SHREYAS To know about the Deendayal Disabled Rehabilitation
1. To improve employability of students by introducing Scheme
employment relevance into the learning process of
the higher education system Issues relating to Health,
2. To forge a close functionallink between education and education and Human
industry/service sectors on a sustainable basis
resources
3. To provide skills which are in demand, to the students
292) Consider the following statements about Unani
in a dynamic manner
medicine.
4. To establish an ‘earn while you learn’ system into higher
education 1. It views disease as an imbalance of the substances
like blood, phlegm, yellow and black bile.
5. To help business/industry in securing good quality
manpower 2. Applying egg oil and almond oil for hair care is a
therapy under unani.
6. To link student community with employment facilitating
3. Central council of Indian Medicine monitors higher
efforts of the Government.
education in unani medicine.
•. The program aims to enhance the employability of
Indian youth mainly the degree non-technical 4. A foundation stone was laid for National Institute
graduates by providing ‘on the job work exposure’ of Unani Medicine at Ghaziabad recently.
and earning of stipend. Select the correct answer using the code given below
Objective : A) 1 and 3 ony B) 2 and 4 only
To know about the government steps to improve the C) 1, 2 and 4only D) 1, 2, 3 and 4 only
skill set of Indian Youth. Key : D
Explanation :
291) Which of the following statements is/ are correct • According to Unani medicine, management of any
about Deendayal Diasbaled Rehabilitation disease depends upon the diagnosis of disease. Proper
scheme(DDRS)? diagnosis depends upon observation of the patient’s
1. To encourage voluntary action for ensuring effective symptoms and temperament.
implementation of the persons with disabilities act, • Unani, like Ayurveda, is based on theory of the
1995. presence of the elements in the human body. According
2. The government provide financial assistance to to followers of Unani medicine, these elements are
voluntary organisations to provide necessary services present in fluids and their balance leads to health
to the disabled. and their imbalance leads to illness.
Select the correct answer using the code given below • The theory postulates the presence of blood, phlegm,
yellow bile and black bile in the human body.
A) 1 only B) 2 only
C) Both 1 and 2 only D) Neither 1 nor 2 • The Central Council of Indian Medicine (CCIM), a
statutory body established in 1971 under the
Key : C Department of Ayurveda, Yoga and Naturopathy, Unani,
Explanation : Siddha and Homoeopathy (AYUSH), monitors higher
• The objectives of Deendayal Disabled Rehabilitation education in areas of Indian medicine including
scheme (DDRS): Ayurveda, Unani, and other traditional medical
To create an enabling environment to ensure equal systems.
opportunities, equity, social justice and empowerment • For Skin care and hair care Unani systems is the most
of persons with disabilities. preferable hen compared to other medicine systems.
To encourage voluntary action for ensuring effective Applying of egg oil and almond oil ,which are the
implementation of the People with Disabilities (Equal Unani medicinal products ,is used for hair care
Opportunities and Protection of Rights) Act of 1995. • A foundation stone was laid recently to set up a
• The approach of this Scheme is to provide financial National Institute of Unani Medicine at Ghaziabad to
assistance to voluntary organizations to make promote quality research and development of the
available the whole range of services necessary for alternative medicine systems especially Unani
rehabilitation of persons with disabilities including Medicine.

Page No.101

Anda mungkin juga menyukai